NCLEX REVIEW

Ace your homework & exams now with Quizwiz!

The nurse cares for the client with radium implant. During the removal of the implant, it is most important for the nurse to do what? 1. Clean the radium implant carefully with disinfectant using long forcepts 2. Handle the radium carefully using forceps and rubber latex gloves 3. Document the date and time of removal together with the total time and implant treatment 4. Double bag the radium implant before the person from radiology removes it from the room

3 At no time should the nurse or client handle the radium, radiology department is responsible for handling implants

On the third postoperative day, a client develops a fever of 103.3ºF (39.6ºC) shivering and nausea. The primary healthcare provider writes these prescriptions. Which should the nurse do first? 1. Apply cooling blanket for fever. 2. Give ceftriaxone 1 gram IVPB stat. 3. Draw blood cultures. 4. Give promazine 50 mg po PRN for nausea.

3 Blood cultures MUST be drawn immediately to identify the causative bacteria. Once the organism is identified, the primary healthcare provider will order organism specific antibiotics. Always draw blood cultures before administering the antibiotic. If antibiotics are given before the blood cultures are drawn, the culture will be inaccurate, and the client cannot be treated appropriately. 1. Incorrect: Application of a cooling blanket is appropriate, but the key in this question is to "fix the problem" ASAP. To treat the infection, the blood cultures must be drawn ASAP and be done before starting the antibiotics. 2. Incorrect: Antibiotics are not given until the cultures have been drawn. Administering the antibiotic first would cause the culture to be inaccurate. 4. Incorrect: Preventing shivering is appropriate, but remember, always pick the answer that is most life-threatening. In this case, treating the bacteria as soon as possible is the priority answer. This requires the culture be obtained ASAP so the antibiotic therapy can be initiated.

Pregnant client admitted with hypertension. Reports her wedding band is tight. the nurse assesses which indication of mild pre- eclampsia? 1. Blurred vision and proteinuria 2. Epigastric pain and headache 3. Facial swelling and proteinuria 4. Polyuria and hypertonic reflexes

3 Blurred vision appears later choice 2 is signs of eclampsia before a seizure. Oliguria is seen later with eclampisa

A client who just got a TURP. Has continuous bladder irrigation (CBI). the spouse asks why the client has a CBI. Which response is best? 1. CBI prevents urinary status and infection 2. CBI dilutes the urine to prevent infection 3. CBI enables urine to keep flowing 4. CBI delivers medication to the bladder

3 CBI prevents formation of clots that can lead to obstruction and spams int he post op turp client

The client is transferred to the rehabilitation facility following ischemic stroke affecting the right side with aphasia. Which nursing action would promote communication with the client? 1. Encourage client to shake head in response to questions 2. Speak in loud voice during interactions 3. Speak using phrases and short sentences 4. Encourage the use of radio to stimulate the client

3 Choice 1 prevents client from expressing themselves

client is 16 weeks gestation and has an amniocentesis. She asks what will be learned from this procedure. How should the nurse respond? 1. Tetralogy of Fallot 2. Talipes equinovarus 3. Hemolytic disease of the newborn 4. Cleft lip and palate

3 Choice 2 is club foot and detected at birth Cleft lip/palate is detected at birth Choice 1 is also detected at birth (pulmonary stenosis, ventricular septal defect, overriding aorta, hypertrophy or right ventricle

The parent of a school age child diagnosed with Type 1 DM reports the child has been sick and has ketones in the urine. Which instruction will the nurse provide to the parents? 1. Hold the next dose of insulin 2. Administer an additional dose of regular insulin 3. Encourage the child to drink caloric free liquids 4. Seek medical attention for additional assessment and treatment

3 Insulin should not be adjusted or changed. this is a medical decision If ketones are present, liquids are needed to aid in clearing them from kidneys Additional medical treatment is not needed at this time

The nurse provides care to a client who had a splenectomy 2 days ago. the nurse is also assigned a client diagnosed with pneumococcal pneumonia. which action should the nurse take first? 1. place the splenectomy client in reverse isolation 2. ensure that both clients receive broad spectrum antibiotics. 3. Request the charge nurse alter the client care assignment 4. Maintain strict aseptic technique while providing client care

3 It is unsafe to risk the transferring pneumococcal infection to at risk splenectomy client -Choice 1 is incorrect as the client typically needs just asepsis technique. Choice 4 would be if assignment can not be changed. but first ask to be reassigned.

A client with baseline decreased mental status is ordered to receive a tube feeding. The nurse knows the best position for the patient once the feeding has finished is: 1. Semi-Fowler's. 2. Left side with head of the bed elevated. 3. Right side with head of the bed elevated 4. High-Fowler's.

3 Placing the patient on the right side is ideal for digestion and having the head of the bed elevated will prevent aspiration.

A client admitted to the PACU after a thoracotomy that required prolonged anesthesia. The client develops noisy, irregular respirations and the oxygen saturation level drops. Which action does the nurse implement first? 1. Deliver breaths with a handheld resuscitation bag and mask 2. Auscultate the client's breath sounds 3. Tilt the client's head back and push forward on lower jaw 4. Increase the supplemental oxygen as prescribed.

3 Prolonged anestheisia cause a relaxed state, the client's lower jaw and tongue fall back and obstruct the airway, causing hypopharyngeal obstruction, the nurse opens the client's airway by tilting the client's head back and pushing the lower jaw forward. Auscultating the lung sounds at this point will not provide additional useful information

The nurse is performing CPR on an adult client with facial and neck trauma. Following the administration of rescue breaths, where is the best location for the nurse to assess for a pulse in this client? 1. Apical area 2. Carotid artery 3. Femoral artery 4. Radial artery

3 Pulses that are best palpated are large and close to the trunk of the body. The femoral artery is large and at the trunk (proximal) of the body. 1. Incorrect: It would take too long to use a stethoscope and listen for an apical pulse on the client. A quicker area of checking the pulse would need to be used. 2. Incorrect: The client has bilateral neck trauma. The carotid artery would not be the best site to assess for a pulse. 4. Incorrect: The radial artery is not as large as the femoral artery and is distal to the femoral artery.

Client returns from surgery. There is a fine, reddened rash around the area where providone iodine prep was applied prior to surgery. The nurse notation in the client's record includes which observation? 1. Time and circumstances under which the rash was noted 2. Explanation given to the client and family of the reason for rash 3. Notation on an allergy list and notification of the health care provider 4. The need for application of corticosteroid cream to decrease inflammation

3 Suspected reaction to substances should be reported to health care provider and noted on list of possible allergies

Which client does the nurse monitor for a heart block after a MI? 1. A client with a non-ST elevation MI 2. Client with a septal wall MI 3. Client with an inferior wall MI 4. Client with subendocardial wall MI

3 The SA node is supplied by the right coronary artery, is located in the inferior wall, and is the type of infarct to SA node that leads to heart block and the related bradyarrhythmia.

The nurse is teaching a client about foods containing tyramine which should be avoided while taking a monoamine oxidase inhibitor (MAOI). Which meal selection by the client would indicate understanding of an acceptable food to eat? 1. Smoked turkey and dressing, sweet peas and carrots and milk. 2. Baked chicken over pasta with parmesan sauce, baked potato and tea. 3. Fried catfish, French fries, coleslaw and apple juice. 4. Liver smothered in gravy and onions, rice, squash and water.

3 The following foods in the options listed above contain moderate to high levels of tyramine and should be avoided while taking MAOIs: smoked turkey, parmesan cheese, tea and liver.

A 70 year old client was admitted to the vascular surgery unit during the night shift with chronic hypertension. At 0830, the unlicensed nursing assistant (UAP) reports that the client's BP is 198/94. What would be the best action for the charge nurse to delegate at this time? 1. Ask the UAP to put the client back in bed immediately. 2. Tell the UAP to take the BP in the opposite arm in 15 minutes. 3. Have the LPN/LVN administer the 0900 furosemide and enalapril now. 4. Ask the LPN/LVN to assess the client for pain.

3 The nurse should recognize the need for measures to reduce the blood pressure. Administering the client's blood pressure medicine is aimed at correcting the problem. It is appropriate to administer the medications at this time in relation to the time that the next dose is due. 1. Incorrect: This is an appropriate action, but does not address the problem of lowering the client's blood pressure. 2. Incorrect: This is an appropriate action, but does not address the problem of lowering the client's blood pressure. 4. Incorrect: This is an appropriate action, but does not address the problem of lowering the client's blood pressure.

While assessing an adolescent for sore throat and fatigue, the nurse notes multiple wounds in different levels of healing on both of the client's arms. which action will the nurse take first? 1. Inquire as to how the wounds occurred 2. Ask about history of sexual abuse 3. Assess wounds for healing and signs of infection 4. Report the findings to the nurse supervisor

3 The physiological needs are a priority. the wounds need to be assessed for infection Although choice 1 is important, assessment of physical status is a priority.

A client diagnosed with schizophrenia who is taking monthly haloperidol injections develops slurred speech, shuffling gait and drooling. Which prescribed PRN medication would the nurse administer? 1. Lorazepam 2. Atropine 3. Benztropine 4. Chlorpromazine

3 These signs and symptoms are reflective of pseudoparkinsonism, a form of extrapyramidal side effects which are side effects of the haloperidol. An anticholinergic agent maybe used for treatment. This is an anticholinergic agent that may be used for extrapyramidal side effects.

The nurse is providing post-operative care to the craniotomy client. Hourly urinary output increases from 100 mL last hour to 500 mL this hour. What action by the nurse takes priority? 1. Elevate HOB 90 degrees 2. Auscultate apical pulse 3. Obtain a blood pressure 4. Assess Glasgow Coma Score

3 This is the best answer because we are "worried" this client is going into SHOCK due to diabetes insipidus. So, you better be checking a BP. This is a time where checking the BP is appropriate. If we "assume the worst" I better check a blood pressure. It could have dropped out the bottom. 1. Incorrect: This client is losing too much fluid. We worry about shock. Lower the HOB. 2. Incorrect: Checking the pulse is a good thing, but not as important as checking the BP. 4. Incorrect: If my client is going into shock, the highest priority is to assess the BP.

The client has just returned from electroconvulsive therapy (ECT) and is very drowsy. What is the position of choice for the nurse to place the client in until full consciousness is regained? 1. Supine 2. Fowler's 3. Lateral 4. High Fowler's

3 When someone is very sedated and not fully conscious, we want them on their side so the airway remains open and the secretions can drain. 1. Incorrect: No, the jaw will fall back, the tongue will block the airway, and the client will have airway obstruction, either partial or maybe even life-threatening. 2. Incorrect: No, if you sit a client up who is not fully conscious, the client's head tips forward and blocks the airway. 4. Incorrect: Again, head may fall forward and block airway.

The nurse is preparing a client for a renal biopsy. Which is most important for the nurse to assess prior to this procedure? 1. BUN and creatinine 2. NPO status and signature on consent 3. Bleeding time and coagulation studies 4. Serum potassium and urine sodium

3 Yes. Before you insert a needle into an organ for a biopsy, it would be best to know the client's bleeding time because there is a risk of bleeding when the biopsy is performed. 1. Incorrect: Although these are related to renal function, they do not impact the procedure itself. Therefore, they are not essential for the procedure. 2. Incorrect: Although both of these are carried out, they are not the priority over risk of bleeding. Always think what could be life threatening. 4. Incorrect: Although both serum potassium and urine sodium are related to renal function, they do not impact the procedure itself. Therefore, they are not essential for the procedure.

Client has terminal illness. the nurse knows that the client's initial denial and isolation will give way to the second stage. the second stage is characterized by which behavior? 1. acceptance 2. bargaining 3. anger 4. depression

3 acceptance is 5th stage depression is 4th bargaining is 3rd

Cortisol is responsible for which action? 1. preparing the body for "flight or fight" 2. Regulating calcium metabolism 3. converting proteins and fat into glucose 4. Enhancing musculoskeletal activity

3 choice 1 is epinephrine choice 2 is parathyroid hormone (PTH) choice 4 is norepinephrine

Contact transmission based precautions. Which personal protective equipment item does the nurse remove next after taking off the contaminated gloves? 1. mask 2. gown 3. eye goggles 4. foot covering

3 the gown is removed after the eye goggles. Gloves, goggles/face shield, gown, mask, hand hygeine

A charge nurse is planning care for several clients on the unit. Which activities can the nurse safely delegate to an unlicensed assistive personnel (UAP)? Select all that apply 1. Administer a nebulizer treatment to a client diagnosed with pneumonia. 2. Obtain vital signs on a postoperative client who required naloxone 5 minutes ago. 3. Report a urinary output (UOP) less than 50 ml/hr on a post-op client. 4. Assist a client with obtaining a clean catch urine sample. 5. Remove an indwelling urinary catheter from a client.

3, 4 3., & 4. Correct: A UAP can report the amount of UOP but cannot interpret it. A clean catch urine sample is a noninvasive procedure. The UAP can assist the client to obtain the clean catch urinary sample. Both activities are the right person and right task of delegation. 1. Incorrect: A UAP cannot administer medications. This is the wrong task for an UAP.2. Incorrect: The client received naloxone to reverse the action of an opioid medication. A UAP should not be assigned to obtain vital signs on an unstable client. This is the wrong person to perform removal of an indwelling urinary catheter. 5. Incorrect: A UAP cannot remove an indwelling urinary catheter.

A client is admitted to the medical unit with a diagnosis of Addison's disease. What nursing interventions should the nurse implement for this client? Select all that apply 1. Monitor for decreased potassium level 2. Assist to select foods low in sodium 3. Administer fludrocortisone as prescribed 4. Monitor intake and output 5. Record daily weight

3, 4, 5

A client on naproxen. The nurse should assess the client for which symptoms? 1. Stomatitits and photosensitivity 2. bradycardia and dry mouth 3. Fluid retention and dizziness 4. gynecomastia and impotence

3. NSAIDS can cause headache, dizziness, gastrointestinal distress, priuritis, rash.

Normal potassium levels? Potassium Excreted by __________. Potassium is inversely related to ___________.

3.5-5.0 Kidneys Sodium

Normal aPTT

30-40 seconds

The hospital bed is accidentally lowered onto the CDU, what should the nurse do? 1. Place chest tube in bottle of sterile water 2. place tube in a cup of water 3. place tube in water pitched 4. Do whatever it takes to re establish a water seal

4

The nurse is assessing a client diagnosed with kidney stones who just returned from extracorporeal shock wave therapy (lithotripsy). The client is supine in bed with indwelling catheter in place. which finding would be the BEST indicator that the treatment has been effective 1. Total absence of pain 2. Indwelling catheter is draining freely 3. Rebound tenderness is absent during abdominal assessment 4. Sand like sediment has settled in the bottom of the indwelling catheter bag.

4

A client receiving TPN. Which nursing diagnosis is the priority when planning care? 1. Risk for aspiration 2. Electrolyte imbalance 3. Impaired body image 4. Risk for infection

4 -Aspiration is associated with enternal feeding, NOT with TPN -Electrolyte imbalance is a risk, but infection is priority. The use of central line and the high glucose content of TPN makes the risk for infection priority nursing diagnosis

How to self administer nasal drops? 1. Occulude one nostril prior to instilling the drops 2. Store the medication vial in the fridge between doses 3. Shake the medication vial for several minutes before opening 4. Sit with the neck flexed backward for 5 minutes after instilling the drops

4 -Both nostrils need to be open and clear before instilling nasal drops -Room temperature -Vigorous shaking is not usually required

Correct way for LPN to administer enema? 1. places the solution 20 inches above the anus 2. adjust temperature of solution 3. inserts the tube 6 inches 4. positions client in left Sims' position

4 1. could result in rapid infusion and be painful and distention of the colon 2. not feasible during the administrative phase 3. tube should be inserted no more than 4 inches 4. allow solution to flow downward along the natural curve of the sigmoid colon and rectum. which improves retention of solution

The nurse will be admitting a client from the operating room following a left total pneumonectomy for adenocarcinoma. Which type of chest drainage should the nurse anticipate that the client will have? 1. Bilateral chest tubes. 2. One chest tube on the operative side 3. Two chest tubes on the operative side 4. No chest drainage will be necessary.

4 4. Correct: A total pneumonectomy means the excision of the entire lung. A drainage tube is not inserted, since the fluid and air must accumulate in the thoracic space. This is to prevent mediastinal shift to the left. 1. Incorrect: A total pneumonectomy is removal of entire lung. The presence of fluid and air in the left thoracic cavity decreases the incidence of left mediastinal shift. 2. Incorrect: The entire lung is removed. The left thoracic space should fill with fluid and air to prevent mediastinal shift. The insertion of a chest tube is not warranted. 3. Incorrect: Entire lung is removed and no chest drainage is needed. The insertion of two chest tubes is not required. Fluid and air should fill the left pleural space to reduce the risk of a mediastinal shift.

The nurse admits a child with a history of cystic fibrosis (CF) with vomiting for 3 days, headache, and unusual behavior. What does the nurse anticipate the lab values will show? 1. Hypernatremia 2. Hypercalcemia 3. Hypocalcemia 4. Hyponatremia

4 4. Correct: CF kids are always losing sodium! That's why mom will often say they taste "salty" when she kisses them and why the sweat chloride test is diagnostic. They are looking for sodium chloride when they do the test. So when they are sick their risk for becoming hyponatremic goes up even more because they are stressed. And we know the brain doesn't like for our Na to be messed up. Did you pick out these Key words: Vomiting x3 days, headache, and unusual behavior.

What is most important for the nurse to have at the client's bedside when inserting a large orogastric tube for rapid gastric lavage? 1. Emesis basin 2. Portable x-ray machine 3. Oxygen 4. Suction equipment

4 4. Correct: Insertion of a large orogastric tube designed for rapid lavage often causes gagging and vomiting, so suction equipment should be readily available to reduce the risk of aspiration. Maintaining the client's airway is the priority. 1. Incorrect: You would need an emesis basin because of the chance of vomiting, but suction equipment is the priority due to aspiration. 2. Incorrect: An x-ray is the preferred method to check initial placement, once the tubing is inserted. Suction equipment is the priority when inserting the tube due to risk of aspiration. 3. Incorrect: There are no key words in the question to suggest the client needs oxygen at this time.

A nurse is caring for a client who has developed ventricular fibrillation. Where should the nurse place the conductive electrodes for maximum defibrillation effectiveness? 1. The left lower sternum and the right side of the thorax in the midclavicular line. 2. On the right shoulder and the left side of the sternum just below the rib cage. 3. The left upper chest to the left of the sternum and the lower right half of the sternum. 4. Below the right clavicle to the right of the sternum and just below the left nipple.

4 4. Correct: One electrode should be placed just below the clavicle to the right of the sternum, and the other electrode placement is on the left side just under the left nipple (pectoral area) and in the midaxillary line. 1. Incorrect: The position of these electrodes is not the recommended and acceptable placement for optimal defibrillation. The positioning of the electrodes is not anatomically correct to allow maximum delivery of the current through the heart. 2. Incorrect: The position of these electrodes is not the recommended and acceptable placement for optimal defibrillation. The positioning of the electrodes is not anatomically correct to allow maximum delivery of the current through the heart. 3. Incorrect: The position of these electrodes is not the recommended and acceptable placement for optimal defibrillation. The positioning of the electrodes is not anatomically correct to allow maximum delivery of the current through the heart.

What is most important for the nurse to do prior to initiating peritoneal dialysis? 1. Aspirate for placement. 2. Have the client void. 3. Irrigate the catheter for patency. 4. Warm the dialysate fluid.

4 4. Correct: The peritoneal fluid is inserted into the abdominal cavity. To promote the exchange of wastes and fluid through the peritoneal membrane, the peritoneal fluid should be warmed. This will promote vasodilation of the capillaries in the peritoneal cavity. 1. Incorrect: The peritoneal catheter should not be aspirated. This would not tell you anything and could irritate the peritoneal membrane. After the cover of the dialysate fluid is removed, the tubing should be connected to the peritoneal catheter. 2. Incorrect: This is not a bad choice, just not the most important. Voiding would make the client more comfortable during the procedure but will not affect the success. 3. Incorrect: It is not necessary to irrigate a peritoneal catheter because you are irrigating with the dialysate.

A client experiencing chest pain is prescribed an intravenous infusion of nitroglycerin. After the infusion is initiated, the occurrence of which symptom would prompt the nurse to discontinue the nitroglycerin? 1. Frontal headache 2. Orthostatic hypotension 3. Decrease in intensity of chest pain 4. Cool, clammy skin

4 4. Correct: This assessment finding of cool, clammy skin is an indication of decreased cardiac output that could be the result of too much vasodilatation. Cardiac output could continue to decrease if the nitroglycerin is not discontinued. 1. Incorrect: A headache is an expected common side effect of nitroglycerin administration. The headache is treated with medication. 2. Incorrect: A decrease in blood pressure when rising from a supine or sitting position is a common effect of the vasodilatation that occurs with the administration of nitroglycerin. The client should be advised to change positions slowly. 3. Incorrect: The decrease in the intensity of the client's chest pain is the desired outcome of the nitroglycerin administration.

The nurse is caring for a client with myasthenia gravis. What is essential for the nurse to teach this client regarding treatment? 1. Frequent low-calorie snacks. 2. Strict monitoring of intake and output. 3. Use of sweeping gaze when walking. 4. Setting the alarm clock for medication times.

4 4. Correct: Yes! Medication must be taken on time. Too early can cause weakness and too late can cause extreme weakness to point of paralysis. 1. Incorrect: No, the client needs frequent high calorie snacks. 2. Incorrect: No, this is not a cardiac or renal client. 3. Incorrect: No, this is done when the client has homonymous hemianopsia.

Buerger's disease. The clinic nurse obtains a health history. which is expected symptom with this? 1. Heart palpitations 2. Dizziness when walking 3. Blurred vision 4. Digital sensitivity to cold

4 A condition in which the blood vessels, especially those supplying the legs, are constricted whenever nicotine enters the bloodstream, the ultimate result being gangrene and amputation.

Which client admitted to the emergency department should the nurse assess first following shift report on assigned clients? 1. Client reporting inability to void and a distended bladder on palpation. 2. Client diagnosed with a confirmed closed fracture of the tibia. 3. Client who has a suspected corneal laceration. 4. Client with abdominal discomfort and a rigid abdomen on palpation.

4 A rigid abdomen may indicate bleeding or other causes of peritonitis which takes priority over the other three, more stable clients. This could lead to shock in this client. Conditions requiring immediate treatment include cardiac arrest, anaphylaxis, multiple trauma, shock, poisoning, active labor, drug overdose, severe head trauma, and severe respiratory distress. 1. Incorrect. Although this condition may be uncomfortable and could lead to renal problems if not resolved, it does not take priority over a client who is bleeding. 2. Incorrect. This person is likely experiencing pain, but this client does not take priority over a client who has peritonitis and may be going into shock from bleeding or third spacing into the peritoneum. Remember, pain never killed anyone. 3. Incorrect. This client with a corneal laceration would be experiencing pain and needs attention to avoid vision loss. However, this client does not take priority over a client who has peritonitis and may be going into shock from bleeding or third spacing in the peritoneum. Remember, ascites is fluid in the peritoneal cavity.

Which finding during a newborn examination requires immediate action? 1. Left side of the newborn's face is drooping 2. Newborn's uvula has two loves 3. Newborn's ears are low set bilaterally 4. Red reflex is absent in the newborn's right eye

4 Absent of red reflex indicates ophthalmic emergency. Which can cause blindness Bifurcated uvula indicates there may be a cleft in the palate. Facial dropping may indicate facial paralysis from cranial nerve VII. this usually resolves within a few days to 3 weeks.

A client was admitted to the medical unit with pneumonia 2 days ago. There is a history of drinking 5-6 martinis every night for the past 2 years. Today, the nurse notes that the client is disoriented to time and place and is seeing imaginary spiders on the ceiling. The nurse cannot understand what the client is saying. What is this client most likely experiencing? 1. Wernicke's Encephalopathy 2. Korsakoff's Psychosis 3. Alcohol Withdrawal 4. Alcohol Withdrawal Delirium

4 Alcohol Withdrawal Delirium usually occurs on the second or third day following cessation of or reduction in prolonged, heavy alcohol use. Symptoms are the same as for delirium: Difficulty sustaining and shifting attention. Extremely distractible; disorganized thinking; rambling, irrelevant, pressured, and incoherent speech; impaired reasoning ability; disoriented to time and place; impairment of recent memory; delusions and hallucinations. 1. Incorrect: Wernicke's Encephalopathy represents the most severe form of thiamine deficiency in alcoholics. Symptoms include paralysis of the ocular muscles, diplopia, ataxia, confusion, somnolence, and stupor. If thiamine replacement therapy is not given, death will ensue. 2. Incorrect: Korsakoff's Psychosis is identified by a syndrome of confusion, personality changes, loss of recent memory, and confabulation (filling in some memory gaps with different life events or created thoughts). It is frequently encountered in clients recovering from Wernicke's encephalopathy. Coordination may be affected, so the client may have difficulty maintaining balance. Treatment is parenteral or oral thiamine replacement. 3. Incorrect: Alcohol withdrawal typically begins 4-12 hours after cessation of or reduction in heavy and prolonged alcohol use. Symptoms include: coarse tremor of hands, tongue, or eyelids; nausea and vomiting; malaise or weakness; tachycardia; sweating; elevated blood pressure; anxiety; depressed mood or irritability; transient hallucinations or delusions; headache; and insomnia

A client on promthazine. client reports nausea and vomiting. After receiving the medication client reports dizziness when stand up. what action does the nurse take first? 1. Notify provider 2. monitors severity of symptoms 3. instructs client to ask for assistance before ambulating 4. Assess hydration status

4 Assessment first. Adverse effect includes anorexia, dry mouth and eyes, constipation, and orthostatic hypotension. client is at risk for dehydration due to vomiting, which exacerbates the orthostatic hypotension

Determine which client is at risk for developing problems with hearing based off their medication? 1. Client receiving spironolactone and cefaclor 2. receiving metformin and alendoronate 3. Paroxetine and cholestyramine 4. Furosemide and indomethacin

4 Both drugs are considered ototoxic

A female client taking captopril for hypertension tells the clinic nurse that she is planning to get pregnant. What recommendation should the nurse make? 1. "Captopril can be taken safely during pregnancy, but we will need to decrease your dose so you do not become hypotensive." 2. "We will need to increase your dose of captopril once you become pregnant." 3. "In order to prevent neural tube defects, start taking folic acid." 4. "Captopril can cause serious harm to an unborn baby, so you must prevent pregnancy while taking this medication. "

4 Captopril should not be taken during pregnancy because serious harm (possibly fatal) to the unborn baby can result when taken during pregnancy.

A client in a psychiatric unit sings over and over, "It is hot, I am a hot tot in a lot, I sit all day on a cot drinking a pop." How should the nurse document this form of thought? 1. Neologisms 2. Dissociation 3. Fugue 4. Clang Association

4 Clang association involves the choice of words governed by sounds, often taking the form of rhyming even though the words themselves don't have any logical reason to be grouped together. 1. Incorrect: The psychotic person invents new words, or neologisms, that are meaningless to others but have symbolic meaning to the psychotic person. 2. Incorrect: Dissociation is the splitting off of clusters of mental contents from conscious awareness. It is a mental process that leads to a lack of connection in the client's thoughts, memory and sense of identity. In its mild form, it is similar to day dreaming. In a more severe form, it can be manifested as multiple personalities. 3. Incorrect: Fugue is sudden, unexpected travel away from home or customary place of daily activities, with inability to recall some or all of one's past. The person is unaware that anything has been forgotten. Following recovery, there is no memory of the time during the fugue.

Client with gastric ulcer. Nurse anticipates the client will have pain at which time? 1. Two to three hours after a meal 2. during the night 3. Prior to ingestion of food 4. One half to 1 hour after meal

4 It does not help pain to ingest food, such as in duodenal ulcer. All other choices are seen in duodenal ulcers

The nurse is teaching a diabetic client who has been prescribed Lispro insulin about avoiding hypoglycemia. What administration teaching is priority? 1. Take insulin 30 minutes before bedtime 2. Take insulin twice daily in AM and PM 3. Take insulin one hour before meals 4. Take insulin with meals

4 Lispro is a rapid-acting insulin that should only be taken with food or within 15 minutes of a meal. 3. Incorrect: Lispro is a rapid-acting insulin and should not be taken without food. Giving an hour before eating is too early and would put the client at risk for hypoglycemia.

The adult client is preparing for a plasma cholesterol screening. Which instruction does the nurse give to the client? 1. eat a vegetarian diet for 1 week before test 2. limit alcohol to two glasses of wine a day before test 3. abstain from dairy products for 48 hours before the test 4. only take sips of water for 12 hours before the test

4 Only sips of water are permitted 12 hours before test to achieve accurate results. client should eat a normal diet the week before the test, alcohol should be avoided and intake will interfere with test results

The nurse is proving care for several clients. which client should the nurse address first? 1. Client with a stroke needing a hand splint reapplied 2. Client with DM and a fasting blood glucose of 78 mg/dL requesting a snack 3. Client with diarrhea needing the bedside commode emptied 4. Client with emphysema requesting assistance with ambulation

4 The client with emphysema has breathing difficulties and wants to ambulate. This person needs to be monitored closely when ambulating and should be the highest priority. Choice 2 is second completion. Choice 1 is third, and choice 3 is last

Best method for the nurse to use when evaluating the effectiveness of tracheal suctioning? 1. Notes subjective date, "my breathing is much improved now" 2. Notes objective findings, such as decreased respiratory rate and pulse 3. Consults with respiratory therapy to determine effectiveness 4. Auscultates the chest for change or clearing of adventitious breath sounds

4 This is the best choice. choice 2 can be correct, but not the most effective

The nurse enters a patient's room and observes via the monitor that their vital signs are as follows: blood pressure 80/42, heart rate 118, respirations 32. The nurse will place the patient in which of the following positions? 1 HOB elevated 30 degrees, head midline, pillow under the patient's knees. 2 Trendelenburg position. 3 Reverse trendelenburg position. 4 Head slightly elevated and midline, knees straight, all four extremities elevated on pillows.

4 This patient's vital signs indicate s/he is going into shock. The correct shock position on NCLEX is all extremities elevated on pillows.

A client diagnosed with a duodenal ulcer is prescribed lansoprazole and sucralfate. What should the nurse teach the client about how to take these medications? 1. Take together immediately before meals. 2. Take together immediately after meals. 3. Take the sucralfate first, wait at least 30 minutes, then take the lansoprazole. 4. Take the lansoprazole first, wait at least 30 minutes, then take the sucralfate.

4 When prescribed any medication along with sucralfate, the client should avoid taking the medication at the same time with sucralfate. Sucralfate can make it harder for the body to absorb lansoprazole because it forms a "coating" or "barrier" on the stomach lining. Therefore, the client should wait at least 30 minutes after taking the lansoprazole before taking sucralfate.

A child with impetigo. Parents are not taking care of child's skin properly. It is most important for the nurse to assess for which finding? 1. white patches on buccal mucosa 2. hearing loss 3. respiratory wheezing 4. Periorbital edema

4 White patches are associated with candidia. choice 4 is correct cause impetigo is caused by staphylococcus that if untreated can cause acute glomerulonephritis. Periorbital edeam is an indicator of that. Impetigo does not cause any respiratory problems

Newborn diagnosed with fetal alcohol syndrome. What characteristics are expected? 1. LGA, craniofacial abnormalities, hydrocephalus 2. Small head circumference, low birth weight, undeveloped cheekbones 3. Large head circumference, low birth weight, and excessive rooting and sucking behaviors 4. Normal head circumference. low birth weight, and respiratory distress syndomre

4 also may see feeding diffuclties and poor sucking abilities (eliminate choice 3)

Restraints: Order must be renewed every ____ hours for adults, every ___ hours for ages 9-17, and every hour for under 9 Check client face to face every ____ minutes. ** client could be harmed or die in restraints, so the need for accurate monitoring of this client cannot be overstated!!

4, 2, 15 minutes

A client with DKA. The health care provider prescribed IV insulin infusion. Which action does the nurse implement when administering the infusion? Select all that apply 1. Monitor blood glucose levels every 4 hours 2. Prepare the infusion of 100 units of intermediate acting insulin in 100 mL normal saline solution 3. Discontinue the insulin infusion as soon as blood glucose levels decrease 4. Monitor potassium levels closely 5. Administer the insulin infusion via an electronic infusion device

4, 5 Insulin infusion should be infused continuously until subcutaneous insulin administration can be resumed. Even if blood glucose levels decrease and return to normal, the insulin infusion should be continued until the start of subcutaneous insulin therapy

Findings that can indicate the nurse that the client has anorexia nervosa? Select all that apply 1. Diarrhea 2. Sensitivity to heat 3. Coarse hair covering most of the body 4. Amenorrhea 5. Muscle weakness

4, 5 NOT coarse hair, but lanugo (a fine, drowny hair covers most the body) May have constipation due to dehydration, and decreased peristalsis Sensitivity to cold due to decreased body fat

Client with stroke and facial paralysis. Nurse should plan to prevent which complication? 1. Inability to talk 2. loss of gag reflex 3. inability to open the affected eye 4. corneal abrasion

4, client will be unable to close eye voluntarily, the lacrimal gland will no longer supply sections to protect eye so eye drops and protection is needed. all other options may occur, nursing care cannot prevent it.

newborn with fetal alcohol syndrome. which action is important consideration for this newborn? 1. Prevent iron deficiency anemia 2. decrease touch to prevent over stimulation 3. provide feedings via gavage to decrease energy ependiture 4. replace vitamins depleted as a result of poor maternal diet

4-adequate intake of B complex vitamins is necessary for normal CNS function choice 1 is not highest priority, choice 2: infants need to be held and cuddled due to poorly developed CNS

A nurse is caring for a client diagnosed with HF. The client currently takes furosemide 40 mg every morning. potassium 20 mEq daily, and digoxin 0.25 every day. which client comment should the nurse assess first in caring for this client? 1. My fingers and feet are swollen 2. My weight is up 1 lb 3. There is blood in my urine 4. I am having trouble with my vision

4. High risk for dig toxicity with furosmide. Vision changes should be assessed

18 month old with larygotracheobronchitis. During initial assessment, the nurse expects to find which early symptoms? 1. Kassmaul respiration's and bradycardia 2. Elevated temperature and slow respiratory rate 3. Expiratory wheezing and substernal retractions. 4. Inspiratory stridor and restlessness.

4. determine how each question relates to croup. Stridor and restlessness characterizes edema and inflammation of upper airway choice 1 is for DKA choice 2 respiratory rate would increase choice 3 most often noted with respiratory distress of newborn.

The nurse is scheduled to administer the morning dose of Levothyroxine to a client with hypothyroidism. The client reports a "feeling of fullness" in her chest. What is the best action by the nurse? 1. Administer aluminum/ magnesium suspension 30 mL 2. Administer the Levothyroxine 3. Obtain a 12 lead ECG 4. Call the primary healthcare provider

4. Think MI will feeling of fullness in her chest Choice 3 may delay care

A Miller- Abbott tube is inserted for which reason? 1. provides an avenue fro nutrients to flow past an obstructed area 2. Prevents fluid and gas accumulation in the stomach 3. Administers medication that can be absorbed directly from intestinal mucosa 4. Removes fluid and gas from small intestine

4. provides intestinal decompression. often used to treat paralytic ileus

Acetaminophen should not exceed ______ mg/day

4000 for an adult

Diabetes both types: Carbohydrates should be _____ of diet Protein should be _____ Fats should be ______ ***Extremes in blood sugar = ________ damage High fiber slows down __________ absorption in the intestines, therefore, elimination the sharp rise/fall in blood sugar

45% carbs 15-20% protein 30-40% fats vascular damage glucose absorption

Nesirtide is a vasodilator an d should NOT be given for longer than _____ hours. REMEMBER turn it OFF 2 hours before drawing a ____ level

48 BNP

Pregnant client should go to the hospital when contractions are ____ minutes apart or when the membranes ___________.

5 minutes apart Rupture

First day post-op, a T-tube can drain anywhere between _____-______ mL. After the first day, it decreases and eventually tapers off.

500-1000

Heroin users may feel withdrawal affects ___ to ___ hours after stoppage Signs are agitation, anxious, mood, n/v, muscle aches, excessive ________, and __________ nose, sweating and pupil ___________

6-8 hours. tearing, running nose, pupil dilation

glycosylated hemoglobin (HbA1C): blood test given an average of what your blood sugar has been over past 3-4 months. _____% is diabetes people with diabetes the goal is ____%. what happens to blood sugar when you are sick or stressed, it ___________. Illness = ____ (increase insulin) -the normal pancreas can handle these fluctuations. an increase in blood sugar when sick or stressed, is a normal reaction to help fight the illness or stressor.

6.5% 7% Increases illness- DKA= increase insulin therapy

First trimester of pregnancy (week 1-13) -Increase protein intake to _____ grams per day -Weight gain expect ___ to ___ lbs in the whole 1st trimester -No high impact exercises, __________ and swimming are best ***DO NOT LET HR GET ABOVE _______. WHY? -No hot tubs are heating blankets, because if they get overhead it can increase body temperature and can cause ___________.

60 1-4 walking 140, decreases cardiac output and decreases uterine perfusion birth defects

When is pincer grasp developed? When does birth weight triple?

9 months 1 year

kidney stones "_______" S/S -Pain, WBCs in _________. HEMATURIA -Anytime you suspect a kidney stone, get a urine specimen ASAP and have it checked for ______. If stone is present the client will get ______ medication immediately

"Lithiasis" nephrolithiasis, urolithiasis, ureterolithiasis -urine -check RBCs -Pain medications

Pulse paradoxus

- A finding in which systolic BP is 10 mmHg or greater on expiration than inspiration. Pt also may present with JVD, bradycardia & hypotension. **A symptom of cardiac tamponade**

What three kind of fractures increase chances of shock or Fat embolism?? -signs of fat emboli: -_____________ or __________ over chest -conjunctival hemorrhages -____________ on chest x ray (one side of chest all white_

- Long bones (femur), Pelvic factures, crushing injuries -Rash or petechiae -Conjunctival hemorrhage -Snow storm

nephrotic syndrome: S/S: -Massive __________. -Hypo_________ -________ (anasarca) -Hyper___________ Treatment: -__________ for fluid retention -____________ to block Aldosterone secretion -____________ to decrease inflammation -Cyclophophamide decreases body's ____________ response (Immunosuppression patients) -____________ is a major complication of nephrotic syndrome

- Proteinuria -hypoalbuminemia -Edema -hyperlipidemia -Diuretics -ACE inhibitors -Prednisone -immune resonse -Infection

Appendicitis: - #1 thing to worry about is ??? S/S - _________ point. localized in the right lower quadrant -Do not give enemas or laxatives because you are worried about what??? -After surgery what is the position of choice??

- Rupture -McBurney's point -Perforation -Elevate HOB (fowlers), never want pressure on suture lines

What are the leading cause of death from accidental injury in infants up to one year of age? ____________ is leading cause or ER visits?

- Suffocation, motor vehicle related injuries, and drowning -Falls

For liver biopsy: What is the position Pre Op?? Then have then exhale and hold breath. WHY? Position Post op??

- Supine, right arm above head, arm out of the way -Gets diaphragm out of the way -Lie on right side- holds pressure to prevent bleeding, Monitor vital signs

In burns protection the GI system is important: -Why would magnesium carbonate, pantoprazole, or famotidine be prescribed? -Why would client be NPO with NG tube hooked to suction? -When you start GI feedings, what should you measure to ensure that supplement is moving through GI tract?

- To prevent Stress ulcer (Curlings ulcer) - Could develop illeus (paralysed GI), caused by decrease vascular volume, decreased GI motility, hyperkalemia - Gastic residual (return back to client) <50 mL is normal

Signs of RSV: -Begins with ____ -_______ discharge - Mild fever -wheezing -nonproductive cough -Tachypnea with _________ nares -Dyspnea and ____________ **Know the onset of signs, because RSV will become worse at days ___ to ___ and can progress to life threatening respiratory distress (s/s can mild to severe, can progress from simple cough and runny nose to severe respiratory distress)

- URI -Nasal -Flaring nares -retractions Day 2-3

Colostomy care -Does it form stool?? -Colostomy in the ascending and transverse- Semi ______ stools -Colostomy in the descending or sigmoid is semi formed or ______ stool. -Which ones do you irrigate?? ----Why irrigate this one?? (what is the best time??)

- Yes -Liquid -formed -Descending and sigmoid -For regularity to gain control. (same time everyday, after meals)

Signs of Hirschsprung's: -The presenting symptom is _______________. ----No nerves, no peristalsis in that section of the colon -______________ like stools that have a foul smelling. Treatment -_______________ to remove the portion of the bowel that is diseased.

- constipation -Ribbon like stools Surgery

A flail chest -occurs when? -______________ chest wall movement (see- ___ chest). Chest sucks inward on ___________ and puffs out on _____________

- multiple rib fractures -Paradoxical chest wall movement. Saw chest. sucks inward on inspiration, puffs out on expiration

Discontinue oxytocin when: -The contractions are too __________ -The contractions last too __________ -with any sign of fetal ___________ -Oxytocin is piggy backed into a mian IV fluid so when discontinuing oxytocin, do you turn off the main IV fluid???

- too often -too long -fetal distressed -No, keep main IV on

glomerulonephritis Patient education: -Diuresis begins ___ to ___ weeks after onset -_________ and ________ may stay in the urine for months -Teach s/s of Renal failure. Which are??? -Notify provider if symptoms are present

-1-3 weeks -Blood and protein -Malaise, headache, anorexia, n/v, decrease output, and weight gain

-Oliguric phase ends ___ to ___ days -Diuretic phase begins with output ___________ -Recovery phase- place client on increase _______ and __________. Resume activities as tolerated

-10-14 days -increase -potassium and calories

Non-stress test (NST) what do we want to see? Do we want it to be reactive or nonreactive?

-2 or more acceleration with 15bpm lasting 15 seconds for 20 min with return to baseline within 2 minutes. Reactive= accelerations are present **this test shows if baby can tolerate delivery

Burn safety: -Keep anything flammable at least ___ feet from heat source. -clean dryer lint screen after _______ use -IF toddlers are in home, ____________ should not be used - Set water heater NO HIGHER than ____*F. use anti scald devise

-3 feet -each use - table clothes should NOT be used -120*F

second trimester of pregnancy: -Add ______________ more calorie intake a day -If adolescent then increase by _____________ calories a day -weight gain is expect to be __________. (may be viable depending on BMI) -Should they still be experiencing: --Nausea and vomiting?? --Breast tenderness?? --Urinary frequency??

-300 -500 -1 lb/week -NO -YES -NO

Third trimester -Pregnancy is term if it advances to how many weeks?? -Weight gain per week?? -Fetal HR should be

-37-40 weeks -No more than one pound a week. If more than that, watch BP closely 110-160

Postpartum Hemorrhage (OB questions think hemorrhage) Early -more than ______ mLs of blood lost in first 24 hours ACCOMPANIED BY a ___% drop from admission hematocrit Late -can happen after ___ hours up to ___ weeks postpartum Causes: -Includes uterine _________, __________, retained ___________, and __________ delivery Medications used to halt excessive hemorrhage: -____________ stimulates uterine contractions -_____________ no for increased BP patients -_____________ not for asthmatic patients -_____________ (may cause spontaneous abortion if given in early pregnancy, but helps uterine contraction in hemorrhage)

-500 mLs, 10% hematocrit -24 hours up to 6 weeks. -uterine atony, lacerations, retained fragments, forceps deliver -Oxytocin -Methylergonovine maleate -Carboprost tromethamine -Misoprostol

What are presumptive signs of pregnancy?

-Amenorrhea (an abnormal absence of menstruation.)- often the first sign -Nausea- *morning sickness* -breast tenderness/ enlargement -Urinary frequency- can be one of the first signs as well. Uterus starts to stretch and press on bladder

What position should the client receiving oxytocin be placed?? -If client has any unreassuring fetal heart tones (ex. fetal bradycardia) then we put the client on her ___________ to enhance uterine perfusion. -What should be done with the infusion if late decelerations occur???

-Any position except supine/flat on her back -Left side -stop infusion

Parenting styles -____________ highly controlling, expected to be obeyed, inflexible rules -____________ sets reasonable limits on behavior, encourages growing autonomy of child, open communication -_____________ few or no restraints, unconditional love, much freedom, little guidance. no limit setting

-Authoritarian -Authoritative -Permissive

Acute Kidney Injury (AKI): Causes: -Pre Renal Failure: --- __________ can't get to the kidneys ---____tension ---Any type of _______ -Intra-Renal Failure: ---Damage has occurred ________ the kidney ---Glomerulonephritis or Nephrotic Syndrome ---Malignant ____________ which is uncontrolled and DM. ---Acute tubular necrosis= damage to the ____________ bodies of the kidneys. caused by hypotension, sepsis, or drugs that damage kidneys --- _____ used in tests such as heart cath and CT Scan --- Drugs such as "_____" antibacteria drugs and ______ type drugs -Post Renal failure ---_______ can not get out of kidneys ---Enlarged __________ --- Kidney stones, Tumors, ---Ureteral ___________ ---Edematous _______ (illeal conduit)

-Blood -Hypotension -shock -Damage inside the kidney -Malignant hypertension -filtering bodies -Dyes -"Mycin" and NSAIDS -urine -prostate -obstruction -Stoma

Treatment for Esophageal varices: -Replace the _______ loss. -Octreotide lowers BP in the ___________.

-Blood -liver

Hydrocephalus S/S: -___________ fontanels with head enlargement -Depressed or sunken _____ -Irritability and changes in the _____ -________ cry - Setting ____ sign. (seeing white all the way around the iris) -_______ circumference: always do at well child check ups until 3 years old Treatment -Insertion of a _____ shunt --Post op care ---Measure _____ circumference ---Assess for __________ fontanels and widening cranial suture line ---_________ position -Hydrocephalus is frequently associated with ____________. --When infant is born with a type of spina bifida, protect the ____ and do not let it rupture. cover with sterile moistening dressing so it does not dry out while waiting for surgery -The baby should be lying ______

-Bulging -Eyes -LOC -High pitched cry -Sun sign Head circumference -Insert a VP shunt ---measure head circumference ---Assess for bulging fontanels ---Supine position -Associated with myelomeningocele --protect the sac -lay prone position

TPN administration: -_______ line needed. Keep refrigerated and warm for administration -Taper it off because why?? -May have to start _________ for increase blood sugar - Blood glucose should be monitored every ___ hours -Check urine for ________ and _______ ----Why not protein?? -can only be hung for ___ hours -change tubing with ____ bag -Needs to be on ______ -most frequent complication - ____________

-Central line -to avoid hypoglycemia -insulin -6 hours -glucose and ketones -protein can not leak through the glomerulus unless there is kidney damage -24 hours -new bag -pump -infection

Nephrotic syndrome: Problems with protein loss: -Blood ________ (thrombosis) ---Losing protein that normally prevents their blood from clotting, -___________ cholesterol and triglycerides (liver trying to compensate) Causes of Nephrotic syndrome: -Bacterial and viral ____________. -__________ type medication

-Clots -High cholesterol and triglycerides -Infections -NSAIDS

AKI S/S -increase in ____________ and _____ -SG goes ______ -Retaining ___________ to decrease _________. It gets pulled from bones -_______ can occur due to not enough erythropoietin -__________ can also cause lethal arrhythmias -____________ (acid base imbalance) ----unable to filter or retain hydrogen or bicarb Treatment: - Prevents ___________ -Eliminate the _________ of the kidney injury -Bed rest to decrease ___________ and caloric need - Turn, cough, deep breath - Monitor intake and output -daily weight - monitor vital signs closely -1 kg= 1000 mL of fluid

-Creatinine and BUN -SG goes up (concentrated) -phosphate to decrease Ca -Anemia -hyperkalemia -metabolic acidosis -prevent complications -eliminate the cuase

Normal function of the liver: -____________ the body -Helps the blood to ______ The liver helps metabolize __________ -The liver synthesizes ___________

-Detoxifies -clots -drugs -albumin

Not enough ADH causes? Urine becomes _______, specific gravity _________ Blood is ______________, and Serum sodium levels will __________

-Diabetes Insipidus (may cause SHOCK) -loose fluid ("you DIurese with DIabetes insipidus) urine- dilute, SG goes down blood is cocentrated serum sodium increases

Hypomagnesmia causes: -________ lots of Mg in intestines -__________. this suppresses ADH and its hypertonic. So when not eating, drinking a lot, increases urination and more urination = too much mag loss - Muscle tone _________ -Seizure occur? Yes or No -Stridor and laryngospasm- airway is a ______ muscle - + chvosteck's and Trouseau's sign - Arrythmias? yes or no - DTR: _________ - how does the mind change? - ____________ problems due to esophagus being a smooth muscle

-Diarrhea -Alcohol -Muscle tone- rigid and tight -seizures- yest -smooth muscle -arrythmias- yes (heart is a muslce) -DTR: increases -mind changes- irrational - Swallowing problems

glomerulonephritis: S/S: -________ pain (costovertebral angle tenderness- CVA tenderness) -Urinary output will __________ -___________ (blood in the urine) -_________uria -BP will ________ -fluid volume _______ -Urine specific gravity will ___________ -Azotemia: abnormally high _____ and creatinine -Malaise and headache (toxins) Treatment: -Get rid of ______ or cause -I & O and daily __________ -Diuretics -monitor _____. -Restrict fluids ( Replacement is 24 hour fluid loss + ____ mL) -Balance activity with ______ Dietary needs: increase _______, decrease __________ and ________

-Flank Pain -UO will decrease -Hematuria -Proteinuria -BP increases -Fluid volume excess -USG will increase (concentrated) -BUN and creatinine= Azotemia -Get rid of strept -daily weight -monitor BP -500 mL -rest - increase carbs, decrease sodium and protein (MOST KIDNEY DISEASES YOU WOULD WANT TO DECREASE PROTEIN CAUSE PROTEIN INCREASES BUN)

Peptic Ulcers: -Common cause of ___ bleeding - can include __________, __________, and ____________ -Erosion is present S/S -_________ pain usually in mid epigastric area/back -Heartburn (dyspepsia)

-GI bleeding -esophagus, stomach, duodenum -Burning pain

Bleeding Esophageal Varices Patho?

-High blood pressure in the liver (portal hypertension) forces collateral circulation to form (extra circulation) -This circulation forms in 3 places: stomach, esophagus, and rectum -When you see an alcoholic client that is GI bleeding, it is usually esophageal varices and is usually not a problem until they rupture -High pressure in the liver and all extra circulation. Protruding vessels under pressure start dripping blood to the stomach and then the patient can vomit blood LIVER CAUSES PRESSURE THROUGH THE WHOLE GI TRACT

Pancreatitis: S/S: -Does Pain increase or decrease with eating?? -_________ like abdomen ---this means that bleeding can lead to ___________ -What is Cullen's sign? -If jaundice occurs we know the ________ is now involved -Hypotension = __________ or _______ because a decrease in vascular volume

-Increase -Rigid, board like abdomen ----can lead to peritonitis -Cullen sign= Bruising around umbilical area. Bruising around flank area is Grey-Turner's sign -Liver -bleeding, ascities

Medication alert on oxytocin: - Always _______ both the IV bag with an oxytocin sticker and IV tubing and ports -Only administer oxytocin with its own bag and close all _______ before you remove the tubing from the pump -Attach the oxytocin at the IV port ___________ to the site -When you discontinue the oxytocin, remove the IV bag and all __________ from the room

-Label -clamps -closest -tubing

Epidural anesthesia: -Position while getting anesthesia?? -given in stage 1 at _____ to ____ cm dilated -Does this usually cause headache?? -Major complication: _______________ --for this give _____________ or ___________ -Put in __________ position to prevetn vena cava compression. Vena cava compression will __________ venous return, which reduces Cardiac Output and blood pressure. If blood pressure is decreased, what happens to perfusion of the placenta?? -Alter positions ______________. -Don't forget to check __________ output and assess the bladder

-Lie on side, legs flexed; prop up over bedside table -3 to 4 cm dilated - NO headache -hypotenson -1000 mL of NS or LR to fight hypotension (may be given ahead of time) -semi folwers, tilted to their side; decrease venous return. Placenta perfusion will decrease -hourly -urine output

Client teaching with pregnancy: -_____________- usually occurs two weeks before term, the presenting part of fetus descends into pelvis, Client notices the ability to breathe more easily because pressure on diaphragm decreases, puts more pressure on bladder-so urination frequency is a problem again -___________ the largest presenting part is the pelvic inlet, hopefully the fetal head is presenting first -______________ measured in cm, measures the relationship of the presenting part of the fetus to the ischial spines of the mother.

-Lightening -Engagement -Fetal station

Ileostomy (colon has been removed) care -It is going to drain ________ all the time. -Should you irrigate ileostomies?? -Avoid foods hard to digest. they increase motility -Replace fluids often -At risk for kidney stones (always a little ___________)

-Liquid -NO -Dehydrated

What is probable signs of pregnancy?

-Positive pregnancy test-based on hCG levles -Goodell's sign (softening of cervix) -Chadwick's sign (bluish color of vaginal mucosa and cervix; 4 weeks) -Hegar's sign (softening of lower uterine segment; 2-3 months) -uterine enlargement -Braxton Hicks contractions -Pigmentation of skin (Linea nigra, the dark line down the center of abdomen, striae, darkening areola, facial chloasma/mask of pregnancy)

lumbar puncture: -How is the client positioned. and why? -CSF should be __________ and ____________. post-procedure -Lie flat or ____________ for 4-8 hours (seals the area) -Increase ___________ to replace lost spinal fluid and reduce risk of complications -What is the most common complication?? -Pain increases when patient ____________, and decreases when they lie down -How is headache treated??

-Propped up over the bedside table with head down. Want a lot of arch to the back. opens up space between vertabrae -clear and colorless -Prone -fluids -increase when sit up, decrease when lie down - Bed rest, fluids, pain meds, and a blood patch

In burns: -why does pulse increase??? -Why does CO decrease??? -Why does Urine output decrease??? -Why is epinephrine and norepinephrine secreted??? -Why are ADH and aldosterone secreted?

-Pulse- increases, anytime someone has FVD, pulse compensates by increaseing -CO: decreases, Less volume to pump out -UO: decreases, kidneys are trying to hold in fluid or are not being perfused adequately from decrease in CO. only 20 minutes can be enough for kidney damage -Epi and Norepi: secretion causes peripheral vasoconstriction, blood shunted to vital organs -ADH and Aldosterone: Aldosterone helps with retaining water and sodium, and ADH retains water. therefore the blood volume will go up (good thing)

lochia: ____________- color dark red. 3-4 days postpartum ____________-color pink brown. 4-10 days postpartum ___________- whitish yellow. 10-28 days (can last up to 6 weeks) Clots are okay as long as they are no larger than a _________.

-Rubra -serosa -alba nickle

hemothorax/pneumothorax s/s: -shortness of _________ -HR ___________ -Diminished breath sounds on the ___________ side -chest pain/cough _what will show up on an X ray? -_______________ is when air trapped in the tissue (usually neck, face, chest **should you ever pull out a penetrating object?? Treatment: -______________- removal of fluid with a needle -___________ tube -Daily chest _______

-SOB -increased HR -diminished breath sounds on *affected side* -chest pain -cough -Air or blood on x ray -*subcutaneous emphysema* -NO. NEVER pull out penetrating object. -Thoracentesis -chest tube -Daily chest x-ray

Cirrhosis Causes by _________ of the liver and BP in the liver goes up this is called _________ hypertension S/S -Liver will feel ________, nodular liver -Chronic dyspepsia (_____ upset) -Change in _________ function -Ascities -_____________ (immune system involved) -___________ ALT, AST - Progress to _____________ due to ammonia build up. acts like a sedative

-Scarring, portal hypertension -Firm, nodular liver -GI upset -Bowel -Splenomegaly (immune system involved) _Decrease serum albumin -Increase ALT, AST -progress to hepatic encephalopathy/coma (ammonia build up. acts like a sedative)

Dumping syndrome is usually secondary to gastric bypass, gastrectomy, or gallbladder disease Treatment: - Position _________ with meals - Lie down after meals on the ______ side. WHY? -NO ______ with meals (may have in between meals) -Avoid foods high in _______. WHY?

-Semi Recumbent -Left side. Left side= leaves it in. keeps food in. Right side= Releases it -NO drinks -Carbs. Carbs and electrolytes empty fast

Tension Pneumothorax S/S -_____________ emphysema -Absence of __________ sounds on one side -_______________thorax -Respiratory distress, cyanosis -Distended _________________ **Can be fatal as accumulating pressure compresses vessels, decrease venous return, decrease _____________ Treatment -Large bore needle is placed into the __________ space (by the primary healthcare provider) to allow excessive air to escape -Treat the cause (chest tube will be inserted)

-Subcutaneous emphysema -absence breath sounds -asymmetric thorax -Distended Neck veins -2nd intercostal space

open pneumothorax also known as _____________ wound -Opening through chest that allows air into __________ space. (gun wounds, stab wounds) Treatment: -Have the client inhale and hold or _________ or hmmmmm ----This will ________________ the intra-thoracic pressure so no more outside air can get into the body -Then place a piece of _______________gauze over the area and tape down on __________ sides. Four side acts like a _________ or flutter valve -Have client sit up if possible to ___________ lungs --Trauma clients stay flat, until evaluated for other injuries

-Sucking chest wound -Pleural space -valsalva -increase the intra-thoracic pressure -petroleum gauze, tape 3 sides, 3, 4th side acts like air vent -expand

tension pneumothorax Causes -Trauma -Too much ______ (from ventilator) -_________ a chest tube -insertion of ____________ line -Taping an open pneumothorax on all ____ sides without an air valve -_____________ has built up in the chest/pleural space and has collapsed the lung- ____________ pushes everything to the opposite side (mediastinal shift) ---Trachea will deviate on the _________ side

-Too much PEEP -Clamping -Central venous Line -All 4 sides -Pressure, presssure ---unaffected side

IBD (inflammatory bowel disease) -Ulcerative Colitis affects just the ___________ -Crohn's Disease is inflammation and erosion of the _______ (small intestine) but it can be found anywhere in the small or large intestine -what is rebound tenderness? Means what?

-UC= Large intestine -CD- Ileum push in, let go, it hurts. Means peritoneal inflammation

What should you have the client do first before leopolds maneuvers?? If the client is having contractions, should the Leopolds be done during or between contractions?

-Void -between

what age: Identity vs identity diffusion? Intimacy vs. isolation? Integrity vs despair vs disgust? Industry vs inferiority?

-adolescent - young adulthood (19-35) -65 and older - school age (6-12)

Magnesium Sulfate: -Acts like a ______________ -causes ____________ -vaso___________

-anticonvulsant -sedates -vasodilates

During hemodialysis two needles are inserted: ___________ end of the access will remove the blood and the return is through the low pressure ___________ end For temporary placement the internal ______ or _______ vein is often used for catheter placement because surgery is not required for temporary placement "feel the thrill, hear the bruit" (thrill= cat purring sensation that is palpated, Bruit is turbulent blood flow that is auscultated) NO blood pressure, needle sticks or constriction (purses, watches etc. keep arm lose and free)

-arterial, venous -jugular or femoral vein

Newborn care Immediate care -maintain _______ temperature -Apgar should be taken at ____ and ____ minute (HR, respiration, muscle tone reflex irritability, and color) -Want a score of ____ to ____ -_____________ an eye ointment. can be used because it will kill Neisseria gonococcus and Chlamydia -_____________ promotes formation of clotting factors. IM vastus lateralis Cord care - Dries and falls off in ___ to ___ days -Cleanse with each diaper change using ________ or _____ -Fold diaper _______ the cord -No immersion until cord falls off. watch for ____________

-body temperature -1 and 5 minutes. score of 8-10 -Erythromycin -Phytonadione (vitamin K) -8 to 14 days -alcohol or NS -below the cord -watch for infection

Assist with central line placement: -Do not start fluids until positive confirmation of placement by _______. -Position of patient should be ___________ to distend veins -If air gets in the line, what position do you have put the client in??? --when changing tubing, how can you avoid getting air into the line??

-chest x ray -trendelenburg -left side trendelenburg -clamp it off -Valsalva -take a deep breath and HMMMMMM

otitis media (OM) -Avoid __________ as it can make pain worse. -May require tympanostomy tubes, or PE (pressure equalizing) tubes. These keep the middle ear drained. they are temporary so if they ________, its okay --if they have PE tubes in their ears, they must wear ___________ in the bathtub or when swimming (anytime there is a risk of getting water in the ears)

-chewing -fall out earplugs

peritoneal dialysis - fluid should be _________ and ________ colored --If fluid is cloudy indicates ___________ What if all the fluid does not come out??

-clear or straw colored -infection Turn patient from side to side

Prevention of Otits media -Have baby sitting up for __________ -no bottle _______ -Gentle _______ blowing -Play"___________" Game -Avoid __________ of any kind

-feeding -propping -nose -blowing -smoking

When does pathologic jaundice occur? -Usually means __________ incompatibility? When does physiological jaundice occur? -this is caused by??

-first 24 hours -Rh/ABO -After 24 hours -due to both the normal hemolysis or excessive RBCs releasing bilirubin, and immaturity of the liver

hemodialysis (HD): -The machine is the ____________ (filter) -done 3-4 times a week, so watch food and fluid intake between treatments - To prevent blood clots from forming, a ____________ is given during diaylsis -_____________ and ____________ are watched constantly

-glomerulus -anticoagulants (usually heparin, bleeding percautions -electrolytes and blood pressure

AKI: Medications: - Loop diuretics and osmotic diuretics - Treat hyperkalemia with IV __________ and insulin -IV ____________ for dysrhythmias -Polystyrene sulfonate is also known as __________ -_________ binding drugs to prevent hypocalcemia Nutrition -increase _______ and ____ -Decrease ____________ -Avoid foods high in _______, ________ PREVENT ____________ . #1 cause of death

-glucose -calcium gluconate -kayxelate -phosphate -increase carbs and fats -decrease protein -avoid foods in phosphate and potassium PREVENT INFECTION

Hepatic Coma: -Change in ____________. -__________ is protein, so having GI bleeding increases ammonia levels -Liver people tend to be GI BLEEDERS

-handwriting -blood

Treatment for Larynogotracheobronchitis: -____________ shower -___________ temperature therapy: this helps _____________ the swollen blood vessels in the trachea that cause swelling If it gets worse and no improvement: -Nebulized ___________ or steroids may be prescribed and administered in the emergency department -Nebulized ______________ has a rapid onset. generally see improvement in 10-15 minutes. always watch for relapse and return of symptoms when it wears off

-hot or steamy shower -Cool (cool air outside, stand near freezer) -Epinephrine

Surgery with Ulcerative Colitis: Total colectomy is performed so an ______________ is formed Surgery with Crohn's disease -Removed the _______ area. -Client may have a _________ or ________ (just depends on the affected area)

-ileostomy -colonostomy, ileostomy

What is a contraindication of lumbar puncture? Can bacterial get into puncture site and spinal fluid?? what would this cause?

-increased ICP -yes, meningitis

why is epiglottis considered a medical emergency?? They will look worse than they ______. the less noise they make, the ________ the airway obstruction Never try to _________ the throat or ________ with a tongue depressor

-infection can lead to partial or full occlusion of the airway sound. less noise the worst the obstruction visualize the throat or tongue

Meningitis: -It is ____________ of the covering of the spinal cord and brain. can be bacterial or viral s/s -____________ rigidity -Severe ___________ -High ________ -______________ (where light hurts the eye) -A positive ________ and _____________ sign Treatment: -________________- to decrease inflammation of the brain and spinal cord -___________ if the infection is bacterial -____________ if seizures are present -____________ precaution for bacterial (bacterial is very contagious and can have high mortality if not treated) -Meningitis immunizations are recommended for ____________ -Viral meningitis is transmitted by feces and requires ___________ precautions. common in infants and children

-inflammation -Nuchal rigidity -severe headache -High fever -photophobia -Kernig (severe stiffness of the hamstrings, inability to straighten leg when hip is flexed 90*).. Brudzinski: severe neck stiffness causes hips and knees to flex when neck is flexed. -corticosteroids -antibacterial -anticonvulsants -Droplet -College students -contact

-A few hours after birth the uterus rises to ________ of umbilicus or the one finger breath above -Fundal height will descent ___________/day -what is involution?? -Afterpains are common for the first 2-3 days and will continue to be common if the mother chooses to do what??

-level of umbilicus -one finger breadth/day -When fundus descends and the uterus returns to its pre pregnancy size -breast feed

Hip Replacement Positioning -want _____________ rotation- toes pointed to the ceiling -Limit __________, want ___________ of hip -Abduction or adduction??_________ -What exercises can be done while client is still confined to bed?? -Avoid __________ the legs -Is it okay to sleep on operated side?? -Is hydration important with this client?? -Is it okay to give pain meds in the operative hips??

-neutral -limit flexion, want extension -Abduction -Isometric- squeezing muscle tone -crossing -no -yes -no

-Why do babies sometimes experience hypoglycemia after birth?? Babies at greatest risk for hypoglycemia include???

-not getting glucose from the mother -Babies that are LGA, SGA, and babies of diabetic moms

Client receiving Oxytocin: -In NCLEX world client need __________ care. BE ALERT for Complication: -____________ labor -Fetal ____________ -__________ rupture: increase chance of rupture includes a previous C section. VBAC moms are high risk -Want a contraction rate of 1 every __ to __ minutes with each lasting ____ seconds

-one to one care -hyertonic labor -fetal distress -Uterine rupture - 2 to 3 minutes with each lasting 60 seconds.

symptoms of intussusception -Sudden _______ -Drawing up of ______ -Episodes of pain that comes and goes -Classic symptom: Currant __________ stools sometimes an air contrast, ___________, or saline enema can be done. What does this help do?? After an enema or surgery, they still keep them in the hospital for several days because it might _____________.

-onset -knees -jelly enema. -Pressure of the enema going through the bowel with these procedures will push out the telescoped area -reoccur

Pancreatitis: Treatment -Control ______ (main goal) -Decrease gastric secretions (______, _____ to suction, bed rest) ---Want stomach _________ and ______. -Maintain fluid and electrolytes. May give _____ to give stomach a rest -Besides pain medication, why would anticholinergics be prescribed???

-pain -NPO, NG to suction -Empty and Dry -TPN - Dry up secretions

Emergency Delivery/ Precipitous Delivery: -Tell the client to ______ to decrease the urge to push -Elevate ______ -When fetal head is out, feel for ??? what is this called?? -_____ baby cause it cannot regulate its temperature. place baby on mother's abdomen

-pant/blow -HOB -Cord around the neck, nuchal cord -Dry

Goal post op after repair of cleft lip/palate is to ?? What position should the child be placed in?? For older children use a ______ diet until well healed. want to avoid disrupting suture line When is the best time for cleft palate repair to be done??

-protect the suture line -Supine or side lying following a cleft lip repair. DO NOT place in the prone position -Soft diet. avoid rough foods, utensils that may disrupt suture line -Before speech develops, usually between ages 1-2 years.

diet for nephrotic syndrome: -Moderate __________: 1-2 grams/kg/day -Client can become ___________ fast -Sodium should be ____________ -Lipid lowering drugs for hyperlipidemia -____________ therapy for up to 6 months -Dialysis Nursing considerations: - Daily __________ - I&O -Measure abdominal girth or extremity _____ -Good skin care

-protein -malnourished -decreased -anticoagulation -Daily weight -Extremity size

Mineralocorticoids (aldosterone): -Makes you retain __________ and ________. -Makes you lose ________

-retain sodium and water -lose potassium ***remember K and Na are inverse so increase Na is a decrease in K

Thoracentesis: Pre-procedure -Do you need a signed consent?? -Stop any ______________ medications -Be sure chest _______ has been performed. Positioning should be?? During procedure -Should client cough or deep breath?? -As fluid is removed, the lung should __________. -Check vitals, oxygen stats, and pain level. compare it to __________ Post-procedure -collect another chest _____ -Check puncture site and dressing for ________________ -Turn, cough and, deep breath

-yes -anticoagulant medications -x ray -Sit on edge of bed, with feet supported, and lean over the bedside table --If they can't sit up? Lie on unaffected side with HOB at 45* angle -NO. client should remain very still -expand -baseline X- Ray -bleeding

maintenance dose of Digoxin? what is Digoxin therapeutic levels?

0.125 to 0.5 mg 0.5-2. greater than 2.5 is very toxic

lithium levels

0.5-1.2

A child in early stage of nephrotic syndrome of nephrotic syndrome. the nurse discusses which dietary change with the parents? 1. adequate protein, low sodium intake 2. low protein, low potassium intake 3. low potassium, low calorie intake 4. limited protein, high carbohydrate intake

1

You are taking care of a 5 month old baby. what toy is most appropriate for the infant? 1. Rattle 2. Stuffed animal they can grasp 3. Puzzle wit big pieces 4. Nursery rhyme CD

1

The nurse observes a client's cardiac rhythm from sinus rhythm to ventricular fibrillation. Which action does the nurse take next? 1. Begin CPR 2. Administer prescribed intravenous amiodarone 3. palpate for a femoral pulse 4. Prepare for defibrillation

1 -CPR should be started while preparing for defibrillation -Amiodarone is antiarrhythmic medication. but not first priority for V fib -Early defibrillation is critical for survival. The change of survival decreases every minute defibrillation is delayed

The nurse provides care for client with pulmonary artery catheter. Which parameter does the nurse monitor to evaluate whether interventions to decrease afterload were effective? 1. Systemic vascular resistance 2. Central venous pressure 3. Cardiac output 4. Pulmonary artery occlusion pressure

1 -SVR reflects afterload (vascualr resistance) -CVP reflects right sided heart failure -PAOP reflects left end diastolic pressure, not afterload

A new mother brings her infant to the clinic for a well-baby checkup. While at the clinic, the mother asks the nurse if there are any reasons why her infant should not have the measles, mumps, rubella (MMR) vaccine. The nurse's response is based on evidence that the MMR vaccine is contraindicated under which condition? 1. A known allergy to gelatin. 2. A family history of autism. 3. In infants with diarrhea. 4. A known allergy to sulfonamides.

1 1. Correct: The MMR vaccine is grown using chicken embryos and manufactured with the use of gelatin. Known allergies to gelatin would be a contraindication for administration.2. Incorrect: The Centers for Disease Control does not recognize a link between the administration of the MMR vaccine and the development of autism.3. Incorrect: Diarrhea is not a contraindication specifically for the MMR vaccine. Diarrhea may result in hypovolumia and electrolyte imbalance which need to be addressed. 4. Incorrect: Sulfonamides are not used in the development of the MMR vaccine. Neomycin is used in the development of the MMR vaccine. Neomycin is the only antibiotic allergy that would contraindicate the administration of the MMR vaccine.

A client tells the nurse, "I am dying from cancer. I have told my primary healthcare provider that I do not want to be revived if my heart stops beating or I stop breathing." What action should the nurse take first to assure that the client's request is respected? 1. Ensure a do-not-resuscitate prescription has been provided. 2. Report client wishes during the end-of-shift report. 3. Have the client sign an advanced directive. 4. Ask the client who holds the durable power of attorney for health care decisions.

1 1. Correct: The nurse should check the medical record for a DNR order. By law, a person who does not have a do-not-resuscitate (DNR) prescription, must be provided CPR in the event of a cardiac/respiratory arrest. This action will ensure the client's end-of-life wishes have been communicated and will honor the client's wishes. 2. Incorrect: It is appropriate to report the client's end-of-life wishes to other care givers, but not before ensuring a DNR order is in place. 3. Incorrect: If the client has advance directives, a copy should be placed in the medical record. However, a DNR prescription must also be in place to ensure the client is not resuscitated. 4. Incorrect: The client's request can be initiated by notifying the primary healthcare provider. It would be helpful for the client to have a durable power of attorney

a client on heparin. A recent activated PTT is 55 seconds. Which action should the nurse take? 1. Document the results and administer the heparinb 2. Withhold the heparin 3. Notify the provider 4. Have the test repeated

1 A normal PTT is 20-25 seconds. Upper limit is 32-39. Therapeutic range is 1.5-2X the normal. so 55 is a therapeutic range

Prior to cardiac catherization on a client, what is the most important for the nurse to report? 1. client has a allergy to shellfish 2. client has diminished palpable peripheral pulses 3. client has cool lower extremities bilaterally 4. client is anxious about the pending procedures

1 All the other options are normal finding before the test

The nurse cares for client with ataxia. which action is more important? 1. supervise ambulation 2. measure intake and output accurately 3. consult the speech therapist 4. elevate the foot of the bed

1 Ataxia means poor coordination, and muscle movement

What is most important for the provider to know when treating a patient with electroconvulsive therapy (ECT)? 1. Client is being treated for glaucoma 2. client's parent had seizure with meningitits 3. client has worn dentures for 10 years 4. Client is allergic to shellfish

1 Before ECT a client is given anticholinergic medication such as atropine or glycopyrrolate to decrease secretions and lessen bradycardia. Anticholinergic medication causes pupil dilation and contraindicated for glaucoma.

ceftriaxone sodium should be monitored for what changes? 1. the surface of the tongue 2. hemoglobin and hematocrit 3. skin surfaces in skin folds 4. changes in urine characteristics

1 Cephalosporin, long term use of ceftriaxone sodium can cause overgrowth of organisms, monitoring tongue and oral cavity is recommended

A client returns to the care area after an esophagectomy. Which intervention will the nurse make a priority? 1. Turning, deep breathing, and coughing every 1-2 hrs 2. Positioning in semi fowler or high fowler 3. Monitoring heart rhythm for arrhythmias and changes in rate 4. evaluating nasogastric tube drainage for character and color

1 Choice 2 is reasonable, but choice 1 reduces mucus accumulation in the lungs which is priority (turning, deep breathing, and coughing)

Teaching for alpha-adrenergic blocker to treat benign prostatic hyperplasia (BPH). Which statement indicates the teaching is successful? 1. I will avoid tasks that require me to be alert 2. I will take medication as soon as I wake up 3. I will take my medication with food or milk 4. I will take my medication with a liquid antacid

1 Common side affects include dizziness and drowsiness, Avoid task requiring alertness Taken at bedtime to reduce the risk of orthostatic hypotension take 30 minutes after an evening meal or snack Antacid is contraindicated "SIN" Alfuzosin. Doxazosin. Prazosin Tamsulosin. Terazosin. Silodosin.

An 82 year old client tells the nurse at the clinic, "I have lived a good, successful life and married my best friend". Which of Erikson's developmental tasks does the nurse recognize that this client has probably accomplished? 1. Ego Integrity versus Despair 2. Generativity versus Stagnation 3. Intimacy versus Isolation 4. Industry versus Inferiority

1 Ego Integrity versus Despair is the major task of those 65 and over: The developmental task for this age involves the individual reviewing one's life and deriving meaning from both positive and negative events, while achieving a positive sense of self. If the individual considers accomplishments and views self as leading a successful life, a sense of integrity is developed. On the contrary, if life is viewed as unsuccessful without accomplishing life's goals, a sense of despair and hopelessness develops. 2. Incorrect: Generativity versus Stagnation is the major task for 40-64 year olds. To achieve the life goals established for oneself while also considering the welfare of future generations. The primary developmental task during this middle age period is one in which the individual contributes to society as well as helping to guide future generations. A sense of generativity (sense of productivity and accomplishment) often results from such things as raising a family and helping to better the society. In contrast, those individuals not willing to work to better society and those who are egocentric and self-centered often develop a sense of stagnation (dissatisfaction and the lack of productivity) . 3. Incorrect: Intimacy versus Isolation is the objective from 20-39 year olds to form an intense, lasting relationship or a commitment to another person. If the individual cannot form the intimate relationships (possibly due to personal needs) a sense of isolation may develop which can lead to feelings of depression. 4. Incorrect: Industry versus Inferiority is the major task for 6-12 year olds in which they attempt to achieve a sense of self confidence by learning, competing, performing successfully, and receiving recognition from significant others, peers, and acquaintances. The child must develop the ability to deal with the demands of learning new social and academic skills, or a sense of inferiority, failure, or incompetence may result.

Adverse effect of haloperidol? 1. Blood dyscrasia and extrapyramidal symptoms 2. Hearing loss and unsteady gait 3. Nystagmus and vertical gaze palsy 4. Alteration in LOC and increased confusion

1 Major effect include hematologic problems, primarily blood dyscrasia (any pathologic condition of the cellular elements of the blood) and EPS

A client suffers from migraine headaches. What assessment finding would the nurse expect to find during a migraine attack? 1. Unilateral, pulsating pain quality. 2. Bilateral, pressing/tightening pain quality. 3. Ipsilateral nasal congestion and rhinorrhea. 4. Headache occurs after recovering from a headache treated with narcotics.

1 Migraine headaches have a pulsating pain quality, unilateral location, moderate or severe pain intensity, aggravated by or causing avoidance of routine physical activity (walking, climbing stairs). During headache at least one of the following accompanies the headache: nausea and/or vomiting; photophobia and phonophobia. 2. Incorrect: This is seen in tension headaches. Headaches last 30 minutes to 7 days. Pain is mild or moderate in intensity. It is not aggravated by routine physical activity. Nausea/vomiting, photophobia and phonophobia are not common manifestations with tension headaches. These usually start gradually, often in the middle of the day. 3. Incorrect: This is associated with cluster headaches, which are severe or very severe unilateral orbital, supraorbital and/or temporal pain lasting 15-180 minutes. Symptoms include stabbing pain in one eye with associated rhinorrhea (runny nose) and possible drooping eyelid on the affected side. The headaches tend to occur in "clusters": typically one to three headaches per day (but may be as many as eight) during a cluster period. 4. Incorrect: Overuse of painkillers for headaches, can, ironically, lead to rebound headaches

The client had a thoracentesis with removal of 2500 mL of fluid from the chest cavity. What is the priority nursing assessment for this client? 1. Vital signs 2. Pain 3. O2 sat 4. Signs of infection

1 Should be watching the vital signs for shock, tachycardia, and hypotension because a lot of fluid has just been removed from the body. 2. Incorrect: Not priority; remember, pain never killed anyone. 3. Incorrect: We will watch but isn't highest priority. 4. Incorrect: Monitoring vital signs would show signs of infection.

Which client should the nurse see first after receiving report on assigned clients? 1. Having dyspnea after surgery. 2. Needing an IV started for the administration of blood. 3. Crying with pain after back surgery. 4. Vomiting dark brown, granular material

1 The client may be having a pulmonary embolism after surgery. This client with oxygenation needs takes priority over the other three clients.2. Incorrect: Needing an IV started for blood administration does not take priority over oxygenation. If blood is needed, tissue perfusion could be altered, so this would need to be addressed in a timely manner after airway issues and other potentially deteriorating situations have been addressed.3. Incorrect: Pain is expected after back surgery and is not a priority over oxygenation. When possible, the pain should be assessed and medications administered. Remember, pain never killed anyone.4. Incorrect: This client with dark brown emesis may have an upper GI bleed that has slowed or stopped. This is the second client to see but is not a priority over oxygenation. This could potentially return to active GI bleeding and the client's condition could deteriorate rapidly, so the client would need to be seen following the client with dypnea.

The outpatient nurse assesses the client's use of crutches 1 week after knee surgery. which outcome is the most important? 1. client is able to perform activities of daily living independently 2. Client has no tingling or numbness in the upper extremities 3. The client reports removing all the loose rugs from kitchen and bath 4. the client is free of DVT signs and has a normal bowel pattern

1 Using crutches the the desired OUTCOME is choice 1.

Estrogen. The nurse identfies which symptom as a common INITIAL adverse effect of this medication? 1. Nausea 2. Visual distrubances 3. Tinnitus 4. Ataxia

1 choice 2 and 3 are seen in long term use choice 4- unsteady gait is usually not seen

An 18 month old has surgery for bilateral ureteral stents. After surgery, the nurse reports a drop in urinary output. What would be the priority nursing intervention? 1. Call primary healthcare provider 2. turn from side to side 3. Irrigate 4. Reassess in 15 minutes

1 Choice 2 is for peritoneal dialysis intervention

A stable client hospitalized with a chest tube is scheduled for a chest x-ray. who can the charge nurse assign the task of transporting this client to radiology for their x-ray? Select all that apply 1. RN 2, LPN 3. Radiology tech 4. Transport tech 5. UAP

1 and 2 LPN cause they are stable, but other options are not trained on chest tubes

what medications should be hold for a client going to dialysis? Select all that apply 1. Lisinopril 2. Nitroglycerin 3. Water soluble vitamins 4. Ampicillin 5. Famotidine

1 through 4 Choice 1 and 2 will decrease BP, which will be also decreased with dialysis so may cause hypotension choice 3 and 4 will just be dialysis out and wasted choice 5 is for stomach, no need to hold

Cardiac Rehabilitation: When can sex be resumed for clients without complications? what is the best time of day?? -No valsalva maneuvers. no barring down. give docusate - what exercise is best _________, avoid __________ exercise

1 week to 10 days for sex. in the morning is best cause well rested walking, avoid isometric

A client on clopidogrel. Which lab values are important to monitor? Select all that apply 1. Hemoglobin 2. Hematocrit 3. Platelet count 4. INR 5. PTT

1, 2 this medication interviews with platelet aggregation. A decrease in hemoglobin may indicate bleeding It suppresses platelet aggregation, but DOES NOT decrease platelet count

Which prescriptions would the nurse recognize as being appropriate for the client with shingles? Select all that apply 1. Private room 2. Negative pressure airflow 3. Respirator mask 4. Face Shield 5. Positive pressure room

1, 2, 3 According to the current standards of Standard Precautions per the CDC, the client with shingles should be placed on airborne precautions which require the use of a private room with negative pressure airflow and a N-95 respirator mask.4. Incorrect: A face shield is used when there is risk of splashing or spraying of blood or body fluids. This is not required for airborne precautions. 5. Incorrect: Negative pressure is required in order to prevent the airborne infection from spreading outside of the room. Positive pressure is used only in protective environments such as when immunocompromised clients require protection from potential infectious agents outside of the room.

A client who has been taking phenytoin for several years arrives to the clinic for follow-up care. During the nurse's history and physical of the client, which findings indicate a possible side effect to the phenytoin? 1. Skin rash 2. Reports fatigue 3. Dyspnea on exertion 4. Pale conjunctiva 5. Heart rate 60/min

1, 2, 3, 4 Side effect of Phenytoin is aplastic anemia. Aplastic anemia is a blood disorder where not enough new blood cells are produced in the bone marrow. The blood cells include red blood cells, white blood cells and platelets. The most common symptom of decreased RBC's is fatigue and dyspnea upon exertion because RBC's are responsible for oxygen transport throughout the body. A common sign/symptom of aplastic anemia is also skin rashes

Symptoms to alert the nurse of a client with alcohol problem? Select all that apply 1. Tremors 2. Elevated temperature 3. depression 4. nocturnal leg cramps 5. night sweats 6. decrease concentration

1, 2, 4 3 and 6 are seen with depressed clients choice 5 is seen in TB, leukemia, or other infections

Hemolytic reaction to blood transfusions. which assessment findings fro this client are expected? Select all that apply 1. Hypotension 2. low back pain 3. wet breath sounds 4. fever 5. urticaria 6. severe shortness of breath

1, 2, 4 choice 3 is caused by circulatory overload. choice 5 is for allergic reaction, not hemolytic reaction choice 6 is also circulatory overload backpain, hypotension, tachycardia, fever are main s/s

The nurse observes a student nurse care for a client with altered mobility. For which action will the nurse immediately intervene? 1. Repositions the client every 4 horus 2. elevates HOB to 90* 3. Performs range of motion of client's extremities 4. Removes stable eschar from the client's heel 5. Massages reddened skin areas

1, 2, 4, 5 -Reposition every 2 hours. -HOB elevated more than 30* puts pressure on sacrum and promotes skin breakdown. -Stable eschar (dry, adherent, intact without erythema or fluctuance) on the heels serve as bodys "natural cover" and should be removed -massaging reddened areas may increase damage to already traumatized skin and tissue.

Suctioning a newborn using a bulb syringe. Which nursing action is appropriate? Select all that apply 1. Place the newborn supine with head turned to the side 2. Compress the bulb before inserting the bulb tip into the mouth 3. insert the bulb tip towards the back of the throat 4. Release bulb quickly to get as much secretions as possible 5. Suction the nose after the mouth is suctioned, if necessary

1, 2, 5 Back of the throat may stimulate gag reflex, stimulate vagal response, resulting in bradycardia and apnea -release bulb gently, not quickly, to minimize trauma to delicate tissues of the mouth

If blood sugar is 60. what foods would be the best choice? Select all that apply 1. Skim milk 2. Apple juice 3. milk chocolate bar 4. three oatmeal cookies 5. Handful of raisins

1, 2, 5 Choice 3 and 4 are to high in fats and will take longer to absorb. not the best choice. For someone unconscious may give honey, syrup etc. to prevent aspiration

The nurse is caring for a client on the psychiatric unit with a diagnosis of obsessive-compulsive disorder. The client has frequent hand washing rituals. Which nursing interventions would be advisable for this client? Select all that apply 1. Allow time for ritual. 2. Provide positive reinforcement for nonritualistic behavior. 3. Provide a flexible schedule for the client. 4. Remove all soap and water sources from the client's environment. 5. Create a regular schedule for taking client to bathroom.

1, 2, 5 Client needs a structural schedule enviornment

A client diagnosed with serotonin syndrome is admitted to the unit. The nurse is familiar with this adverse reaction to the serotonin reuptake inhibitors. Which symptoms can the nurse expect on assessment? 1. Fever and shivering 2. Agitation 3. Decreased body temperature 4. Constipation 5. Increased heart rate

1, 2, 5 Serotonin syndrome is a group of symptoms that can result from the use of certain serotonin reuptake inhibitors. These symptoms can range from mild to severe and include high body temperature, agitation, increased reflexes, diaphoresis, tremors, dilated pupils and diarrhea. The client is likely to experience shivering with fever. Increased heart rate and blood pressure are also commonly experienced. More severe symptoms, including muscle rigidity and seizures, can occur. If not treated, serotonin syndrome can be fatal. 3. Incorrect: Increased body temperature is expected as is increased diaphoresis. 4. Incorrect: Diarrhea, not constipation, is a symptom of serotonin syndrome.

Which tasks would be appropriate for the nurse to assign to an LPN/VN? Select all that apply 1. Changing a colostomy bag. 2. Administer antibiotic via intravenous piggyback (IVPB). 3. Teach insulin self administration to a diabetic client. 4. Administer IV pain medication to a two day post op client. 5. Check for urinary retention. 6. Remove wound sutures

1, 2, 5, 6

The nurse is caring for a client with hyperparathyroidism. The nurse will monitor the client for which complications? Select all that apply 1. Kidney stones 2. Diarrhea 3. Osteoporosis 4. Tetany 5. Fluid volume deficit

1, 3 1. & 3. Correct: Yes, because too much calcium in the blood equals too much calcium in the urine and increased risk of kidney stones. Increased parathyroid hormone (PTH) is pulling the calcium from the bones, leaving them weak. 2. Incorrect: A clinical manifestation of hyperpararthyroidism is constipation. Diarrhea is not a clinical manifestation of hyperparathyroidism. 4. Incorrect: Tetany is a clinical manifestation of hypoparathyroidism. 5. Incorrect: Fluid volume deficit (FVD) is not a clinical manifestation of hyperparathyroidism.

An adult client has just returned to the nursing care unit following a gastroscopy. Which intervention should the nurse include on the plan of care? 1. Vital sign checks every 15 min x 4 2. Supine position for 6 hours 3. NPO until return of gag reflex 4. Irrigate NG tube every 2 hours 5. Raise four siderails

1, 3 2. Incorrect: Supine position for 6 hours is contraindicated. The HOB should be elevated. In the event the client vomits, he/she is less likely to aspirate with the HOB elevated. Supine position for 6 hours is used after a heart catheterization.4. Incorrect: A client who is going for a gastroscopy procedure cannot have a nasal gastric tube. An NG tube would interfere with the procedure. 5. Incorrect: Raising all side rails is a form of restraint. Have the bed in low locked position. Raise three side rails, and have call light within reach.

What medications can not be taken? select all 1. Carvedilol with a nadolol allergy 2. Hydralazine with hydroxyzine allergy 3. Thioridazine with a promethazine allergy 4. Ciprofloxacin with azithromycin allergy 4. Ceftriaxone with cefazolin allergy

1, 3, 5

Preparing rapid insulin. The nurse draws up the insulin. what nursing actions should be taken next? 1. Verify insulin dose a third time with the MAR 2. Discard the filter needle and apply a new insulin needle 3. Administer at 90* angle, unless client is emaciated 4. Administer insulin within 15 minutes of preparing it 5. Cap the needle until ready to administer

1, 3, 5 choice 2: insulin is in vials, not ampules: therefore a filter needle is not needed Choice 3: Insulin is given within 5 MINUTES of preparation

Antisocial personality disorder. which findings does the nurse expect? Select all that apply 1. Disregards the safety of self or others 2. Flat affect with emotional detachment 3. Expresses grandiose sense of self importance 4. demonstrates excessive impulsive behaviors 5. Suspicious of other peoples motives

1, 4 choice 2 is for schizod choice 3 is for narcissistic disorder choice 5 is paranoid disorder

Erythromycin eye ointment in newborns. Which nursing action is appropriate when administering this medication? Select all that apply 1. wear disposable gloves while administering medication 2. Hold the ointment tube in a vertical position to prevent injury 3. Apply ointment from the outer canthus to the inner canthus 4. Apply ointment to the lower conjunctival sac on each eye 5. Rinse each eye with sterile saline after administering the ointment

1, 4 the ointment tube should be held horizontally to prevent eye injury

A client on prednisone. What nurse teachings about common adverse effects are appropriate? Select all that apply 1. Osteoporosis 2. Decrease white count 3. Low blood glucose 4. Low serum potassium 5. Retinal detachment 6. Fluid retention

1, 4, 6 2- it depresses the immune response, but not the white cell count 5- causes risk of cataracts and glaucoma, NOT retinal detachment

Early signs of lithium toxicity? Select all that apply 1. fine motor tremors 2. Involuntary muscle movement 3. Seizures 4. Nausea and vomiting 5. orthostatic hypotension 6. diarrhea

1, 4, 6 choice 2 and 5 = antipsychotics choice 3= severe lithium toxicity

Chemo NCLEX crital thinking exercise. Chemotherapy solution was spilled on the floor. what should the nurse do in order of steps?? 1. Wash hands thoroughly with soap and water 2. Put on respirator mask 3. Put on chemotherapy gown 4. Use absorbent pads to wipe up spills 5. Get spill kit from wall in client's room 6. Put on 2 sets of gloves 7. Put on goggles

1, 5, 2, 3, 6, 7, 4

An RN on the general pediatric unit has been reassigned to the spinal/neurology unit. What assignment by the charge nurse would be appropriate for this RN? 1. Child with spina bifida with a previous shunt revision 2. Adolescent who is 4 days post op from a spinal fusion 3. Child with a ventriculoperitoneal shunt one day post-op 4. Child with spinal muscle atrophy who is ventilator assisted 5. Child with cerebral palsy who had a tracheostomy performed this AM

1,2 The child who had a previous shunt revision and the adolescent who is 4 days post spinal fusion will be the most stable and will require the least skill level when compared with the other choices. On a general pediatric unit, the nurse would be familiar with checking for increased ICP, which would be necessary for caring for any client with a previous shunt revision. Immediately postop, the adolescent with spinal fusion would require special turning and lung assessment to prevent and observe for congestion/pneumonia, skills not acquired on a general floor. However, at 4 days postop this client should be ambulating and will not need specialized turning, so the nurse from the general pediatric unit could care for this client. 3. Incorrect: This client is more acute and requires a higher skill level. Nursing care for this child would involve frequent neurologic assessments and monitoring for infection. The child should also be monitored for signs of possible complications including bowel perforation. 4. Incorrect: This client is more acute and requires a higher skill level. The nurse on the general pediatric unit would not be experienced in caring for a child on a ventilator. 5. Incorrect: This client is more acute and requires a higher skill level. A child who is fresh post-op following a tracheostomy is at risk for airway obstruction from thick secretions, mucous plug, blood clot or dislodgement of the tube. Cardio-respiratory arrest can occur from these complications. The child is also at risk for hemorrhage. Nursing care would include frequent suctioning as needed, monitoring for early signs of airway obstruction, and trach care. The nurse from the general pediatric unit may not have the skills required to care for this child who also has cerebral palsy which could complicate the care required.

What should the nurse tell the parents of a newborn about a Guthrie test? 1. The purpose of this test is to determine the presence of phenylalanine in the blood. 2. A positive test indicates a metabolic disorder. 3. To conduct this test, a sample of blood is taken from the baby's heel. 4. An increase in protein intake can interfere with the test. 5. This test will be done when your baby is 6 weeks old.

1,2,3 A positive test indicates decreased metabolism of phenylalanine, leading to phenylketonuria. The normal level of phenylalanine in newborns is 0.5to 1 mg/dl. The Guthrie test detects levels greater than 4 mg/dl. Only fresh heel blood, not cord blood, can be used for the test. The main objective for diagnosing and treating this disorder is to prevent cognitive impairment. A LACK of protein can interfere (not increase protein) Test is done no later than 7 days after birth. If done when infant is less than 24 hours old then repeat test before infant is 2 weeks old

The nurse is caring for a client admitted to the emergency department with a history of asthma. Which assessment findings would the nurse anticipate? 1. Coughing 2. Chest tightness 3. 3 + pitting edema to ankles 4. Kussmaul respirations 5. Increased respiratory rate

1,2,5

What signs/symptoms would the nurse expect to assess in a client diagnosed with multiple sclerosis (MS)? 1. Fatigue 2. Ptosis 3. Blurry vision 4. Leg weakness 5. Limited facial expression 6. Electric shock sensation when bending neck forward

1,3,4,6 1., 3., 4., & 6. Correct: Multiple sclerosis causes fatigue which often comes on in the afternoon and causes weak muscles, slowed thinking, or sleepiness. Vision problems are common with this diagnosis and include blurry vision, double vision, and pain on eye movement. Partial or complete vision loss can occur in one eye. Because this disease affects nerves, symptoms often affect movement such as extremity weakness, numbness, tingling, and coordination. Electric-shock sensations that occur with certain neck movements, especially bending the neck forward (Lhermitte sign) develop because of the nerve damage that is occurring. 2. Incorrect: Drooping of one or both eyelids (ptosis) would be seen in myasthenia gravis rather than multiple sclerosis. 5. Incorrect: Limited facial expressions occur in myasthenia gravis rather than multiple sclerosis. The muscles (not nerves) that control facial expressions have been affected.

The nurse is caring fro a client with HIV+ that is in a semi private room. which client is best to assign in the room with a client who has HIV+? 1. The client with asthma 2. The client that is 8 hours post appendectomy 3. The client with brochitis 4. The client with partial thickness burns

1. choice 2 has an open wound area and would not want to room with HIV, choice 3 and 4 or also not a good fit cause or risk of infection and disease.

The nurse provides care for a client receiving a heparin drip via an infusion pump. The health care provider prescribes warfarin 5 mg PO. Which action does the nurse take next? 1. Administer medication as parescribed 2. Notifies the health care provider 3. Checks the recent PTT 4. Assess client for signs/ symptoms of bleeding

1. Oral anticoagulant therapy must be instituted 4-5 days before discontinuing heparin therapy PTT- monitor heparin INR= monitored warfarin

A six year old client has been receiving chemotherapy for two weeks. the laboratory results shows a platelet count of 20,000/mcl. what is the priority nursing action? 1. Encourage quiet play 2. Avoid persons with infection 3. Administer oxygen PRN 4. Provide foods high in iron

1. to decrease injury choice 3. the question says nothing about low oxygen choice 4. for RBC's, not platelets

Trouble shooting a chest tube: -What do you do if tubing becomes disconnected?? -what if the chest drainage unit (CDU) falls over and the water leaks out or shirts to the drainage compartment? -----Set CDU, check all chambers and fill water to ____ cm of water -----Have client do what in case any air went into pleural space ----- if no water in water seal chamber, then air can do what??

1. Keep another sterile connector at bedside 2. Reconnect as fast as you can 1. do whatever you can to re-establish the water seal 2. Set CDU upright, check all the chambers, and fill the water seal chamber to 2 cm or water 3. Have the client deep breathe and cough in case any air went into the pleural space 4. If there is no water in the water seal chamber, then air can do what? Collapse the lung

Lithium levels greater than ____ mEq/Liter can produce signs of toxicity

1.5 0.5-1.5

What should the fetal HR be during second trimester??

110-160 If less than 110=PANIC

When does the anterior fontanel close? When does posterior fontanel close?

12-18 months 2-3 months

Normal sodium levels? Hypernatremia = ___________ Causes: Hyperventilation, Heat stroke, DI, vomiting, Diarrhea. S/S: Dry mouth, thirst, Swollen ________, _________ changes. Treatment: Restrict __________, Dilute client with fluids (diluting makes sodium go down), If you have a sodium problem you have a ___________ problem (monitor I & O, daily weights, lab work. Feeding tubes tend to ___________ the client. Getting more sodium than water concentration in feeding tubes

135-145 Dehydration Swollen tongue, NEURO changes Restrict sodium, May give D5W (according to my best grade) sodium problem= fluid problem feeding tubes= dehydration

Rubella incubation

14-21 days Not contagious until this time

When is fetal movement felt for the first time?

16-18 weeks gestation called quickening-flutterlike feeling.

PT (warfarin)

16-31 seconds

In NCLEX world: Type 1 = ____, Type 2 = ____ DKA and HHS are both hyperosmolar states caused by _________ and _________, but there is no acidosis in HHS

1= DKA 2- HHNK (HHS) caused by hyperglycemia and dehydration

stages of labor??

1st: dilating stage 3 phases: Latent (0-3cm) Active (4-7cm) Traditional (8-10cm w/ urge to push) 2nd stage: delivery 3rd: placental delivery 4th: recovery- primary goal to prevent hemorrhage from uterine atony, 1st void within 1 hour and then q2-3 hrs, Rhogam

A 3 month old infant is scheduled for a barium swallow in the morning. prior to the procedure, it is most appropriate for the nurse to take which action? 1. offer infant only clear liquids 2. make infant NPO for 3 hours 3 feed infant regular formula 4. maintain infant NPO for 6 hours

2

A primigravida at term is admitted to the birthing room in active labor. Later, when the client is 8 cm dilated, she tells the nurse that she has the urge to push. The nurse instructs her to pant-blow at this time because pushing could: 1 rupture the uterus. 2. Cause cervical edema 3. prolapse the cord. 4. lead to a precipitous birth.

2

Teaching plain for a client regarding the use of beclomethasone? 1. Instruct to drink copious amounts of water before taking beclomethasone 2. Instruct client on proper use of inhaler 3. Instruct client to discontinue beclomethasone if not response in 2 weeks 4. Take it during acute asthma attack

2

Client with inflamed pruritic dermatitis. Which dressing is appropriate for the nurse to use for this client's condition? 1. Sterile dry gauze dressing 2. Cool tap water dressing 3. Warm sterile saline dressing 4. Warm acestic acid dressing

2 -Dry dressing does not help the itching and inflammation, and it does not need to be sterile -Dressings use to treat pruritic lesions should be cool to promote vasoconstriction and to have anti inflammatory effect -water is most commonly used for dermatitis and should be cooled, not warmed. -Bactericidal is used to treat skin infections (not inflammation)

The nurse is teaching a newly diagnosed diabetic about the action of regular insulin. The nurse verifies that teaching has been successful when the client verbalizes being at greatest risk for developing hypoglycemia at what time following the 8:00 a.m. dose of regular insulin? 1. 8:30 AM 2. 11:00 AM 3. 1:30 PM 4. 4:00 PM

2 2. Correct: 11:00 AM: Regular insulin peaks 2-3 hours after administration. Clients are at greatest risk for hypoglycemia when insulin is at its peak. 1. Incorrect: 8:30 AM: Rapid acting insulin will begin peaking in 30 minutes. 3. Incorrect: 1:30 PM: Intermediate acting insulin begins peaking at 4 hours. So at 1:30 PM this would be a time of worry. 4. Incorrect: 4:00 PM: At 4 PM you would still be worried about intermediate acting insulin. But you would also be worried about long acting insulin as well. Which starts to peak at 6 hours.

The nurse is obtaining a health assessment from the preoperative client scheduled for hip replacement surgery. Which statement by the client would be most important for the nurse to report to the primary healthcare provider? 1. "When I was 8 years old I had chickenpox." 2. "I had rheumatic fever when I was 10 years old." 3. "There is a strong history of gastric cancer in my family." 4. "I have pain in my hip with any movement."

2 2. Correct: After having rheumatic fever, a client would need to be pre-medicated with antibiotics prior to any surgical or dental procedure to prevent a recurrence. 1. Incorrect: Chicken pox would have no implications on this surgery. 3. Incorrect: Cancer history in the family would have no implications for this surgery. 4. Incorrect: Pain in the hip is likely the reason for the surgery.

The nurse is caring for a client who is receiving a prostaglandin agonist for the treatment of glaucoma. Which comment by the client indicates a lack of understanding of the treatment regimen? 1. I must only use the drops in the eye with the increased pressure. 2. My eyes may be different colors, so I will use the drops in both eyes. 3. I must be careful not to overmedicate even if it is just an eye drop. 4. The eyelashes in the eye with the higher pressure may get longer.

2 2. Correct: The color of the iris may darken in the eye being treated; however, it is important that the client understand that drops should not be placed in the unaffected eye. Prostaglandins cause increased permeability in the sclera to aqueous fluid. So, as the prostaglandin agonist increases this activity, the outflow of aqueous fluid increases and the ocular pressure decreases. Administering the drops in the unaffected eye may result in a subnormal intraocular pressure.1. Incorrect: This comment shows adequate understanding. The client should only treat the eye with the increased pressure.3. Incorrect: This comment demonstrates that the client does understand the treatment regimen. Overmedicating the affected eye could reduce the intraocular pressure too much. 4. Incorrect: This comment shows understanding. The lashes in the eye being treated will lengthen as opposed to the untreated eye. The changes of the eyelashes (increased length, thickness, pigmentation and number of lashes) are typical with these eye drops and are viewed as a benefit by many clients.

A client has been prescribed chlorpromazine for the treatment of schizophrenia. The nurse makes afternoon rounds and finds the client's temperature to be 104.7º F/40.4º C. The client has extreme muscle rigidity, and the vital signs have been fluctuating for the last four hours. What should the nurse do first? 1. Provide a tepid sponge bath. 2. Notify the primary healthcare provider immediately. 3. Administer an antipyretic immediately. 4. Administer the chlorpromazine as prescribed.

2 2. Correct: These symptoms are consistent with neuroleptic malignant syndrome (NMS), which is an adverse reaction to antipsychotic drugs. The symptoms of NMS are fever, altered mental state, muscle rigidity, and autonomic dysfunction. This is a medical emergency, and immediate action should be taken. 1. Incorrect: The symptoms indicate a medical emergency and the need for an immediate response. The nurse should notify the primary healthcare provider first. 3. Incorrect: The high temperature should be assessed, but the extreme muscle rigidity and fluctuating vitals are a medical emergency. The client needs further immediate attention. 4. Incorrect: The nurse should not administer another dose of the antipsychotic medication due to the client's presenting symptoms. Usually, the primary healthcare provider would discontinue the medication immediately.

A terminally ill client that received intravenous morphine 20 minutes ago develops dyspnea and bilateral rales. Which medication will the nurse plan to administer next? 1. Ondansetron IV 2. Atropine sublingual 3. Dexamethasone IV 4. Haloperidol Sublingual

2 Atropine is anticholinergic that can reduce secretions. Dyspnea should improve and rales should dissipate

The primary healthcare provider prescribed an analysis of chest drainage for WBCs. where would you obtain the specimen 1. Drainage collection chamber 2. chest tube

2 Can use a needle to puncture chest tube and obtain analysis

The nurse assesses clients waiting to be seen by the health care provider. Which client does the nurse identify to be seen first? 1. Client with myasthenia gravis reporting double vision and drooping of the right eye lid 2. Client with a flat 9 mm induration area at the site of a TB skin test placed 48 hours ago 3. Client with a Mean arterial pressure of 80 mm Hg. 4. client with lung disease reporting dyspnea after walking up stairs.

2 Choice 1 is expected findings. MG causes weakness of skeletal muscles. most often those involved in the eyes, eyelids, chewing, swallowing, and breathing. with TB the patient needs to be isolated to reduce the risk of spread

The nurse provides care to a client who experienced a stroke 1 day ago. Which action does the nurse safety delegate to the nurse assistive personnel? 1. Monitor neurological status using the glasgow coma scale 2. Check and document oxygen saturation every 1-2 hrs 3. Complete bath, change linen, and perform ROM by 1000 hours 4. Notify the provider about the need to assistive walking device.

2 Choice 3. clustering cares in a short period of time may increase the ICP. thereby making the client stable. so that makes in inappropriate.

Toddler with a 3 day history of diarrhea and nausea. The child is treated for mild dehydration in the emergency department. Which dietary instructions does the nurse provide for the parents to care for the child at home? 1. Clear liquids and crackers until the nausea resolves 2. Regular food and fluids, as the child's appetite allows 3. Broths and juices today, then soft diet as tolerated 4. Bananas, rice, applesauce, and toast until tomorrow

2 Early reintroduction of nutrients is desired and has gained acceptance. No restriction on foods is needed

The nurse is giving discharge instructions to an Asian client following a colonoscopy. During the instructions, the client stares directly at the floor, despite being able to speak English. Based on the client's body language, how would the nurse classify this behavior? 1. Embarrassment. 2. Attentiveness. 3. Disinterest. 4. Confusion.

2 Nurses must be aware of clients' specific cultural or religious beliefs in order to provide appropriate care and discharge planning. Asian societies have a deep respect for others and making eye contact with the nurse would be considered rude and offensive. The nurse is considered superior to the client, so direct eye contact with a superior shows a lack of respect. This client is displaying attentiveness while also showing respect for the nurse. 1. Incorrect: There is nothing in the question to suggest the client is embarrassed. In Asian cultures, making eye contact is considered disrespectful to the superior; therefore, this client's demeanor is a respectful display of cultural influences.

A newly admitted client with schizophrenia has an unkempt appearance and needs to attend to personal hygiene. Which statement by the nurse is most therapeutic? 1. A shower will make you feel better. 2. It is time to take a shower. 3. Have you thought about taking a shower? 4. I need you to take a shower

2 Schizophrenia is a thought disorder. Many clients with schizophrenia are concrete thinkers and have difficulty making decisions. The nurse needs to be direct, clear and concise in communicating with the client. This is a direct, clear and concise statement that guides the client to perform the needed activity.

A new nurse has a prescription to insert a feeding tube. The new nurse has never performed the procedure, but learned how to do it while in nursing school. What would be the best action by this nurse? 1. Ask to observe another nurse perform the procedure. 2. Look up how to perform the procedure in the policy and procedure manual. 3. Tell the charge nurse that someone else will have to place the feeding tube down the client. 4. Insert the feeding tube as learned in nursing school.

2 The best action for the nurse to take is to look up how the procedure is done in the agency by looking it up in the policy and procedure manual. The nurse could then discuss the procedure with an experienced nurse and ask the nurse to observe the new nurse while inserting the feeding tube. 1. Incorrect. This is passive and would not benefit the new nurse to strengthen the skills. The best action would be to look up how to do the procedure, discuss with another nurse, and ask that nurse to observe the insertion of the feeding tube. 3. Incorrect. This is not the best option. The new nurse needs to insert the feeding tube in order to become more proficient with this skill. This option will not help the new nurse gain confidence in nursing skills. 4. Incorrect. Although the new nurse should have the basic knowledge of feeding tube insertion, the nurse should follow agency policy and procedure. It is then best to discuss the procedure with another nurse and ask the nurse to observe the feeding tube insertion since this nurse has never performed the skill.

The primary healthcare provider has prescribed phenytoin 100 mg intravenous push (IVP) stat for an adult client. What is the least amount of time that the nurse can safely administer this medication? 1. 1 minute 2. 2 minutes 3. 5 minutes 4. 10 minutes

2 The rate of IV administration should not exceed 50 mg/min. for adults and 1-3 mg/kg/min (or 50 mg/min, whichever is slower) in pediatric clients because of the risk of severe hypotension and cardiac arrhythmias. So 100 mg can safely be delivered over a period of at least 2 minutes.

A client has been taught guided imagery as a method to relieve pain. How should the nurse first assess for pain relief after completion of guided imagery by the client? 1. Assess vital signs 2. Use of pain intensity scale 3. Ask client to describe the pain 4. Observe ability to perform activities of daily living

2 The use of pain intensity scales is an easy and reliable method of determining the client's pain intensity.Although some other assessments regarding pain may be performed, the initial assessment should be performed using the pain intensity scale. 1. Incorrect: Although respiratory and heart rate may decrease with guided imagery and pain reduction, the most objective measure is to ask the client to rate the pain. 3. Incorrect: First, ask the client if pain is present. If present, the client should be asked to rate the pain. Once pain has been rated, the client should be asked to describe the pain. 4. Incorrect: The client may be able to perform activities of daily living and still have pain. Therefore, this would not be an accurate means of assessing pain relief.

Client diagnosed with pneumonia secondary to chronic pulmonary disease. which nursing goal is most appropriate? 1. Maintain and improve the quality of oxygenation 2. Improve the status of ventilation 3. Increase oxygenation of peripheral circulation 4. Correct the bicarbonate deficit

2 To improve the quality of ventilation refers to levels of carbon dioxide and oxygen Choice 1 is incorrect cause the primary problem is not levels of oxygen, but levels of CO2

A women 8 weeks gestation with a positive VDRL. What teaching is most important? 1. Advise not to take any over the counter medications 2. instruct her on importance to take all of the medication 3. inform to refrain from sexual activity 4. maintain confidentiality of sexual partners

2 VDRL= syphillis think maslows. choice 2 is the most physical, vital important to complete

A client is curious about visible appearance changes related to menopause. What menopausal changes, in general, would the nurse explain to the client? 1. Bone loss and fractures. 2. Loss of muscle mass. 3. Improved skin turgor and elasticity. 4. A reduction in waist size.

2 Visible changes associated with menopause include loss of muscle mass, increased fat tissue leading to thicker waist, dryness of the skin and vagina, hot flashes, sleep abnormalities, and mood changes. 1. Incorrect: Bone loss is dependent on bone mass, weight-bearing exercise, and nutrition. Some bone loss may occur, but may not lead to fractures. 3. Incorrect: A decrease in turgor and elasticity may occur as we grow older. 4. Incorrect: There is increased fat tissue with an increase in waist size.

The nurse is unable to locate an older client's left popliteal pulse. Which action will the nurse take next? 1. Check for the femoral pulse 2. Check for the pedal pulse 3. Ask another nurse to check for the popliteal pulse 4. Measure the blood pressure on the left thigh.

2 Want to measure more distal pulse

Which electrolyte imbalance would be the nurse's priority concern in the burn client? 1. Hypernatremia 2. Hyperkalemia 3. Hypoalbuminemia 4. Hypermagnesemia

2 When the cells lyse they release potassium, and then the serum potassium goes up. And if the kidneys stop, we are in real trouble. 1. Incorrect: Well this one does occur when the client becomes very dehydrated, but it's not as dangerous as the potassium one. 3. Incorrect: Low albumin can cause problems keeping fluid in the vascular space, but albumin is not an electrolyte. 4. Incorrect: No, the magnesium doesn't go up unless the kidneys shut down.

Erikson's stage related to a 50 year old client? 1. integrity versus despair and disgust 2. Generativity versus stagnation 3. Intimacy versus isolation 4. identity versus role diffusion

2 choice 1 is for 65 and older choice 3 is young adult choice 4 adolescent

A client diagnosed with pneumothorax 3 days ago. The client has a chest tube connected to a three chamber water seal drainage system with 20 cm suction. the nurse determines the lung has re expanded if which observation is made? 1. There is no drainage in collection chamber for 3 hours 2. Fluid in the water seal chamber does not fluctuate with respiration 3. There is continuous bubbling in the water seal chamber 4. There is gentle bubbling in the suction control chamber

2 choice 3 indicates air leak. choice 4 is a normal finding

A client with Crohn's disease develops a fever and symptoms of an infection. The nurse recognizes this complication may occur as a result of which finding? 1. Perianal irritation from frequent diarrhea 2. Fistula formation with an abscess 3. Stricture formation 4. Impaired immunologic response to infectious microorganisms

2 2. Correct: Clients who suffer from Crohn's disease are at risk for developing fistulas, and an abscess can result from the fistula. 1. Incorrect: Perianal irritation from frequent diarrhea can occur, but irritation does not result in an infection.3. Incorrect: Stricture formation is a complication, however, these s/s indicate an abscess. 4. Incorrect: Impaired immunologic response is not associated with Crohn's disease.

A client with chronic alcoholism has been admitted to the intensive care unit after overdosing on alcohol. Which medication should the nurse prepare to administer? 1. Disulfiram 250 mg po daily 2. Thiamine 100 mg IV twice a day 3. Naloxone 0.4 mg IV prn 4. Clonidine TTS patch 2.5 mg per week

2 2. Correct: Prescribing of thiamine action is to alleviate dehydration, prevent delirium and precaution treatment for vitamin B complex deficiency. Thiamine 50-100 mg IV or IM is indicated twice a day for clients with chronic alcoholism. It is usually given for several days, followed by 10-20 mg once a day until a therapeutic response is obtained. 1. Incorrect: Disulfuram is an aid in the management of selected chronic alcohol clients who want to remain in a state of enforced sobriety so that supportive and psychotherapeutic treatment may be applied. It is not a cure for alcoholism. Without proper motivation and supportive therapy, it is unlikely that it will have any substantive effect on the drinking pattern of the chronic alcoholic. 3. Incorrect: Naloxone prevents or blocks the action of narcotics (opioid medication). Naloxone is indicated for opioid overdose. Naloxone is sometimes prescribed to verify whether the client has overdosed with an opioid. 4. Incorrect: Clonidine is used to suppress opiate withdrawal symptoms. Serves effectively as a bridge to enable the client to stay opiate-free long enough to facilitate termination of methadone maintenance.

A client is in the surgical suite to have a left total knee replacement. Prior to the surgeon initiating the first incision, what should the circulating nurse remind the surgical team to perform? 1. Surgical scrub 2. Time-out 3. Sponge and instrument count 4. Inspection of the surgical site

2 2. Correct: Time-out, done immediately before the procedure, is a final verbal verification of the correct client, procedure, site, and implant. Time-out is active communication among all members of the surgical/procedural team, initiated by a member of the team before surgery.1. Incorrect: Surgical scrub should be done before entering the surgical suite.3. Incorrect: The scrub nurse does instrument and sponge counts numerous times before, during, and after the procedure.4. Incorrect: Simple inspection of the surgical site is not enough. The team must verbally communicate what is to be done, on what limb. Additionally, the limb should be marked as "This knee" or "yes".

Which statement, made by a client scheduled for a total laryngectomy, indicates to the nurse a need for further preoperative teaching? 1. After the surgery, I will breathe only through a hole in my neck. 2. My wife will have to get a hearing aid because I will not be able to talk above a whisper. 3. I must have smoke detectors installed at home since I may not be able to smell after surgery. 4. After surgery, I will have a tube going through my nose to my stomach for feeding.

2 With a total laryngectomy, the vocal cords are removed. The entrance to the trachea is closed, so no air moves upward into the throat or mouth areas. The client will not be able to speak or whisper. The client's wife does not need a hearing aid, so further teaching is necessary. 1. Incorrect: The client will breathe through a hole in his neck (tracheostomy) for the rest of their life. This is a true statement by the client. We are looking for the false statement. 3. Incorrect: Since the entrance to the trachea is closed, the client can no longer move air through the nasopharynx. Therefore, the capacity to smell may be diminished or lost. The ability to smell remains intact because the sensory nerves in the nose are not impacted by the surgery. However, in order to smell normally, air must pass over the sensory cells which is not occurring in this case. This is a true statement by the client. We are looking for the false statement. 4. Incorrect: During surgery, a feeding tube is placed in the stomach or jejunum to assist in nutritional requirements until the surgical area in the throat is healed. With a total laryngectomy, the client will eventually be able to eat because the trachea and esophagus are completely separate from each other. This is a true statement by the client. We are looking for the false statement.

the nurse is monitoring the lab values of a client on long term steroid therapy. Which values would the nurse expect to be altered in the urine? Select all that apply 1. Protein 2. Glucose 3. Ketones 4. RBCs 5. Uric acid

2 and 3 Protein can not leak into urine unless kidney damage occurs Excess sugar in blood will leak into urine, Ketones from break down of fat due to high sugar levels.

For Addison's disease: may give corticosteroids- "sone" Give _______ a day in split doses. give ____ in the morning and ______ in the evening. Fludrocortisone acetate is man made ___________. Daily ______ and ______ must be monitored

2 doses daily. 2/3 in the morning and 1/3 in the evening. man made aldosterone daily weight and BP must be monitored

A serum sodium level of 119 mEq/L. which action will the nurse take? Select all that apply 1. Obtain an order for nothing my mouth 2. monitor for neurological changes 3. teach about a fluid restriction 4. Anticipate hypertonic saline 5. assess for signs of fluid overload

2 through 5

you are caring for an 11 year old boy in the hospital. What would be an appropriate activity for him? Select all that apply 1. Frequent visits from friends 2. playing video games in the room 3. coin collection 4. watching T.V 5. Reading his favorite book.

2 through 5.

The nurse is planning discharge teaching for a family of opioid abusers. Which information should be included? Select all that apply 1. Naloxone by mouth is the preferred administration method 2. CPR techniques should be learned by family members 3. Location of the naloxone kits should be known by family members 4. Naloxone dosages should be repeated every 10 minutes for up to 3 doses 5. Hospitalization is not required if client recovers and respiration's are greater than 16.

2, 3 Dosages should be repeated every 3 minutes (not 10)

Where should a nurse place the stethoscope when auscultating heart sounds? 1. First intercostal space left of the sternum to hear sounds from the pulmonic valve area. 2. Fourth intercostal space to the left of the sternum to hear sounds from the tricuspid area. 3. Second intercostal space to the right of the sternum to hear sounds from the aortic valve area. 4. Fifth intercostal space left side of sternum to hear sounds from the mitral area. 5. Apex of the heart to hear the loudest 2nd heart sound (S2).

2, 3 Second intercostal space left of the sternum to hear sounds from the pulmonic valve The fifth intercostal space in the midclavicular line is where you will hear sounds in the mitral area. Apex of the heart is where you will hear the loudest 1st heart sound (S1). Listen at the base to hear S2 the loudest.

A nurse is caring for a client who delivered a baby vaginally two hours ago. What signs and symptoms of postpartum hemorrhage should the nurse report to the primary healthcare provider? 1. Two blood clots the size of a dime. 2. Perineal pad saturation in 10 minutes. 3. Constant trickling of bright red blood from vagina. 4. Oliguria 5. Firm fundus

2, 3, 4 Lochia should not exceed an amount that is needed to partially saturate four to eight peripads daily, which is considered a moderate amount. Perineal pad saturation in 15 minutes or less is considered excessive and is reason for immediate concern. Saturation of a peripad in one hour is considered heavy. Also, trickling of bright red blood from the vagina can indicate hemorrhage and is often a result of cervical or vaginal lacerations. Bright red blood indicates active bleeding. Oliguria is a sign of fluid volume deficit. As blood volume goes down, renal perfusion decreases and urinary output (UOP) decreases. The kidneys are also attempting to hold on to what little fluid volume is left. 1. Incorrect: A few small clots would be considered normal and occur due to pooling of the blood in the vagina. Passage of numerous or large blood clots (larger than a quarter) would indicate a problem.

What drugs would be contraindicated with St. John's wort? 1. Hydrochlorothiazide 2. Digoxin 3. Nifedipine 4. Simvastatin 5. Escitalopram 6. Metformin

2, 3, 4, 5 Basically heart medications and SSRI

The nurse provides care to a client diagnosed with Bulimia nervosa. Which sign will the nurse expect to observe while providing care to this client? 1. Hypernatremia 2. tooth erosion 3. Parotid gland swelling 4. Gastric narrowing 5. Hypokalemia

2, 3, 5 -Hyponatremia and hypochloremia occur due to vomiting and use of laxatives -Gastric dilation occurs secondary to binge eating, (gastric widening)

Which signs and symptoms would the nurse expect to see in a client who has taken prednisone for two months? 1. Weight loss 2. Decreased wound healing 3. Hypertension 4. Decreased facial hair 5. Moon face

2, 3, 5 Decreased wound healing is a side effect of prolonged steroid use due to immunosuppressive effects

Signs of neonatal hypoglycemia? 1. Hyperthermia 2. Poor feeding 3. Jitteriness 4. Hypertonia 5. Seizures

2, 3, 5, • Jitteriness, tremors • Poor muscle tone • Diaphoresis (sweating) • Poor suck • Tachypnea • Tachycardia • Dyspnea • Grunting • Cyanosis • Apnea • Low temperature • High-pitched cry • Irritability • Lethargy • Seizures, coma • No signs (some infants may be asymptomatic)

What interventions should the nurse initiate to keep the airway free of secretions in a client with pneumonia? 1. Evaluate results of ABG's and report abnormal findings. 2. Increase oral intake to at least 2000 mL/day. 3. Administer a cough suppressant medication. 4. Educate client on incentive spirometry. 5. Perform percussion to affected area.

2, 4, 5 Liquefy secretions by increasing oral intake to at least eight, 8 ounce glasses of liquid/day unless fluid restrictions are required. Incentive spirometry helps keep alveoli open and prevents further pneumonia and atelectasis. Prescribed percussion can assist with loosening secretions for expectoration. 1. Incorrect: This does not get rid of secretions. This monitors respiratory effectiveness. 3. Incorrect: The nurse knows that client needs to expectorate the sputum to remove bacteria or prevent bacterial growth. If the cough is suppressed, the sputum will remain in the lungs, providing a medium for bacterial growth.

What symptoms would the nurse anticipate in a client being admitted to the hospital with a calcium level of 3.2 mg/dL (0.80 mmol/L)? 1. Slowed deep tendon reflexes. 2. Muscle rigidity and cramping. 3. Hypoactive bowel sounds. 4. Positive Chvostek's sign. 5. Seizures 6. Laryngospasms

2, 4,5,6 With hypocalcemia, the muscle tone is rigid and tight. Therefore, the client may report muscle cramping. A hallmark sign of hypocalcemia is a positive Chvostek's sign, which is a twitching of facial muscles following tapping in the area of the cheekbone, indicative of hyperirritability. The client may be at risk of having seizures due to the neuromuscular irritability. Prolonged contraction of the respiratory and laryngeal muscles causes laryngospasm and stridor and may result in cyanosis.

Neutrophil normal range

2,000 - 8000 cells / µL

Complications of autonomic dysreflexia? Select all that apply 1. urinary bladder spasm pain 2. severe pounding headache 3. Profuse sweating 4. Dysrhythmias 5. Severe hypotension 6. Nasal congestion

2,3, 6 Bladder Pain is not usually perceived Bradycardia is the most common change in pulse. Dysrhythmias are not a symptom thought Causes HYPERTENSION

How many cups of milk should a 15 month old toddler consume daily?

2-3 cups

Normal CVP values

2-6 mmHg (values of blood returning to the right atria) 5-10 cmH2O (depends on measuring device

A client just placed in physical restraints. which nursing intervention is most important to client's care? 1. Prepare PRN dose of psychotropic medication 2. Check that the restraints have been applied correctly 3. Review hospital policy regarding duration of restrains 4. Monitor client's need for hydration and nutrition while restrained

2. Assessment, Monitor positioning, tightness and peripheral circulation. Choice 4 is also assessment but can be done after the client is safely restrained correctly.

A client is having difficulty breathing because the tracheotomy tube has been dislodged. Which is the Initial action the nurse should take? 1. Perform mouth to stoma breathing 2. Extend the client's neck 3. Place the client in high fowler's position 4. Administer oxygen

2. Provides patent airway. call for help. place supine, then check breath sounds. use hemostat to open airway. Oxygen and high fowler's position does not work unless there is and established airway by extending the client's neck

INR on warfarin

2.0-3.0

Pre-eclampsia develops after ______ weeks gestation and the client will have: -increased _________ - ________ in their urine -edema -________ or more pounds of weight gain in a week, watch BP closely and worry about pre eclampsia -Pre-eclampsia is defined as BP of _______ or greater that is documented 6 hours apart -client is at risk of ____________. what drug of choice is given for severe pre-eclamsia?

20 -BP -proteinuria -2 or more 160/110 -magnesium sulfate

How many calories are required for active adolescent female? How much iron is required?

2000-2400/day 15 grams/day

Baby is considered viable at?

24 weeks gestation

When is gestational diabetes screened for? Complications to baby include __________ in birth weight, _________ at birth (due to baby use to mom's sugar level so baby sugar drops)

24-28 weeks (If risk factors screen at first prenatal visits) increased birth weight, and hypoglycemia at birth

Top 5 High Alert Medications: insulin Opiates/ narcotics Injectable potassium chloride or phosphate concentration IV Anticoagulants (Heparin) Sodium Chloride Solution above 0.9% DOUBLE CHECK WITH A _______.

2nd licence nurse

A newborn in a neonatal unit is to receive penicillin G benzathine 50,000 units/kg intramuscularly (IM). The newborn weighs 6 lbs (2.7 kg). The dispensed dose is 25,000 units per 1 mL. What should the nurse do? 1. Administer the drug intravenously (IV) since a large volume is required. 2. Choose three injection sites and give the medication as prescribed. 3. Consult with the pharmacy for a different medication concentration. 4. Read the available drug information to determine how to administer the medication.

3

Postoperative cataract client is told not to make sudden movement or bend over. What is the rationale for this recommendation? 1. impairment of cerebral blood flow and headache 2. increase intracranial pressure 3. pressure on the ocular suture line 4. displacement of the lens implant

3

The nurse is working in the ED and is assigned to care for the clients in examination rooms 1, 2, and 3. The nurse received the following report from the off going nurse. Which client would you go see first? 1. Client in room 1 is an elderly person who has fallen and is currently in CT to rule out a subdural hematoma 2. Client in room 2 is diagnosed with kidney stones, positive for hematuria and has 8/10 pain 3. Client in room 3 has blood pressure of 90/40

3

The change nurse is making assignments for the shift. The staff includes and a RN pulled from the neonatal ICU (NICU) who has not worked on an adult floor in six years. what is the appropriate action by the charge nurse? 1. send the RN back to NICU and give the nurses who are already working on the floor an extra client 2. Call the nursing supervisor and demand an RN with medical surgical experience 3. Attend shift report 4. Assign the NICU nurse to do UAP duties Part II Which client is best to be assigned to the NICU nurse pulled to the adult medication surgical floor? 1. 4 hour post cholecystectomy client experiencing pain every 3-4 hours 2. Elderly client with unexplained syncope 3. Teenage client 8 hours post hypophysectomy 4. New admit diagnosed with adrenal insufficiency.

3 Part II 1. Cholecystectomy is galballder removal and the most stable choice 3 may affect ADH and pituitary so client is unstable and may develop D.I

The nurse is preparing to provide oral care to an unconscious client. What is the most important step for the nurse to provide? 1. Performing hand hygiene. 2. Explaining the procedure to the family. 3. Positioning the client in side-lying position. 4. Raising the head of bed 30 degrees.

3 3. Correct: Positioning the client in a side-lying position allows secretions to drain from the mouth and prevents aspiration. The most important aspect of care is the protection of the airway of this unconscious client. This is accomplished through proper positioning of the client in a side-lying position.1. Incorrect: Hand hygiene is a key component of standard precautions. Hand hygiene is not a priority over preventing aspiration.2. Incorrect: Informing the family about the procedure should be done, but is not the most important step in oral care. The nurse should explain the oral care procedure to the family. Maintaining the clients airway is the priority action. The client should be placed in the side lying position.4. Incorrect: No, side lying is the appropriate position to allow drainage of secretions from mouth and prevent aspiration. Positioning the client with the HOB elevated to 30 degrees will not promote drainage of secretions from the client's mouth. The priority action is to maintain the client's airway.

The nurse is caring for a client who is unresponsive during a postictal state. Which position is correct for this client? 1. Orthopneic 2. Dorsal recumbent 3. Sims' 4. Reverse trendelenburg

3 3. Correct: Sim's is a semi prone position where the client is halfway between lateral and prone positions. Often used for enemas or other examinations of the perianal area. Sim's is used for unconscious client's because it facilitates drainage from the mouth and prevents aspiration. 1. Incorrect: Orthopneic position places the client in a sitting position with arms resting on an overbed table. It allows maximum expansion of the chest. This would not be a safe position for an unresponsive client. 2. Incorrect: Dorsal recumbent is a back lying position where the shoulders are slightly elevated on pillows. it is used after surgeries and anesthetics. 4. Incorrect: Reverse trendelenburg is where the body the body is completely straight but the head is elevated and the feet are down. This position helps with gastroesophageal reflux disease, snoring, and with some surgeries.

What should a nurse teach a client who has been diagnosed with hepatitis A? 1. Hepatitis A is spread through blood and body fluid. 2. Chronic liver disease is a common complication of hepatitis A. 3. Symptoms of hepatitis A include malaise, dark colored urine, and jaundice. 4. Treatment includes alpha-interferon and ribavirin.

3 3. Correct: Symptoms of hepatitis A include fever, malaise, loss of appetite, diarrhea, nausea, abdominal discomfort, dark-colored urine, and jaundice. 1. Incorrect: Hepatitis A is spread when an uninfected person ingests food or water that is contaminated with the feces of an infected person. This disease is closely associated with unsafe water, inadequate sanitation, and poor personal hygiene. 2. Incorrect: Hepatitis A infection does not cause chronic liver disease and is rarely fatal, but it can cause debilitating symptoms and fulminant hepatitis, which is associated with high mortality. 4. Incorrect: Used for hepatitis C.

Complete Blood Count RBCs 5 million/mm3 (5 X 106 /mm3)​ (5 X 1012 /L​) WBCs 5,000 (5 X 103/mm3) ​(5 X 109 /L​) Urinalysis RBCs 2 to 3/hpf WBCs greater than 5/hpf. Blood and urine samples are sent to the laboratory for a client who has had a spinal cord injury. After reviewing these results, the nurse would expect which finding? 1. Gross hematuria 2. Septicemia 3. Urinary tract infection 4. Anemia

3 3. Correct: The urinalysis results of red blood cells (RBC) of 2/hpf or greater and urine white blood cells (WBC) of greater than 4/hpf indicate a urinary tract infection (UTI). 1. Incorrect: The urinalysis results of 2 to 3/hpf RBCs is not indicative of gross hematuria 2. Incorrect: The blood WBCs are normal. In septicemia, the blood WBCs are elevated. 4. Incorrect: Blood RBCs of 5 million/mm3 (5 x 106 ​/ mm3) (5 x 1012/ mm3) is a normal finding.

What is a CT scan?

A computed tomography is cross-sectional x-ray images of the brain and spinal cord. CT scans can detect bone and joint problems, like complex bone fractures and tumors. If you have a condition like cancer, heart disease, emphysema, or liver masses, CT scans can spot it or help doctors see any changes. They show internal injuries and bleeding, such as those caused by a car accident.

Standard medication therapy for HF is ?

ACE inhibitors and ARBs

what hormone makes you retain just water?

ADH- Retain just water

Bed rest can induce diureses by the release of _____ and decrease production of ____

ANP ADH

Normal AST & ALT levels

AST (10-34) ALT (10-40)

Anticholinergic drugs block the action of a neurotransmitter called ___________. This inhibits nerve impulses responsible for involuntary muscle movements and various bodily functions. These drugs can treat a variety of conditions, from overactive bladder to chronic obstructive pulmonary disorder.

Acetylcholine Anticholinergics are the bronchodilators of choice in the management of chronic obstructive pulmonary disease (COPD). They work by blocking muscarinic receptors in airway smooth muscle

Side effects of corticosteroids use?

Acute mania, immunosuppression, thin skin, osteoporosis, easy bruising, myopathies, hypernatremia, hypokalemia, hyperglycemia (steroids make BS go up by inhibiting insulin)

A small hematoma that develops rapidly may be fatal, while a massive hematoma that develops slowly may allow the client to _______

Adapt

Adrenal cortex problems: Not enough steroids- ____________ disease (adrenocorticol insufficiency) ---Shock, ______kalemia, ________glycemia

Addison's disease- not enough glucocrticoids, mineralcortciods, or sex hormones Hyperkalemia, hypoglycemia

Where is aldosterone found? Its normal action: when blood volume gets low (vomiting, hemorrhage, etc) Aldosterone secretion increases to cause ____________ and __________ retention in the vascular space there by cause volume and fluid volume to go up

Adrenal glands (top of kidney) Water and sodium

Left sided heart failure is when blood is not moving forward so back flows into the ____________

Affects the lungs 1. Pnea, SOB, dyspnea 2. Crackles 3. Oliguria 4. Frothy Sputum 5. Displaced Apical Pulse (Hypertrophy) 6. tachycardia 7. S3 sound 8. restlessness. 9. cough

client getting TPN. Which morning laboratory results will the nurse report to the provider? Select all that apply 1. Potassium 4.9 2. sodium 149 3. Fasting glucose 125 4. Serum calcium 15 5. Serum albumin 3.0

All but 1 Fasting glucose normal range is 60-115. this is elevated. Normal albumin is 3.5-5.0

A nurse is attempting to develop trust with a psychiatric client exhibiting concrete thinking. Which nursing intervention would promote trust in this individual? select all that apply 1. Attend an activity with the client who is reluctant to go alone. 2. Allow the client to break an insignificant rule. 3. Consider client preferences when possible in decisions concerning care. 4. Provide a blanket when the client is cold. 5. Provide food when the client is hungry.

All but 2 1., 3., 4. & 5. Correct: Trust is demonstrated through nursing interventions that convey a sense of warmth and care to the client. These interventions are initiated simply, concretely, and directed toward activities that address the client's basic needs for physiological and psychological safety and security. Concrete thinking focuses thought processes on specifics, rather than generalities, and immediate issues, rather than eventual outcomes. Examples of nursing interventions that would promote trust in an individual who is thinking concretely include such things as: providing a blanket when the client is cold, providing food when the client is hungry, keeping promises, being honest, providing a written, structured schedule of activities, attending activities with the client if he is reluctant to go alone, being consistent in adhering to unit guidelines, and taking the client's preferences, requests, and opinions into consideration when possible in decisions concerning care. 2. Incorrect: The client should be informed of all rules, simply and clearly, with reasons for certain policies and rules. Be consistent and provide written, structured, scheduled activities. Allowing a client to break a rule would not encourage them to think about the outcomes of their actions.

To reduce the risk of developing a complication following balloon angioplasty, the nurse should implement which measure? 1. Monitor cardiac rhythm 2. Assess the puncture site every 8 hours 3. Measure urinary output hourly 4. Prevent flexion of the affected leg 5. Avoid lifting buttocks off the bed

All but 2. The client should avoid lifting the buttocks off the bed because this increases pressure at the insertion site which increases the risk of hematoma formation/bleeding. 2. Incorrect: Assessments are needed more frequently than every 8 hours. Although policies may differ, assessment of the insertion site is usually every 15 minutes for 1 hour, every 30 minutes for 1 hour, and then hourly for 4 hours. More frequent monitoring may be required. During the assessment, the nurse should observe the catheter access site for bleeding or hematoma formation and should assess the peripheral pulses in the affected extremity.

An elderly client arrives in the emergency department (ED) after a fall. What assessment findings would lead the nurse to suspect that the client has a fractured right hip? Select all that apply 1. Severe pain in the right hip and groin. 2. Inability to bear weight on the right leg. 3. Right leg slightly longer in length than the left leg. 4. External rotation of right lower leg. 5. Bruising and swelling around the right hip.

All but 3 1., 2., 4., & 5. Correct: Pain in the affected hip, often severe, is one of the main signs of a hip fracture. This pain may radiate to the groin area. The pain and bone injury generally prevent the client from being able to bear weight on the affected leg. The client will often assume a position in which the leg on the injured side is held in a still and externally rotated position (the foot and knee turns outward). Discoloration and swelling can be an indication of a hip fracture in some clients. 3. Incorrect: A client who has a hip fracture often appears to have shortening of the extremity on the affected side. This is a result of the location of the break and the positioning of the body in response to the injury and pain.

A client diagnosed with rheumatoid arthritis has been prescribed celecoxib. What should the nurse include in the client's education regarding this medication? 1. Do not take celecoxib with ibuprofen. 2. GI complaints and headache are among the most common side effects. 3. Drink a lot of water to offset the dehydration that may occur. 4. Notify the healthcare provider immediately if black stools are noted. 5. This medication provides relief of pain and swelling so you can perform normal daily activities.

All but 3 Concomitant use of celecoxib with aspirin or other NSAIDs (for example, ibuprofen, naproxen, etc.) may increase the occurrence of stomach and intestinal ulcers. This would increase the risk of GI bleeders. GI complaints and headache are two of the most common side effects. The client should stop taking celecoxib and get medical help right away if the client notices bloody or black/tarry stools. This would be an indication of GI bleeding. This medication is a nonsteroidal anti-inflammatory drug (NSAID), which relieves pain and swelling. It is used to treat arthritis. The pain and swelling relief provided by this medication should help the client perform normal daily activities 3. Incorrect: The client may develop fluid retention while taking this medication. They should decrease the intake of sodium to decrease fluid retention.

Foods high in fiber? Select all that apply 1. popcorn 2. oatmeal 3. cheese 4. raisins 5. oranges

All but 3 (cheese)

What should the nurse teach a pregnant client who comes to the clinic reporting hemorrhoids and constipation? 1. Increased rectal pressure from the gravid uterus may result in hemorrhoids. 2. Hormones decrease maternal GI motility, resulting in constipation. 3. The client needs more fiber in the diet. 4. A mild laxative is recommended to alleviate constipation. 5. The client needs to increase fluid intake.

All but 4 1., 2., 3. & 5. Correct: As pregnancy progresses, the enlarging uterus increases abdominal and rectal pressure. GI motility slows due to hormonal influences. Pregnant clients may benefit significantly from dietary changes including adequate hydration and increased fiber intake. 4. Incorrect: Medications, including laxatives, should not be taken by pregnant women unless prescribed by the primary healthcare provider. If needed, the primary healthcare provider may prescribe a stool softener but rarely a laxative because of possible fluid and electrolyte shifts.

A nurse is planning to provide an education class on preconception health care to a group of young women wishing to become pregnant. What points should the nurse include in this class? Select all that apply 1. Attain a healthy weight. 2. Make sure immunizations are up to date. 3. Avoid drinking alcohol. 4. Learn family health history. 5. Maintain folic acid intake at 200 micrograms/day.

All but 5. 1., 2., 3., & 4. Correct: All of these actions are needed to promote the birth of a healthy baby. A preconception care visit or class can help women take steps for a safe and healthy pregnancy before they get pregnant. 5. Incorrect: Folic acid intake should be 400 micrograms per day in order to reduce neural tube defects by 70%.

_____________ is an antiarrhythmic drug, contains high levels of iodine and may affect thyroid function.

Amiodarone

When is MRI indicated?

An MRI (magnetic resonance imaging) lets your doctor see the organs, bones, and tissues inside your body without having to do surgery. This test can help diagnose a disease or injury. You might need an MRI if an X-ray or CT scan didn't give enough information about your condition.

Chloridazepoxide is what kind of medication?

An anti anxiety and sedative hypnotic medication may cause drowsiness and confusion

Diphenhydramine causes what kind of effect?

Antihistamine with anticholinergic effects. Avoid giving to patients with glaucoma, urinary retention, peptic ulcer, small bowel obstruction

Acetylsalicylic acid is also known as ____________.

Aspirin. May be chewed if not coated to increase absorption

Biophysical Profile (BPP) is what and is done when?

Assessment of five variables in the fetus that help to evaluate fetal risk: breathing movement, body movement, muscle tone, amniotic fluid volume around the baby, and fetal heart rate reactivity. Done in last trimester, but can be done earlier for risk pregnancies Results are: 8-10= good 6: worrisome (redue in 24 hours) >4: ominous (consider immediate delivery)

_____________ is secreted by the ventricles in the heart when ventricular volume and pressure in the heart are increased IF positive can be a sensitive indicator for HF when the CXR (enlarged heart, pulmonary infiltration; fluid in lungs) does not indicate a problem If client is on Nesiritide turn it off 2 hours before drawing a ____.

B-Type Natriueretic peptide BNP

For Celiac Disease what kind of stuff can they NOT have?? -What CAN they have?

BROW (barley, rye, oats, wheat) RCS (rice, corn, soy)

The most common method of daily dosing insulin is? ________ dosing. With Rapid acting insulin give before meals to cover the food eaten at meals. so have food ready

Basal/ Bolus

Why is wound healing, kidney's and eye damage a issue with diabetes?

Because very vascular, and sugar destroys that (high A1C)

How can you help prevent dislodgement of the internal radiation implant: Keep client on ________ Decrease ______ in diet (low residue) Prevent _______ distention. if implant does dislodge and you see it: 1. gloves 2. Use forceps to pick up the implant 3. Place in ______ lined container 4. leave in room, call radiation department

Bed rest decrease fiber prevent bladder distention lead lined container

___________ is the nurses action to do good

Beneficence

___________ can be prescribed in addition to ACE inhibitors. --Relax the vessels, decrease blood pressure, decrease afterload, decrease workload of the heart

Beta blockers

If your liver is sick, what is your number 1 concern? What would you want to decrease? Never give ___________ to people with liver problems. Antidote for this is?

Bleeding Medication Acetaminophen --Acetylcysteine

What is cardiac tamponade?

Blood, fluid, or exudates have leaked into the pericardial sac resulting in compression of heart can happen from Motor vehicle collision, right ventricular biopsy, MI, pericarditis, or hemorrhage post CABG

Hypercalcemia: S/S: Bones are ________ increasing chances of ___________ --Kidney stones - DTR will ________, --muscle tone will become ___________, -Will there be arrythmias? yes or no --LOC will ____________ --Pulse will __________ --Respiration will __________

Brittle, increase change of osteoporosis with HYPERcalcemia -DTR- decrease -Muscle Tone weak and flaccid -Arrythmias- yes -LOC will decrease -Pulse- decrease -Respirations will decrease Hypermag/hypercalcemia= think sedation= think HYPO/ decrease in VITALS and LOC, DTR

Patient teaching after mastectomy include: _________ hair, squeeze ________, wall climbing, flex and extend elbow Why should they do this? Promotes new _____________

Brush hair, squeeze tennis ball Promotes new (collateral) circulation

What can cause third spacing?

Burns, heart failure, hip fracture, hepatic failure, acute intestinal obstruction, acute peritonitis, crushing traumatic injuries, pleural effusion, hypoalbuminemia Ascites- liver disease, fluid in abdomen, hard to breath especially when pushing on diaphragm (measure abdominal girth, worry about hypotension)

IF heart is weak then CO is ______ and Kidney perfusion is ______ , Urinary output is _____. the volume stays in the _________ space to cause fluid volume excess (hypervolemia)

CO, kidney perfusion, Urinary output is all decreased vascular space (1st area the fluid builds up before overloading into the tissues)

Treatment for Hypermagnesia: -Ventilator -Dialysis -Antidote for magnesium toxicity= ______________ this is administered IVP very ___________. (1.5-2 ml/minute) -Safety precautions

Calcium gluconate very slowly

Tissue perfusion is dependent on adequate _____________. _____________ changes according to the body's needs

Cardiac output

Risk factors for abrupto placenta??

Causes: unknown but factors increasing the risks are motor vehicle accidents, domestic violence, previous Cesarean Section, PIH and smoking. Associated strongly with cocaine use.

Hypotonic solutions: goes into the vascular space and then shifts out into the ______ to replace cellular fluid. **they re-hydrate but does not cause _________. Examples- ___________, _________, __________ Used in cases of hypertension, renal, or cardiac disease and needs fluid replacement because of nausea, vomiting, burns, hemorrhage ALERT- watch for cellular edema because this fluid moves out INTO the cells, which could lead to fluid volume _______ and decrease ____________

Cells hypertension D2.5, 1/2 NS, 0.33% NS can lead to fluid volume deficit and decreased blood pressure

An orofacial defect that affects the oropharynx and increase the risk of malnutrition and aspiration

Cleft palate/ cleft lip

_____________ is when brain injured but the skull is not broken, fractured, or penetrated, the dura is not torn . __________ brain is injury where the skull is broken, fractures, or penetrated. The dura is torn in a penetrating injury. *___________ skull fractures are the most serious

Closed TBI Open TBI Basilar

Suicide interventions: Direct _________ ended questions Provide a safe __________ Safe proof the room

Closed ended Environment

The aorta has a narrowing (pretend there is a tourniquet tied around the aorta). This makes it harder for the LV to pump so the client may wind up with left sided heart failure. Signs and Symptoms: - Hallmark Sign: There is a BIG difference in the pulses and ____of the upper and lower extremities. For example, you may have a pediatric client with an Upper extremity BP = 100/60 and Lower extremity BP = 70/40 - The _____ pressures are much greater than the lower pressures. Treatment: - Surgery - Angioplasty in some

Coarctation of Aorta BP Upper pressure

Too many steroids cause ___________ syndrome

Cushing's

Disease with too much Aldosterone?? Disease with to little Aldosterone??

Cushing; Hyperaldosterone, A.K.A Conn's syndrome Addison's disease

This is a genetic disease that involves symptoms related to the exocrine glands and commonly involves both the GI system and the Respiratory system??

Cystic Fibrosis (CF)

Another name for ADH is Vasopressin. So Vasopressin or Desmopressin may be utilized as an ADH replacement in ____________. (may be given nasal spray.

DI

Diabetes Type 1: insulin dependent First sign may be _______. Appears abruptly, classic 3 P's include??

DKA polyuria, polydipsia, polyphagia

Second leading cause of death in cancer clients is??? Why is cancer a big risk for development of this: -Prolonged ________ -Surgery -Use of central line -External ____________ of vessels by the tumor -Invasion of vessels by the tumor -Certain Chemotherapy drugs

DVT Bedrest External compression with DVT most afraid of PE

What days of the menstrual cycle are best for doing the breast self exam

Day 7 through Day 12

hyperemisis gravidarum will see -__________ in Blood pressure -____________ in H&H. -_______________ in UO -_____________ in K+ -______________ weight -Remain NPO for ____ hours.

Decrease BP increase in H&H due to dehydration decrease in everything else NPO 48 hours

Action of Chemotherapy: The goal of chemotherapy is to eliminate or ____________ the number of cancer cells by destroying the cells as they are ___________ Chemotherapy drugs that attack a specific phase of cell development are called cell __________ chemo drugs Chemo Drugs that work at all or any phase of cell development are called cell cycle _________ Drugs work best while tumor cells are actively _________. most active growth is when the tumor first begins to develop. EARLY DETECTION for cancer is best

Decrease, growing Specific non-specific growing

Cardiac Tamponade: ____________ Cardiac output HALLMARK SIGNS CVP will be __________ BP will _________ heart sounds will be? Neck veins will be? Would you have a narrowed or widened pulse pressure?? -what is pulse pressure??

Decreased CO CVP will increase (fluid) BP will decrease Heart- muffled or distant. pressure in all 4 chambers the same Neck veins distended Narrow pulse pressure (for widened pulse pressure think increased ICP) -pulse pressure is the difference between systolic and diastolic reading

What happens to the growth rate between 6-12 years of age?? A school age child requires, on average, how many calories per day?? How much earlier do girls experience the onset of adolescence than boys??

Decreases- slows down 2400 calories 1-2 years.

The major reason alcoholics don't seek treatment?

Denial and rationalization

Nitroglycerin: Causes venous and arterial _____________ --this will cause a __________ in preload and afterload Also causes dilation of the _____________ arteries to increase blood flow to the actual heart muscle (Myocardium) How should it be taken???? Should it be swallowed???? How is it stored?? Normal side affect?? After Nitroglycerin what do you expect the BP to do???

Dilation, decrease preload and afterload Coronary arteries 1 every 5 minutes, 3 doses. DO NOT swallow Keep in DARK, glass bottle, dry and cool May cause headache (normal) BP will drop due to vasodilation. WHEN CHANGE IN BP STAY WITH CLIENT AS THEY MAY BE UNSTABLE AND FALL RISK

Hyponatremia = ______ (Too much ____, not enough _____ ) Causes: Drinking _______ for fluid replacement (vomiting, sweating), this only replaces water and dilutes the blood (still losing sodium without replacing it) --_______ solution (sugar and water) --__________ : retaining water S/S: ____________, ___________, ____________ Treatment: give _________, restrict _________. If having neuro problems may need ___________ solution (means packed with particles, 3% NS, 5% NS)

Dilution (too much water, not enough sodium) -water -D5W -SIADH headache, coma, seizures, Give sodium, restrict water. may need hypertonic solution

Medication effects on Cardiac Output: Diuretics (Furosemide) and Nitrates: how do they affect preload?? ACE inhibitors (PRIL), ARBS (SARTAN), Hydralazine, Nitrates: How do they affect afterload?? Inotropes (dopamine, dubutamine) __________ contractility _________ control drugs include Beta Blockers, Calcium Channel blockers (Diltiazem, verapamil, "dipine"), Digoxin ____________ control drugs include Antiarrhythmics (amiodarone)

Diuretics and nitrates- preolad: vasodilation or diureses to reduce (decrease) preload. Afterload: these vasodilate to reduce afterload Improve contractility Rate control Rhythm control

When viewing the auditory canal or using ear drops in young children (under 3) how is the earlobe positioned?

Down and back

The most common birth defect in the U.S, and is also known as Trisomy 21?? What type of infections are children most prone to develop?? WHY?? What is the most common type of physical defect associated with Trisomy 21?? Risk factors for this??

Down syndrome Respiratory infections, poor immune system Congenital heart defect Advanced maternal age, primary aim in genetic counseling is to inform the parents of their risk.

this is the most common and severe muscular dystrophy of childhood and is inherited as an X link recessive trait.

Duchenne Musclar dystrophy

-With basilar skull fracture, you see bleeding where? -Battle's sign: bruising over the ___________. -_________ eyes (peri orbital bruising) -Cerebrospinal rhinorrhea- leaking spinal fluid from the _____________. -How do we tell CSF from other drainage. Positive for ___________ and the halo test.

EENT (eyes, ears, nose, throat) -Mastoid -Racoon eyes -nose -glucose

Gallstones drop into the pancreas causing occlusion in the pancreas so enzymes in the pancreas cannot get out causing those enzymes to ____ the pancreas

Eat

Bleeding precautions include: __________ razor _________ toothbrush NO ____ injections ________ play for children (includes when the spleen is affected)

Electric razor soft toothbrush NO IM injections Quiet play

What kind of hematoma works by: injury- loss of consciousness- recovery period- bleeding into their head- can not compensate anymore-neuro changes Treatment: -Ask them did they pass out and stay out? -________ to remove the clot, stop the bleeding and control ICP

Epidural hematoma Burr holes

what is a serious obstructive inflammatory process: there is absence of a cough presence of dysphagia, drooling, and rapid progression to severe respiratory distress. what is the primary organism that causes this and what vaccine can make it rare?

Epiglottis H-Flu Hib vaccine

If vascular check in an extremity is decreased, what procedures will help relieve pressure?

Escharotomy, Fasciotomy (both theses relieve pressure and restores circulation)

With hypertension there is even more ______________ for the left ventricle to pump against. That's why hypertension can eventually lead to _________ and ____________, because high afterload ___________ cardiac output and __________ forward flow. Plus, it wears the heart out

Even more resistance. Lead to HF and pulmonary edema High after load decreases CO and Decreases forward flow

Glucocorticoids (cortisol) is responsible for breakdown of ______ and protein to help regulate glucose metabolism. may also have higher risk for ___________ when there is too much

Fat and proteins Infection

Early sign of blood cancers and lymphoma?

Fatigue is the #1 symptom. Exhaustion after adequate rest May also report pain

Placenta previa complications

Fetal Complications: Preterm delivery. Intrauterine growth retardation. Fetal distress. Anemia Maternal Complications: Hemorrhage. Potential DIC risk. Delaying birth and treatment with corticosteriods to mother speeds maturation of fetal lungs and may increase maturity and birth weight until delivery is necessary.

what is positive signs of pregnancy?

Fetal Heartbeat --Doppler 10-12 weeks gestation --Fetoscope 17-20 weeks gestation -Fetal movement felt by provider -Ultrasound

__________ is the duty to keep promises

Fidelity

48 hours after burn client will start to diurese. WHY??

Fluid shift back into vascular space, now worry about FVE, Urine output will increase

Major problem with oral potassium??? Give with ??? IV potassium should always be on a ______. Never give IV PUSH. Should be diluted May burn during infusion. it is vesicant to the veins

GI upset With food

Weight _______ in mild depression, weight ________ in severe depression ___________ spells in mild depression to moderate depression ______ with severe depression Can clients who have depression experience delusions and hallucinations??

Gain; loss Crying sells No more tears yes=major depression

__________ ulcers- appear malnourished; pain is usually half hour to 1 hour after meals, food does not help, but __________ does. vomits blood __________ ulcer- appears well nourished; night time pain is common and also occurs 2-3 hours after meal. _________ helps, blood in stool

Gastric ulcer, food does not help, but vomiting does Duodenal ulcers- Food helps

Peptic Ulcer Diagnosis: -____________ can be done, client should be NPO pre procedures, sedated, NPO until _____ reflex returns. --Watch for perforation by watching for pain, bleeding. or if they have trouble swallowing

Gastroscopy- NPO until gag reflex returns

If metformin is not controlling the blood glucose levels, another anti diabetic will be ordered. possibly __________ -Clients undergoing surgery or any radiologic procedure that involves contrast dye should temporarily discontinue metformin. They can resume ____ hours after the procedure if kidney function has returned and the creatinine level is normal

Glargine (lantus) 48 hours.

adrenal cortex steroids include??

Glucocorticoids, Mineralocorcticoids, Sex hormones

thrombolytics: Goal of therapy is to ?? Examples of medications include? What is the time frame to give this in MI clients? What about for stroke clients? Major complications include _____________ Absolute contraindications include what 4 things??

Goal- dissolve the clot that is blocking blood flow "plase" Alteplase (t-PA), tenectelplase, reteplase MI- 6-8 hours Stroke- 3 hours Bleeding not for intracranial neoplasm, intracranial bleeding, suspected aortic dissection, or internal bleeding

Pain that begins in the great toe is caused by ______, a condition triggered by meats, dense proteins, and alcohol.

Gout

allopurinol used for

Gout Avoid caffeine, can be crushed, increase fluid intake, Can be used with ibuprofen for discomfort

how to don PPE

Gown, maks, goggles, gloves

Hyperthyroid is TOO MUCH energy. Known as _________ disease.

Graves disease

Normal breath sounds over trachea?

Harsh hollow sounds

What is a contraindication for thrombolytic therapy?

Heat trauma within 3 months is an absolute contraindication

Thyroidectomy (partial/complete): -Post op priority: monitor for _____________ - Report feelings of __________ -Check for bleeding where?? -Assess for recurrent laryngeal nerve damage by listening for _____________. ----Could this lead to vocal cord paralysis?? ---When there is paralysis of both cords, _______ obstruction will occur requiring immediate __________

Hemorrhage Pressure in the neck Check neck and incision site and back of neck -Listen for hoarse voice (weak voice) -Paralysis- yes -Airway obstruction and immediate trach

Bone marrow and stem cell transplants are primarily used as treatment for the ___________ cancers. ___________ transplant- when stem cells are transplanted from the BLOOD STREAM ___________transplant- Stem cells are transplanted from bone marrow Stem cells from the blood or the bone marrow can come from the client themselves, a matched donor, or from ____________ siblings or twins Stem cells are given into a _______, much like a blood transfusion, and over time they settle in the bone marrow and produce healthy blood cells

Hemotologic cancers Stem cells- blood stream Bone marrow- stem cells from bone marrow identical Vein

____________ is caused when a hole in the diaphragm is too large, so the stomach moves up into the thoracic cavity Treatment: -____________ meals - Sit up __ hour after eating -Elevate ______ -Lifestyle changes

Hiatal Hernia -Small frequent meals -sit up 1 hour after eating -elevate HOB

Reed-Sternberg cells - diagnosis?

Hodgkin's lymphoma (most treatable)

All clients given Naloxone in the field must go to the __________. Naloxone is short acting and must be administered every few __________ until opiate levels are non toxic Opiate is still in system so it is important to monitor and continuous naloxone may be needed

Hospital hours

what to do if expected abruptio placenta??

Hospitalization and evaluation at once. Conservative treatment of bedrest, fetal monitoring and administration of tocolytics is rarely the chosen option. Method of delivery: C-section. RULE: Do not do vaginal exams in the presence of unexplained vaginal bleeding. Two priorities: manage fetal status and maternal shock.

autosomal dominant disorders

Huntington's disease

In this condition there is a disturbance of the ventricular circulation of cerebral spinal fluid. The cerebral spinal fluid builds up and causes increased intracranial pressure (ICP)

Hydrocephalus

You enter a diabetic client's room and they are unconscious. Do you treat this client like he is hypo or hyperglycemic?

Hypoglycemia= more dangerous

Liver disease treatment: Ascities= ______tension= increase in ascities= decrease in fluid in the __________ space. Paracentesis: --Have client ________. --How should client be positioned?? --Anytime you pull fluids you can throw them into _______. --Monitor vitals signs. shock will have _____ BP & ______ HR Avoid _______- liver can not metabolize drugs well when its sick Diet: -Decrease _________ and __________ -Increase in protein causes an increase in _________ with liver disease clients

Hypotension, vascular space -have client void --Sitting up so fluid can settle --Shock --Decrease BP and increase HR --Avoid Narcotics Diet: Decrease protein and sodium Protein= Ammonia (decreases LOC

If a client is restless think ________ FIRST! ***restless and tachycarida other signs are sleepy, lethargic, confusion, headache

Hypoxia

Hypocalcemia: -causes: hypoparathyroidism, radial neck, thyroidectomy S/S- same as hypomagnesemia Treatment: PO Calcium -IV Ca (GIVE ___________) and always make sure client is on __________ monitor (Decrease HR, wide QRS, if calcium is given to fast). -Vitamin D -May also give _________ binders cause of the inverse relationship (calcium acetate, sevelamer hydrochloride)

IV SLOWLY AND HEART MONITOR phosphate

In burn victims why is IV meds preferred over IM?

IV is quicker, and IM to work correctly there must be adequate perfusion to the muscles. DRUG OF CHOICE: OPIOIDS

When does a nurse initiate reverse precatuions?

Immunocompromised, such as client who has cancer or HIV infection

Unstable chronic angina = ___________

Impending MI

When "MYCIN" antibiotics are given (gentamycin)... WORRY about ????

Increase in BUN or creatinine, or reports of hearing loss known to cause ototoxicity (irreversible), and/or Nephrotoxicity. (check BUN and creatinine, if increase assume nephrotoxicity.... ALWAYS ASSUME THE WORST))

If you have hypervolemia you have ___________ in volume, __________ in preload, __________ in workload

Increase in all. More volume = more pressure = more preload and workload for the heart

Why does plasma seep out into the tissues after burn? ----Increase ___________ permeability. --- when does the majority of this occur? (worry about shock)

Increased capillary permeability... leaky vessels first 24 hours

Thyroid storm (thyrotoxic crisis) could be rebound effect of thyroidectomy or post radioactive iodine therapy. S/S of thyroid storm include??

Increased fever (usually severe), Can lead to Heart attack, MEDICAL EMERGENCY. It is hyperthyroidism multiplied by 100

What is a hydatidiform mole? and what does it do to hCG levels?

Increases hCG levels similar to pregnancy. also called Trophoblast It is a benign neoplasm grape like vesicles that can become malignant If not malignant, a D&C is required with close follow up for 6 months to 1 year If the molar pregnancy is malignant, treatment will be based on the stage and grade of the cancer. Tell patient not to get pregnant for at least a year so hCG levels can decrease

What is the Triad symptoms of MI in women?

Indigestion or feeling of fullness Unusual fatigue inability to catch "one's" breath

the primary focus in chicken pox is to prevent ____________.

Infection

four phases of acute kidney injury?

Initiation phase (injury occurs) Oliguric phase (output <100 mL/ 24 hours) Diuretic phase (kidney recovering) Recovery phase (3-12 months)

Order for physical assessment

Inspection Palpation Percussion Auscultation For abdomen assessment it is: Inspection auscultation percussion palpation

Diabetes Type 2: Insulin ___________. (Not enough insulin or insulin is ineffective) This type is usually found by accident. or the client keeps coming back to provider for things like __________ that wont heal, repeated __________ infections, etc. Do you see Metabolic acidosis? Ketones? DKA? Break down of fat?

Insulin resistant wounds that wont heal, repeated vaginal infections No to acidosis, ketones, DKA, break down of fat

Hallmark sign for arterial disorders Arterial disorders of the lower extremities are usually treated with ?

Intermittent claudication Angioplasty, endarterectomy

If the Glasgow coma cale is below 8, think ___________?

Intubate

Name for the condition where a piece of the bowel telescopes in to itself forming an obstruction?

Intussusception

chronic stable angina: Intermittent decrease in blood flow to the myocardium that leads to _______________. This can lead to temporary pain/pressure in chest. What brings this pain on: Low _________ usually due to ___________. What relieves the pain: ______ and/ or ______________ Sublingual.

Ischemia Low oxygen due to excretion Rest and Nitroglycerin

what type of exercises are avoided for angina patients?? When taking Nitroglycerin tell patient to ___________ because they may be dizzy due to decrease BP

Isometric exercises. (activity that uses muscle tension to improve muscular strength with little or no movement of the body part) Sit down

What is D-dimer used for? The most sensitive and specific test for PE??

It is used to rule out PE. Cannot rule in. -will tell if clot is located anywhere in the body. not just in the lungs) pulmonary angiography (real invasive though)

When the lungs have issues with acid base problems what organ compensates? When respiratory acidosis is occurring the kidneys will excrete ______ and retain ________

Kidney Hydrogen, retain bicarb

Magnesium is primarily excreted through ___________ but can be lost in other ways (GI tract)

Kidney's in urine

For Tetralogy of Fallot put infants in the __________ position. WHY?? have them use ________ play, minimize stress (respond to crying quickly).

Knee chest position. this decreases venous return from the lower extremities (the desaturated blood) and increases systemic resistance which diverts the blood to the pulmonary artery for oxygen quite play

DKA - metabolic acidosis compensates by causing ___________ respiration's. Insulin Therapy for DKA includes _________ insulin IV. ---insulin therapy can ________ potassium

Kussmaul respiration Regular insulin IV decrease potassium

Risk factors for thrombocytopenia: Bleeding disorders such as: -Hemophilia -___________ disease - ITP (idiopathic thrombocytopenia purpura) - Anticoagulant medications- Aspirin, Clopidogrel, Heparin, warfarin -cancer treatment Treatment give _____________. Never infuse them _________. Need to be ________ temperature of spleen will reject them

LIVER disease can cause thrombocytopenia Platletts, never infuse cold. needs to be room temperature

the most sensitive indicator of neuro status. A change in _________ may be the first sign that there is a problem

LOC is always #1 with neurological assessment

What is Hirschsprung's disease? this usually affects the ????

Lack of ganglion cells in the distal colon, leading to uncoordinated peristalsis and reduced motility. Also known as aganglionic Mega colon, results in mechanical obstruction usually affects the sigmoid colon

What med is give for ammonia toxicity? Decrease protein in diet

Lactulose- draws ammonia into the colon to be pooped out.

Anorexia Nervosa: -They may have ____________ (soft hair on newborns to keep them warm) -May have ___________ due to loss of subcutaneous tissue -Decreased ____________ development - Dehydration and electrolyte imbalance will result in _____ BP and Low Pulse Treatment: - Increase _________ gradually Weight them in their underwear because they will manipulate weight - Monitor _________ routine - Monitor suicide thoughts

Lanugo hypothermia sexual LOW Weight exercise

A viral infection that can result in: -Slight to severe dyspnea -Barking or brassy cough -Elevated temperature -Viral organisms include parainfluenza, adenovirus, RSV (they will sound like a barking seal)

Laryngotracheobronchitis (CROUP)

Renal threshold for glucose is elevated in the elderly. So if a patient has diabetes why is a blood glucose test preferred over urine test for glucose testing?

Level at which glucose starts to appear in the urine increases, leading to false negative readings.

Treatment for hypothyrodism: _____________ medication is most common, may also use liothyronine Give on _________ stomach in the morning. -Start medication with low dose cause or risk of _____. Drug will increase ____, _________, and ____. People with hypothyrodisim tend to have ______ (worry about ____ when starting mediation regimen) --Do they take these medication forever?? -what will happen to their energy level when they start taking these meds??

Levothyroxine Empty Risk of MI, Increase HR, workload, BP CAD, worry about MI Yes forever Energy increases

What to do for prolapsed cord?

Lift head off cord until provider arrives. Place client in Trendelenburg or Kneechest position. Administer oxygen. Monitor fetal heart tones. Check FHTs when membranes rupture (spontaneously or artificially). If cord is being compressed, variable decelerations in FHTs occur and immediate delivery is needed, usually C-section. **DO NOT PUSH CORD BACK IN

If hyperventilation causes respiratory alkalsosis what are some signs and symptoms of this?

Lightheaded or faint feeling (has cerebral vasoconstriction), numbness and tingling in the fingers and toes

What is the reason for methadone?

Long acting opioid that can substitute for addiction and be titrated downward during rehab to ease withdrawal symptoms Users do not crave methadone. they now have time to do purposeful things such as work, school, and therapy

Duchenne's muscular dystrophy (DMD) S/S: -_______________ (curvature of lower spine) -____________ gait -Frequent _________ (weakness in the muscles) -____ walking -___________ sign (classic sign of DMD) Treatment -Maintain optiumum ___________ function with physical therapist -Prevent _____________ -____________ may help improve muscle strength and respiratory function -Support groups and ____________ care as disease progresses

Lordosis Waddling Falls toe walking GOWERS sign -muscle function -prevent contractures -Steroids -Palliative care

Treatment of IBD: Diet should be high or low residue? --try to limit GI motility to help save _______. -Avoid ______ foods and smoking (they increase motility)

Low Fluids cold

The best time to perform SBE is one week after the onset of the woman's menstrual period, when hormones are at the ___________. It is normal for hormonal changes during the month to make breast tissue tender or enlarged.

Lowest

Pulmonary artery is deoxygenated blood that takes blood to the __________. Pulmonary vein is oxygenated blood that takes blood to the __________

Lungs Left Atrium

Panic disorder starts in late 20's. client may go to ED with classic symptoms of an _____ but may be a panic attack

M.I

Acute coronary syndrome (ACS) is a term used to describe: __________, ____________ -decrease blood flow to the myocardium = ischemia, necrosis or both??? Does the client have to be doing anything to bring this pain on?? Will rest or nitroglycerin relieve this pain??

MI, Unstable angina Both No No

Treatment for hypercalcemia: --_________ around can cause calcium go transfer into bones and out of blood --Ca can have INVERSE relationship with _________. so add __________ to diet --Steroids --Safety precautions (sedative ---Medications to decrease serum Ca, include ______ and __________ (puts Ca back into bones and out of blood)

MOVE around Inverse with phosphate, add phosphate to diet (protient) Biphosphates (" NATE" Alendronate (Fosamax), Risedronate Ibandronate) Calcitonin

Airborne precautions

MTV (measles, TB, varicella)

normal hemoglobin levels

Male: 13-18 g/100mL Female: 12-16 g/100mL

normal hematocrit levels

Male: 45%-52% Female: 37-48%

In burn cases what diuretic can be ordered to flush out kidneys?

Mannitol

DKA, STARVATION, Renal failure, and severe diarrhea can cause what acid base balance??? Kidneys have the problem so lungs compensate...what will happen to respiratory rate to compensate?

Metabolic acidosis Increase Respiration to blow off acid (hyperventilation)

Individuals with Type 2 diabetes should be evaluated for __________ syndrome (syndrome X)

Metabolic syndrome

______________ is the first choice of oral anti-diabetic medication. It works to decrease the amount of circulating __________. It is a favorite because it reduces glucose production and enhances how glucose enters the cell. Does NOT stimulate the release of more insulin, so does NOT cause ___________ with this drug. Which is good cause we do not want hypoglycemia because it destroys vessles!

Metformin glucose hypoglycemia

Anti thyroid drugs give for hyperthyroidism: _________ and ________. Stops the thyroid from making thyroid hormone. Can be used pre-op to stun the thyroid. We want this client to become Euthyroid (EU= __________)

Methimazole (most common), Propylthiouracil EU= normal

Contraction Stress Test (CST)

Method for evaluating fetal status during the antepartum period by observing response of the fetal heart to the stress of uterine contractions that may induce by oxytocin. check for recurrent episodes of fetal hypoxia. -looking for deceletations. do NOT want to late decelerations. Want this text to be negative. Test rarely perfromed beofre 28 weeks cause it causes uterine contraction and risk of early baby. Results are good for only one week

what is used to stop the growth of the embryo in ectopic pregnancy so that the fallopian tube can be saved?

Methotrexate

Magnesium and calcium normal lab values? Magnesium and Calcium act like a _________. HINT if you want to get MG and CA questions right think ________ first.

Mg: 1.3 to 2.1 Calcium: 9.0- 10.5 acts like a sedative muscles first (may include lungs and heart as they are muscles)

A spontaneous abortion is also called ______________ Signs of this include _____________, ____________, __________ (THINK SPONTANEOUS ABORTION) The hCG level will begin to ___________ Most occur before ____ weeks gestation

Miscarriage Bleeding, cramping, backache (THINK MISCARRIAGE) decrease 20

Neutrophils

Most abundant white blood cell., The most abundant type of white blood cell. Phagocytic and tend to self-destruct as they destroy foreign invaders, limiting their life span to a few days.

Why is the ventrogluteal muscle for IM injections not used in children who have not been waling for at least a year?

Muscle is not developed well enough

If your CO is decreased, will you perfuse properly?? Brain: LOC will go _________ Heart: Reports __________ pain Lungs: Sound _______. Short of breath: yes or no? Skin: ________ and __________ Kidneys: UO goes _________ Peripheral pulses: _____

NO Brain: LOC will go DOWN Heart: Reports CHEST pain Lungs: Sound WET/CRACKLES. Short of breath: YES Skin: COLD and CLAMMY Kidneys: UO goes DOWN Peripheral pulses: WEAK

For a child with heart defect, would you give them something to do that will get them excited and increase their HR?

NO because that would increase the workload on the heart, and we NEVER want to increase the workload on the heart of a client with heart disease.

Associated nursing care, if any lymph nodes were removed with a mastectomy: Avoid procedures on arm of the affected side for lifetime --NO ___________, _____, ______ with elastics, ___________, __________ on the wrist, IV or injections. ---Wear _______ when gardening, watch small _____, No nail ________, and No ________ or _______

NO constriction, BP, Blouses, purses, watches Wear gloves, watch small cuts, no nail biting, no sunburn

Hypothyrodism/Myxedema: S/S: No ________. Speech ______ and ________, fatigue, Weight ______, GI _____, ___________ intolerance, Amenorrhea (absent periods) Why would you not want to give them a heating pad? T4 will be __________, TSH will be ___________.

NO energy, slow and slurred speech, weight gain, GI slow (constipation), cold intolerance. -may not tell when heating pad is too hot T4- decreased TSH-increased

RECAP ____= looks for acceleration sin HR. want reactive test ____= look for D-cells (late). want negative test

NST= looks for acceleration sin HR. want reactive test CST- look for D-cells (late). want negative test

Limit protein with kidney problems except with ____________ and __________ dialysis patients

Nephrotic Syndrome Peritoneal Diaylsis

Lidocaine Toxicity: Any ___________ changes Amiodarone is the FIRST anti-arrhythmic of choice --important side effect is ___________. This can lead to further arrhythmia

Neuro (too much "caine" drugs think neuro changes Low BP. Hypotension can lead to futher arrhythmia

When sodium levels is affected think ________ changes. -Sodium levels in blood is totally dependent on how much ________ you have in your blood

Neuro changes water

Glascow Coma Scale (GCS) measures? -what numbers do we want?

Neurologic assessment of a patient's best verbal response, eye opening, and motor function. *13-15

Filgrastim indications

Neutropenia, increases hematopoietic stem cells in autologous bone marrow transplants

Exercise teaching in Diabetes: -Wait until blood sugar __________ to begin exercise. - _____ before exercise to prevent hypoglycemia

Normalizes eat exercise when blood sure is at its highest, and same time and amount daily

What is the main reason for ascities in liver disease?

Not enough albumin to pull fluids into vascular space

Nausea and vomiting is most common side effect of chemotherapy and a routine antiemetic is given first week of chemo _____________ is a common antiemetic that works by blocking the effects of serotonin, so its serotonin receptor antagonist

Ondansetron Serotonin receptor antagonis end in "tron" (ondansetron, granisetron, dolasetron)

_________- are considered "gold standard" for cancer pain. There is NO ceiling on the dose for pain in cancer clients, it is client dependent. Limit dose is based on side effects, not dose and time. Call provider for increased in dose if needed

Opioids

Wernicke-Korsakoff syndrome

Organic brain syndrome resulting from prolonged heavy alcohol use, involving confusion, unintelligible speech, and loss of motor coordination. It may be caused by a deficiency of thiamine, a vitamin metabolized poorly by heavy drinkers.

infection in the middle ear characterized by bulging, bright red, tympanic membrane and is usually preceded by an upper ______________ infection

Otitis Media -Respiratory

What medications are used for chest pain when they get to the ED?? Know the order Why do we want head up position: --Decreases ______________- on the heart and increases ____________

Oxygen (if O2 < 90%) Aspirin (chewable- platlet agrigation) Nitro Morphine "MONA" but needs to be in this order listed above Decreases workload, increases CO

Arteries carry __________ blood. Veins carry ___________ blood -inflammation and chronic ulcers can occur with _______ disorders.

Oxygenated; deoxygenated venous

If chest pain after PCI what should be done?

PCI includes all interventions like PTCA and stents. If pain call the provider right away. could be reoccluding!!

Acyanotic Defects

PDA ASD VSD coartcation aortic stenosis pulmonary stenosis mitral stenosis mitral valve polapse

Radioactive iodine therapy: Given ____ route in one dose. --Used to destroy ______ cells. so ___________ is an expected outcome. --Rule out __________ first. following radioactive precatuions: -Stay away from _______ for one week -DO NOT _________ anyone for one week

PO Destroys thyroid cells, Hypothyroid is expected Rule out pregnancy Stay away from babies Do not kiss anyone

PPI drugs?? H2 antagonist drugs?? ________ is given to form a barrier over the wound so acid cannot get on the ulcer

PPI- "ZOLE" pantoprazole, esomeprazole, omeprazole H2= "tidine" Ranitidine, Famotidine Sucralfate

Normal PT/INR

PT: 10-13s INR: <1.2

Hypercalcemia: Causes- to much _____ When serum calcium gets low, the hormone kicks in and pulls Ca from the bone and puts it in the blood. causing serum Ca to increase --________ retain calcium --Immobility cause of lack of ?

PTH- parathyroid hormone Thiazides Immobility cause of lack of bearing weight to bones so keeps Ca from going into the bone and makes it stay in the blood

Circulation checks "5 P's"

Pain Paresthesias Paralysis Pulse Pallor Pulse, skin color, skin temperature, Cap refill, sensation

When client is restless after burn injury, it could suggest three problems: _______, inadequate _____________, ___________ Nurse priority __________. To assess fluid volume is adequate for burn victims would you measure weight or urine output??

Pain, inadequate fluid replacement, hypoxia pain never killed anyone, focus on hypoxia urine output (30-50 ml/hr in adult, 1 ml/kg/hr for child. ) for electrical injury UO of 75-100 ml/hr is goal

Why is insulin given for pancreatitis clients?

Pancreas is sick and TPN makes sugar levels increase

_____________ enzymes must be given to help improve digestion with every meal and every snack. With cystic fibrosis these enzymes are lacking and need to be taken WITHIN 30 minutes of eating. and the beads should NOT be crushed or chewing with cystic fibrosis nutrition is a major concern. Usually underweight from digestive problems. Must be on a well balanced high _________ and high __________ diet.

Pancreatic enzymes High fat and high calorie diet

____________ is auto digestion of the pancreas Pancreas has two functions: Endocrine- __________ Exocrine- ____________ Two types #1 cause = __________ #2 cause= __________

Pancreatitis Endocrine- Insulin Exocrine- Digestion #1= gallbladder disease #2- Alcohol

Normal fetal circulation: Blood goes Right Atrium....Right Ventricle.... Pulmonary Artery...doesn't go to lungs....instead when blood leaves PA it goes straight over to the aorta via the ductus arteriosis. Why does the blood do this? Because in utero the baby's lungs are collapsed, and the baby gets oxygen through the placenta. This ductus arteriosis is supposed to close when the baby is born and takes their first breath. When it closes then blood flows from the Pulmonary Artery to the lungs etc....just like in the adult. But when it doesn't...... This leads to increased workload on the left side of the heart and therefore left sided heart failure. Some blood is going like it's supposed to, but some is going over to the right side because the left side is pushing it to the right. Signs and Symptoms: - May be asymptomatic - May be in heart failure - They have a machinery - like murmur Treatment: - ____________ (Indocin®) (prostaglandin inhibitor) will close PDA - May need surgery to close the ductus

Patent Ductus Arteriosus (PDA): Treatment- Indomethacin

Client should eat with insulin is at its _______? When insulin is at its peak, the blood sugar is at its __________.

Peak Lowest

What antibiotic is given for mastitis?? can you still feed the baby during mastitis and treatment with antibiotics?? Feed the baby frequently and on the _____________ breast first

Penicillin (erythromycin if allery) Yes. take antibiotic right after feeding baby Affected breast first

PCI stands for what? what is this procedure

Percutaneous coronary intervention (placement of a catheter and stent in a coronary artery to open the artery; balloon angioplasty)

Treatment for Cardiac Tamponade?

Pericardiocentesis (to remove blood from around the heart) and surgery. Position the patient in a semirecumbent position at a 30- to 45-degree angle. This position brings the heart closer to the anterior chest wall.

Severe perineal pain after labor and delivery could be a sign of?

Perineal hematoma. Medical emergency

Triad of opiate intoxication? Give _________. May be given IV, IM, SQ, or intra nasally

Pinpoint pupils Respiratory depression Coma Naloxone

How should a client be positioned after a tonsillectomy? why are brown and red fluids not give post operative? what would indicate that hemorrhage is occurring?? how many days post op is at risk for hemorrhage? WHY? Common complaints post op? what can bad breath be a sign of?

Place on their side and elevate head of bed. they may also be placed prone to prevent aspiration. We do not want to have anything that can be confused with bleeding Frequent swallowing 10 days. the scabs on the surgical sites will begin to slough off at 7 to 10 days. no chips or rough foods for several days sore throat and slight ear pain. low grade temp. Foreign body in the nose. bad breath can be caused by infection

Thrombocytopenia is the decrease in the number of circulating __________ in the blood. _________ are responsible for clotting? Must have ____________ to clot blood to prevent bleeding

Platletts

What causes hyperkalemia in a burn victim?

Potassium is found inside the cell. when burned cells rupture and lyse. thus increasing potassium in the vascular space...cause hyperkalemia

Describe preolad and afterload of the heart??

Preload is the amount of blood returning to the RIGHT side of the heart and the muscle STRETCH that the volume causes. ANP (Artrial natriuretic hormone) is released when we have this stretch to help excrete sodium and water and decrease BP Afterload is the PRESSURE in the aorta and PERIPHERAL ARTERIES that the left ventricle has to pump against to get the blood out. This pressure is referred to as resistance. In other words the WORKLOAD of the heart is defined by the Preload, which affects amount of blood that goes to the R ventricle. Afterload is the resistance the blood has to overcome when leaving the heart.

Diuretics and Low sodium diet can decrease _____________ Report weight gain of ____ to ____ lobs in a week. WHEN YOU HAVE FLUID RETENTION- THINK ________ PROBLEM 1ST

Preload= decreases the volume and fluid retention 2-3 LBS HEART problem first

What is abruptio placentae? S/S -___________ like abdomen with or without vaginal bleeding -___________ pain, uterine tenderness at site or separation. Dull back pain

Premature separation of a normally implanted placenta after the 20th week but before birth Rigid, board-like abdomen, with or without vaginal bleeding. Abdominal pain, uterine tenderness at site of separation, and increased uterine tone. Difficult to palpate fetus. Dull back pain.

What is the name of the hormone that induces amenorrhea? what does it do to temperature during ovulation?

Progesterone increases temp

what is the normal pathway of ammonia without liver disease?

Protein -> breaks down ammonia -> the liver converts ammonia to urea -> Kidneys excrete the urea

Two main things needed in diet to promote healing??? What lab work is checked to ensure proper nutrition and positive nitrogen balance??? Elevation of the extremity may help improve circulation by reducing _________

Protein and Vitamin C Prealbumin Edema

What untreated arrhythmias will put the client at risk for sudden death?

Pulseless V tach V fib Asystole

Arrhythmias are no big deal UNTIL they affect your cardiac output. The THREE that are always a big deal are ____________, ___________, ______________

Pulseless v-tach V-Fib Asystole

Chronic arterial vs chronic venous disorders: Pulses? Color? Temperature? Edema? Skin changes? Ulceration? Gangrene? Compression?

Pulses: arterial= decreased or may be absent Venous= Normal (may be difficult to palpate cause of edema) Color: A- pale when elevated, red with lowering of leg v- Normal (may see petechiae or brown pigmentation with chronic condition Temperature: A- Cool, V- Normal Edema: A- absent or mild, V- present Skin changes: A- thin, shiny, loss of hair over foot/toes, nail thickening V- Brown pigmentation around ankles and possible thickening of skin, scaring may develop Ulceration: A- If present will involve toes or areas of trauma on feet (painful) V- If present will be on sides of ankles Gangrene: A- may develop, V- does not develop Compression: A- not used. V- used

40 mg of Furosemide IV push for pulmonary edema should be give ___________ over 1-2 minutes to prevent hypotension and ototoxicity

Push IV SLOW

A disorder that results in projectile vomiting during or after a feeding. -on assessment of the abdomen, it's possible to feel an olive shaped mass in the epigastric region, near the umbilicus. It's the enlarged pylorus Treatment is usually surgery. But why would you think its important to monitor urine specific gravity??

Pyloric stenosis Monitors dehydration status

Leading cause of lower respiratory tract illness in children LESS than 2 years of age, caused by an acute viral infection that affects the bronchioles

RSV (Respiratory Syncytial Virus)

Arterial lines can be placed in multiple arteries, but the most common site is the _______ artery. It provides continuous intra-arterial blood pressure monitoring and allows for repeated ABG samples to be collected without injury to the client

Radial

Only __________ insulin is used in infusion pumps. Obtains better control: receiving a basal level of insulin from the pump and boluses of additional insulin as needed with meals, or if they have and elevated blood sugar.

Rapid acting insulin

Hypermagnesium: Causes: __________, Antacids S/S: _________ and ________ of the skin, MG causes ____________ - DTR will ________, --muscle tone will become ___________, -Will there be arrythmias? yes or no --LOC will ____________ --Pulse will __________ --Respiration will __________

Renal failure warmth and flushing due to vasodilation -DTR- decrease -Muscle Tone weak and flaccid -Arrythmias- yes -LOC will decrease -Pulse- decrease -Respirations will decrease Hypermag/hypercalcemia= think sedation= think HYPO/ decrease in VITALS and LOC, DTR

Hyperkalemia: causes: ________ failure/ trouble, __________ diuretic, S/S: Begins with _________ twitching, then proceeds to _________ weakness, then flaccid paralysis. ECG changes include???? Treatment: ________ (kidneys aren't working) --__________ to decrease arrythmias ---Glucose and insulin. Insulin carries ______ and potassium into the cells (anytime IV insulin- monitor for hypoglycemia and hypokalemia, --__________ (Kayexalate)

Renal failure, spironolactone Muscle twitching, muscle weakness, flaccid paralysis ECG- bradycardia TALL and PEAKED T waves, prolonged PR interval, flat or absent P waves, Widened QRS, conduction block, V fib -Dialysis -Calcium gluconate -Insulin carries glucose -Sodium polystyrene sulfonate

ACE inhibitors -Suppress the _____________ system Results in arterial ____________ and _____________stroke volume MAY CAUSE _____ COUGH ARBS: -Block angiotensin II receptors and cause a _____________ in arterial resistance and decrease BP ACE inhibitors and ARBs both block aldosterone. When aldosterone is blocked we lose water and sodium and retain ____________ (monitor for this) It is standard to send patient with HF home on ACE inhibitors and/or beta blockers. WHY??

Renin Angiotensin system (RAS) dilation and increase SV DRY COUGH ARBs= decrease Potassium (monitor for hyperkalemia These drugs decrease workload, prevents vasoconstriction (decrease afterload). This increases the CO and keeps blood moving forward out of the heart (that is what we want)

Order of obtaining vital signs in pediatric population??

Respiration, Heart rate, BP, Temperature

The central venous pressure is measured where? the CVP number goes up during ______? more volume= more ____________.

Right atrium FVE more volume=more pressure

The natural pacemaker of the heart is the _______ -it sends impulses and makes the heart contract If heart rate drops to 60 or below, ____________ can decrease. -Pacemakers are used to increase the heart rate with _______________ bradycardia

SA node (sinus node) SA node- heart contraction Cardiac output symptomatic

What is the dangers of smoking while pregnant??

SGA, low birth weight, cleft lip/palate, and risk for placental abruption doubles with smoking

What makes the urine specific gravity, sodium levels, and HEMATOCRIT concentration go up?

SIADH, Dehydration

WHat is worse STEMI or NSTEMI?

STEMI ST stegment elevation Myocardial infarction- this indicates the client is having a heart attach and the goal is to get them to the cath lab for PCI in less than 90 minutes NSTEMI- non st stegment elevation myocardial infarction- these are less worrisome

lateral curvature and rotation of the spine?

Scoliosis

ANP (atrial natriuretic peptide) responsibility?

Secreted in response to excess blood volume in the heart, promotes salt and thus water excretion. release with increase BP - kidneys excrete more Na+ and water, thus decrease BP

hyperparathyroidism= hypercalemia= hypophosphatemia -To much PTH. Serum calcium is high. Phosphate is low. S/S- ___________ cause of to much Ca. Treatment- partial parahyroidectomy (removal or ___ of the parathyroids)....PTH secretion will ________. What are you going to monitor post op?? Hypoparthyroidism is the opposite effect. patient will not be sedated due to decrease in Ca. Treatment includes ____ calcium, ___________ binding drugs

Sedation removal of 2 of the parathyroids. PTH will decrease monitor for hypocalcemia (tetany, tight rigid muscles, tingeling fingers, LARYNGOSPAMS) Hypoparthyroidism- IV calcium, phosphorus binding drugs

For a CT scan and cerebralangiography assess if the client is allergic to ?

Shellfish , iodine

With FVD (hypovolemia: Big time deficit = __________

Shock

MI What is the #1 sign of MI in the elderly?? Cardiac output is going ___________ Skin feels ___________ and ___________ and BP will _____________ Vomiting may occur which stimulates vagus nerve. Vagus nerve causes the HR to ___________ and BP to _________

Shortness of breath Down cold and clammy, BP drops HR decreases and BP decreases

________________ is Diagnosed when head is visualized but the body cannot be delivered, even after maneuvers. "__________ " where fetal head after it is delivered, retracts back against the perineum. _________ maneuvers (hyperextend legs) or Mazzanti techniques (suprapubic pressure). Never apply fundal pressure. The primary healthcare provider must do this or call another primary healthcare provider to assist.

Shoulder dystocia Turtle Sign McRoberts Maneuvers

This is a hereditary form of anemia in which the normal hemoglobin is partly or completely replaced by abnormal hemoglobin and sickle shaped RBC's

Sickle cell

with a mononucleosis disease the _________ will be enlarged, so limit participation in _____________ to prevent injury

Spleen contact sports

what is the cause of mastitis?

Staphylococcus bacteria poor hygiene, or inappropriate breast feeding

Cystic Fibrosis __________ is a term for fatty, frothy stools seen in clients with CF. the stools have the appearance because client has such poor intestinal absorption -They do not absorb fat well and __________ soluble vitamins may be given. ADEK - _____________ test is the diagnostic test for CF -In the Newborn what is the earliest sign of CF?? -With CF the mucous secretions are _________ and __________ -CF is a inherited disease, but you MUST get the gene from _________ parents. This makes it an autosomal __________ disorder

Steatorhea Fat Sweat chloride Meconium Ileus- can not pass meconium thick and sticky (with CF THINK thick and sticky secretions everywhere!!) both parents- autosomal recessive disorder

Adrenal glands are needed to handle __________. Two parts: adrenal ____________ and adrenal ____________

Stress adrenal medulla and adrenal cortex

____________ hematoma: Subdural hematomas are a collection of blood between the dura and the brain. usually a _______ bleed. can be acute, subacute, or chronic

Subdural venous bleed

If left main coronary artery occlusion occurs: THINK _______________ What procedure is done for multiple vessel disease or left main coronary artery occlusion. it can be done as a scheduled procedure or emergency?

Sudden death or widow maker The left main coronary artery supplies the ENTIRE left ventricle CABG- basically open heart surgery

__________ heart failure is when heart can NOT contract and eject __________ heart failure is when ventricles can NOT relax and fill

Systolic Diastolic

Cyanotic Defects: 4 T's

T- Tetralogy of Fallot T- Truncus Arteriosus T- Transportation of the Great Vessels T- Tricuspid Atresia

Thyroid produces what three hormones?? Role of calcitonin?? You need ______ to make hormones (dietary) Thyroid hormone gives us ___________.

T3, T4, calcitonin decrease serum calcium levels by taking calcium out of blood and pushes it back into the bone. Need iodine Gives us energy

what is examples of true labor vs false labor?

TRUE regular contractions contractions that are increasing ***discomfort in the back that radiates to the abdomen pain increases with change in activity FALSE irregular contractions discomfort just in the abdomen pain decreases with change in activity

autosomal recessive disorders

Tay-Sachs, Cystic fibrosis, sickle cell anemia, phenylketonuria (two parents)

Hypercyanotic spells seen in childredn with Tetraology of Fallot

Tet spells

Consists of 4 defects: a. Ventricular Septal Defect (VSD) b. Pulmonary Artery Stenosis c. Overriding Aorta d. Right Ventricular Hypertrophy Signs and Symptoms: - Infants may be cyanotic at birth: others may have mild cyanosis that progressively worsens during first year - Murmur - Acute cyanotic or hypoxic spells (blue spells/Tet spells) *usually seen during crying, after feeding, during bowel movements *at risk for sudden death, seizures - Older children: Squatting, nail clubbing, poor growth, exercise intolerance Treatment: - Surgery

Tetralogy of Fallot decreases blood flow= cyanotic

what is tidaling with chest tubes and why do we want to see this?

The fluctuation when you see a slight rise and fall or water in the water seal tube as the client breaths. fluctuations will stop when the lung has re-expanded, or if there is a kink/clot in tubing, or a dependent loop is present in the system

Beta blockers (can prevent angina)- "olol" How do beta blockers work?? What do Beta blockers do to BP, pulse, and myocardial contractility?? What does this do to the workload of the heart?? We just decreased the contractility...so what happens to CO?? This is a good thing to a certain point, because when we decrease the work of the heart, the need for oxygen is decreased, and that decreases angina. But could we decrease the clients CO (HR and BP) too much with these drugs??

They block beta cells. these are the receptor sites for catecholamines (epi and norepi) Decrease BP. Pulse, contractility Decrease workload Decrease CO Yes. monitor HR and BP so it does not decrease to much.

Cushing's syndrome: S/S related to too much glucocorticoids?? -- _______ extremities/skin (lipolysis), __________ risk of infection, _______glycemia, Moon face, _______ obesity, buffalo hump Related to Sex hormones? --_______ skin, women with _______ traits Related to Mineralocorticoids? - High ____, CHF, Weight _______, Fluid volume _________

Thin extremities, increase risk of infection, hyperglycemia, truncal obesity - oily skin/acne, women with male traits (hirtusim- facial hair) -High BP, weight gain, FVE

How is topical agents applied to a burn: ______ layer using _____ gloves.

Thin layer using sterile gloves

_______________- is defined by fluid going to places that is NO good. Fluid does not belong (hypotension). Fluid is out of the vascular space and into places like tissues,etc.

Third Spacing

With FVE the vessels be come full and eventually can not hold anymore so the leak into the tissues and cause peripheral edema and ___________.

Third spacing

What is ectopic pregnancy? What is the first sign? -______________ is given to stop the growth of the embryo in order to save the tube. _If this does not work then a ____________ incision is made into tube to remove the embryo. the entire tube may be removed. A ___________ is done if the tube has ruptured of if the pregnancy is advanced

This is a gestation outside of the uterus usually occurring in the Fallopian tube -Pain is first sign. Client will usually exhibit the usual signs or symptoms of pregnancy, then pain, spotting, or maybe bleeding into the peritoneum. If the fallopian tube ruptures, vaginal bleeding may be present. -Methotrexate: given to stop the growth of the embryo in order to save the tube. If methotrexate does not work, a laparoscopic incision is made into the tube to remove the embryo. The entire tube may be removed. A laparotomy is done if the tube has ruptured or if the pregnancy is advanced.

what is tocolytics?? Examples: -______________: relaxes the uterus (IV) -______________: helps fetal lung maturity (IM) -______________: give SQ -______________: (PO) -______________: (PO)

Tocolytics help stop preterm labor -Magnesium sulfate: Relaxes the uterus -Bethamethasone- fetal lung maturity -Terbutaline -Indomethacin -Nifedipine

After thyroidectomy keep _______ set at bedside. If signs of hypocalcemia. assess for ___________ removal. ----Signs would include?? would this patient seem sedated or non-sedated???

Trach set at bedside Assess for parathyroid removal ---Tetany, muscle rigidity, tingeling fingers, seizures, Non- sedated.

Total larygectomy (removal of vocal cords, epiglottis, and thyroid cartilage). Since the whole larynx (including epiglottis) is removed the client will have a permanent ______________ or laryngectomy. Position client post op??? NG feeding to protect the suture line. Peristalsis (the involuntary constriction and relaxation of the muscles of the intestine or another canal, creating wave-like movements that push the contents of the canal forward) can disrupt the ________ line Watch for _________ artery rupture. Frequent _______ care to decrease bacterial content. NPO people tend to get pneumonia. Humidified environment helps. With total laryngectomy ALL breathing is through the ________. Can the client with total laryngectomy: Whistile? Drink through straw? Smoke? Swim?

Tracheostomy 35-45* Mid fowlers Suture line Carotid Mouth- pneumonia due to bacteria accumulation in the mouth that is not being washed down by food or drink Stoma Whistle- no straw-no smoke- yes (not recommended though_ Swim- NO- may drown

Most sensitive marker for MI? This cardiac biomarker is high specificity to mycaordial damage - Elevates within ____ to ____ hours and remains elevated for up to ____ weeks

Troponin 3-4 hours 3 weeks

Hemoptysis is a late sign of lung cancer or ?

Tuberculosis Not a sign of pneumonia. Pneumonia usually has a unproductive cough

During FVD: Hypovolema Urine output will ____________ BP: _____ (less volume = less pressure) Pulse:____, will feel weak and _______ Respiration: ______ CVP: ______ Cool extremities due to peripheral ____________ in the effort to shunt blood to the vital organs. Urine Specific Gravity will ___________. if putting out any urine at all it will be very concentrated

UO: Decrease (kidneys either aren't being perfused or they are trying to hold on to every bit of fluid) --BP: Decreased --Pulse: up (heart trying to pump what little fluid left to compensate), feels weak and thready --Respiration- increase CVP: Decreases Cool extremities due to peripheral vasoconstriction USG: increases

Signs of UTI

Urgency, frequency, burning, odor, color, blood and confusion

What are observable signs of respiratory distress in children?

Use of accessory muscles Nasal Flaring Sternal Retractions Grunting with respiration's

What is MRI (magnetic resonance imaging)?

Uses magnetic fields and radio waves (better for soft tissue than CT)

What is VEAL CHOP?

V;Variable Decel ---reposition mom---C;Cord Compression E; Early Decel----------Expected------- H; Head Compression A; Accels----reassuring----------------O;O2 adequate (okay) L; Late decel-O2, no pitocin, MD, reposition- P; Placental Insufficieny (problem) *early decelerations: (Not bad) benign- caused by physiological hypoxia from fetal head compressions *Late Dcels: (bad) caused by placental insufficiency *Variable Dcels: (Bad)- caused by umbilical cord compression

Iodine compounds: -Potassium iodine. Decreased the size and the __________ of the gland -All endocrine glands are very __________. -Give iodine in milk or juice, and use a straw. WHY??

Vascularity VERY VASCULAR (bleeds easily) - may stain teeth

Calcium Channel Blockers (To prevent angina), "Dipine", diltiazem, verapamil They cause ______________ of the arterial system What do they do to BP?? They dilate ___________ arteries Two benefits are they ______________ afterload and __________ oxygen to the heart muscle

Vasodilation Decrease BP dilate coronary arteries Decrease afterload and increase oxygen

right sided heart failure is when blood is not moving foward into the lung and flows backwards into the ____________ system

Venous 1. Jugular Vein Distention 2. Ascending Dependent Edema 3. Weight Gain 4. Hepatomegaly (Liver Enlargement)

This is when there is an opening between the left and right ventricle (in the septum) This increases the volume on the right side of the heart. The right side is having to pump harder so this can lead to right sided heart failure. Many close spontaneously during the first year of life, but if not will have surgical repair. Signs and Symptoms: - Signs of heart failure - Murmur

Ventricular Septal Defect (VSD):

Most chemo drugs are given IV central line, A ________ is a type of chemo drug that if it infiltrates will cause tissue necrosis S/S of extravasation: Pain, Swelling, and NO _________ return Number 1 thing to remember with extravasation is to ___________. If extravasate does occur... __________ the infusion and send for extravasation kit Stay with the client the ENTIRE TIME

Vesicant Prevention STOP the infusion

Hypokalemia: Causes: ___________, _____ suction (potassium is found in the stomach), Diuretics, Not eating S/S: __________ cramps, ________ weakness, arrythmias EGC changes?? Treatment: Give potassium, Spironolactone makes client retain potassium, eat more potassium (spinach. potato, oranges, bananas, cucumbers, bell peppers. avocado)

Vomitting, NG suction, also may be caused be IV insulin therapy (may need to give potassium with insulin therapy) muscle cramps. muscle weakness ECG- U waves, PVCs, Ventricular Tachycardia

Four Signs of Heart failure??

Weight gain, shortness of breath, confusion, ankle edema

pacemaker: Always worry when Heart rate does what?? What is the set rate mean? A demand pacemaker kicks in only when client needs it A fixed rate pacemaker fires at a fixed rate constantly Is it okay for the heart rate to increase past the set rate?

Worry when HR drops below set rate Set rate- pacemaker maintains a certain minimal HR depending on settings yes the HR can increase past the set rate, but never decrease.

Do you deflate the tracheostomy cuff prior to giving the client solid foods to make it easier to swallow?

YES Then after deflation suction the client before they eat to reduce the risk of aspiration

Depression: Could they need help with self care? Prevent _________. Encourage ___________. As depression lifts, what happens to suicide risk? A sudden change in mood towards the better may indicate what?? what cultural group has highest suicide rate?? __________ patients are particularly at risk for increased suicide.

Yes Isolation, interaction (may need to seek out the client, prevent isolation) Increases Decisions to commit suicide American Indians Elderly Men- more lethal methods

Does taking Tylenol regularly or overdose on Tylenol cause concern for bleeding precautions?? what about liver disease? Other drugs that require bleeding precautions i, nclude "Rin" and "Ban" drugs. Heparin, Warfarin, Aspirin (acetylsalicyclic acid), Apixaban, Clopidogrel Should we do an ABG on clients on thrombolytic? Would you want to put an IV in a Jugular vein or peripheral vein, given the choice??

Yes bleeding precautions for both No- Sticking an artery when on thrombolytics can cause severe bleeding Peripheral- want an area that is easy to put pressure on. can not really put pressure on the jugular area

Is MMR a live vaccine?

Yes. But not transferrable from child to pregnant mother because it does not shed.

Chlorpromazine is used for?

a drug that reduces the symptoms of schizophrenia by blocking dopamine D2 receptors

Meniere's syndrome is? where is the best position for the nurse to stand?

a rare chronic disease in which the amount of fluid in the inner ear increases intermittently, producing attacks of vertigo, a fluctuating hearing loss (usually in one ear) and tinnitus Stand directly in from of the client when speaking. so the client does not have to turn his head to see nurse and it decreases movement of head, vertigo attacks may be decrease

Hyperosmolar Hyperglycemic Syndrome (HHS) is similar to DKA but not _________ or ____________. Make just enough insulin so they are not breaking down body ______. - NO fat breakdown....no ketones -no ketones..... no acidosis Will they have kussmaul respiration's? Biggest concern with HHS is ???

acidosis or ketones does not break down fat no kussmaul respiration's Dehydration

Immunizations: Tetanus toxoid= _________ immunity. IVIG= __________ immunity

active (takes 2-4 weeks to develop own immunity) Passive (immediate protection)

When a client has burns to neck, face , and chest focus on what?? Provider may perform ____________ prophylactically? --try to consider least invasive first

airway intubation

Asterixis

aka Liver Flap, a flapping tremor of the hands. When the client extends the arms & hands in front of the body, the hands rapidly flex & extend. Affects handwriting that is associated with liver disease

What is an echocardiogram?

an ultrasound of the heart; noninvasive method for monitoring cardiac performance looks at the pumping action or ejection fraction of the heart.

Hematopoietic system (bone marrow): Bone marrow suppression can decrease RBCs, WBCs, and Platelets. Client is at risk for _______, _________, and _________ __________ is the #1 cause of cancer related deaths

anemia, infection, bleeding Infection

Cancer can invade bone marrow and cause ___________ , _____________ & ______________________.

anemia, leukopenia, thrombocytopenia

Respiratory alkalosis can be caused by acute __________ overdoes. the client is breathing too fast and removing too much CO2

aspirin

Echolalia? Neologism?

automatic and immediate repetition of what others say Making up new words- may need to seek clarification. "I dont understand"

A pt on warfarin should avoid

black cohosh and the 4 G's

Treatment for emergency management of burns: -Wrap in ____________ to stop burning process (this also holds in body heat and keeps out nasty germs) -__________water (no more than 10 minutes) -Remove _________ cause swelling will occur, also metal jewelry gets hot. -should clothes be removed??

blanket cool water. Jewelry Remove non adherent clothing (not sticking to the skin) and cover with clean, dry cloth

what is hemothorax/pneumothorax

blood or air accumulate in the pleural space and then collapses the lung

Insulin therapy decreases ___________ and __________ - they need to be on _______ monitor. During DKA regular insulin IV is given with NS. then when blood sugar gets down to about 250 to 300 mg/dL switch to _____ to prevent hypoglycemia. Anticipate provider will want to add ________ to the IV solution at some point.

blood sugar, potassium- drives then out of vascular space and into cells ECG/ heart monitor D5W. Potassium.

Parathyroid hormone secretes PTH which makes you pull calcium from the _________ and place it in the blood causing serum calcium level to go up

bone

What is internal radiation also called?? Brachytherapy is used to get the radiation close to the cancer or target tissue. With all types of brachytherapy does the client emits ___________ for a period of time and is hazard. Brachytherapy can be sealed or unsealed. Unsealed means? Sealed or solid means?

brachytherapy radiation Unsealed- Client AND body Fluid emit radiation Sealed- Client emits radiation, but BODY FLUID are NOT radioactive

adrenal medulla hormones include??? How does this hormones affect: BP, HR, Palpitations? Problems with the adrenal medulla can be Pheochromocytoma (Benign tumor that secretes ___________ and ________ in boluses. tends to be familial, so screen the family ----Can this cause headache? ----May also see flushing/ extremely ___________

catecholamines (epinephrine and norepinephrine) Increase BP, HR, Palpitations Secretes epi and norepi --Headache- yes --flushing and extremely diaphoretic

Treatment for hypomagnesemia: -give mag, but check ______ function (before and during IV Mg) -seizure precautions, eat magnesium (Green leafy veggies) **What to do if client reports flushing and sweating when start IV Mg?

check renal function Stop the transfusion

Which two clients would the Rn assign to the LPN? 1. A newly diagnosed diabetic who has a quarter size food ulcer, pedal pulses are present, but weak, and morning glucose is 200 mg/dL 2. A post-partum client delivered 12 hours ago. her blood pressure was 118/70 mm Hg at 8 am. At 10 am her blood pressure is 140/90 mm Hg and now at 11 am the blood pressure is 150/96 mm Hg.

choice 1 is the most stable

Hemophillia is an inherited condition in which:

clotting factors do not work properly mother transfers it to her son

Adherence means:

commitment

When taking prenatal vitamins what is the biggest complaint with IRON?? Always take iron with vitamin ___. Folic acid prevents ______________ defects Daily dose of folic acid should be??

constipation and GI upset vitamin c neuro tube defects (spina bifida, Myelomenigocele) 400 mcg/day

RSV is what kind of precatuions

contact

What is the definition of premature/preterm labor?? what is the goal during premature/perterm labor?? Treat any existing vaginal or urinary ___________ and ___________ the mom will often stop preterm labor Will client be placed on bedrest??

contractions WITH dilation between 20-37 weeks Goal: stop labor treat infections yes

_____________ disease is the most common type of cardiovascular disease. It is a broad term that includes chronic stable angina and acute coronary syndrome

coronary artery disease

Hydrotherapy is used to ____________ wounds. most important thing to provide to client before hydrotherapy???

debride pain management

Sickle cell crisis: _________ blood flow- __________ oxygen- pain -What is the most important intervention??? -_________ transfusions, antibiotics, analgesics, -____________ is a chemo drug to reduce pain episodes

decrease blood flow, decrease oxygen, pain -HYDRATION and oxygen -blood transfusions -Hydroxyurea

Cushing's syndrome: Since client has too much mineralcorticoids the potassium will be _________. Treatment: -____________ectomy (unilateral or bilateral), If both are removed LIFETIME REPLACEMENT IS NEEDED -__________ environment. -Avoid __________. -Diet pre-treatment: ______ potassium, _____ sodium, _____ protein, ______ calcium

decreased potassium, increase sodium (too much aldosterone) Adrenalectomy -quite environment -avoid infection -Increase potassium, decrease sodium, increase protein, increase calcium (long term steroid use can case osteoporsis cause it pulls Ca from bones, Steroids decrease serum Ca by excreting in through GI tract.)

priority treatment for V fib?? If defibrillation doesn't work, what is the first medication or vasopressor we give? ______________ is an anti arrhythmic and is used when V Fib and pulseless VT are resistant to treatment, and also for fast arrhythmias What anti arrhythmic drugs are commonly given to prevent second episode of V FIB? _____________ and __________

defibrillation (Defib to VFib with CPR in between DeFib) Epinephrine Amiodarone Amidarone and Lidocaine

Hemolytic transfusion reaction means?

destruction of red blood cells as a result of transfusion of incompatible blood

Bulimia Nervosa (BN): -Teeth have erosion of ___________ -Laxatives and diuretic use occures -Weight will be __________ Treatment: -sit with client at meals and observe for 1 hour after __________ -Allow ____ minutes for meals

enamal normal weight eating 30

Carbamazepine indications

epilepsy, bipolar disorder, trigeminal neuralgia (treats agonizing pain)

For generalized anxiety disorder (GAD) re channel anxiety through ___________

exercise pick answers that give most exercise and activity to get energy out (ex. kick boxing etc. )

What is quickening?

fetal movement

How often should a pregnant client visit the primary healthcare provider: First 28 weeks ? 28-36 weeks ? After 36 weeks ?

first 28 = once a month 28-36- every two weeks or twice a month after 36 weeks- once a week *high risk clients will have more prenatal visits

-In profound coma, the pupils will be ________ and _________.

fixed and dilated

Treatment for FVE: Restrict ___________ and low __________ diet Measure ___ & ___ and daily ________ Give __________ medication

fluids. low sodium diet. (sodium causes water retention) I and O, weight Diuretics (Furosemide, Bumetanide, Spironolactone, Hydrochlorothizide)

When is digoxin used? what is the normal Digoxin level? Contractions are __________ Heart rate is ___________. This gives the ventricles more time to fill with blood Cardiac output and kidney perfusion will __________

for a pt with sinus rhythm or a-fib that is accompanying chronic HF Level- 0.5-2 ng/mL Contractions are stronger (increased) HR is slowed CO and Kidney perfusion will increase

cachexia means

general weight loss, wasting

Chemo drugs can be given orally. WEAR ________ when administration. ____ pairs. one under gown, one over gown cuff Chemotherapy is excreted ____ to ____ days after administration. Disposal of chemotherapy should be done in a: __________ rigid chemotherapy waste container. used for sharps and IV equipment

gloves (not latex gloves, special thicker, longer gloves) **2 pairs 3-7 days yellow container

nephrotic syndrome -Losing WAY more ________ than glomerulonphritis -It is an inflammatory response in the glomerulus which causes big ______ form so _______ starts leaking out in the urine. Now the client is ___________ (low albumin in the blood).

group of clinical signs and symptoms caused by excessive protein loss in urine. -big holes so protein starts leaking. now the patient has hypoalbuminemic

Alopecia means

hair loss

In the rule of nines, the head gets____, front trunk gets_____, the posterior trunk gets_____, each leg gets ______, each arm gets _______ and the genitalia gets________.

head and neck- 9 Trunk (front and back)= 18 each arm- 9 each (4.5 each side) leg- 18 each ( 9 each side_ Genital- 1

TACHYCARDIA + POSTPARTUM think what?

hemorrhage

placenta previa risk factors??

history of placenta previa prior c-section multiparity advanced age tobacco use

what is the purpose of balloon tamponade? how long can it be used?

hold pressure on bleeding varices --no more than 12 hours

To prevent chin to chest contractures after burns, the nurse should _________ the clients neck, To prevent other contractures: wrap each finger ___________, use splints

hyper extend separately

Insulin carries glucose out of blood and into cell...when there is no insulin, the glucose just builds up in the blood. the blood becomes ___________ and pulls fluid into the vascular space...the kidneys filter excess glucose and fluids causing _________ and ___________. the cells in turn are starving to break down protein and fat for energy (polyphagia).... when this occurs ketones form. Now the client is ____________ and kussmaul respiration occur.

hypertonic causing polyuria and polydipsia metabolic acidosis Kussmaul respiration's = Kussmaul breathing is a type of hyperventilation that is the lung's emergency response to acidosis. Kussmaul breathing causes a labored, deeper breathing rate. It is most commonly associated with conditions that cause metabolic acidosis, particularly diabetes

Addisonian crisis = severe _________ and vascular collapse Can occur with __________, emotional _________, ________ exertion, or stopping steroids abruptly

hypotension infection, emotional stress, physical exertion, or stopping steroids abruptly

If a client has anemia focus on __________. Thrombocytopenia focus on ________ and avoid ____ injections Leukopenia focus on _________ and ___________

hypoxia bleeding. avoid IM injections Bleeding and Infection

Neuropathy related to diabetes can cause sexual problems like ___________/ decreased sensation Foot care: -Cut toenails straight ________ - Dry toes completely - Well fitted shoes - Inspect feet daily - No harsh chemiclas

impotence straight across

Anhedonia

inability to feel pleasure, loss of pleasure

Agnomia

inability to name objects

Apraxia

inability to perform particular purposive actions, as a result of brain damage.

If headache or vomiting occurs after head injury, assume _____?

increased ICP

-itis means

inflammation

glomerulonephritis means? Antibodies lodge in the ___________; causing scarring and decrease filtration Main causes??

inflammation of the glomeruli of the kidney (inflammation of filter in the kidneys) Glomerulus Strep, mono, skin infection

Beta blockers (propranolol) Works by inhibiting __________ and _____________. this decreases myocardial contractility, Decrease cardiac output, decrease HR and BP, and can be used as supportive therapy to decrease _____________. ****TESTING STRATEGY- DO NOT give beta blockers to ___________ or _____________ clients.

inhibits epi and norepi Anxiety Not for Asthmatic or diabetic clients **Asthma and chronic obstructive pulmonary disease (COPD) have been classic contraindications to the use of beta blockers because of their potential for causing bronchospasm **Search Results Featured snippet from the web Beta-blocker use in patients with type 2 diabetes mellitus (T2DM) and established cardiovascular (CV) risk factors is associated with an increased risk of CV events and severe hypoglycemia, according to a recent study

Hypertonic solutions: volume expanders that will draw fluid into _________ space from the _________ Examples include? Uses: for hypoatremia or a client who has shifted large amounts of vascular volume to a 3rd space or has severe edema, burns, ascites. ALERT: watch for fluid volume ______. Monitor ICU setting with frewuent monitoring of Blood pressure, pulse, and CVP.

into vascular space from the cells TPN, Albumin, D10W, 3% NS, 5% NS, fluid volume excess

When a baby is born, they have an ____________ grasp reflex

involuntary.

Direct Coombs test is done on the _________. tells if there are any antibodies stuck to the RBCs

is done on infant blood to determine the presence of antibody-coated RBCs, Bilirubin levels of the infant blood indicate the extent of RBC destruction Done on the CORD

for thyroidectomy teach how to support the neck: -Put personal items ________ to them -Position HOB: _________. to prevent _________ in airway -Nutrition pre & post op: Client needs __________ calories.

items close to them HOB elevated to prevent edema in airway More calories

for hypoglycemia give patients: Dextrose water D50W is hard to push, if you have a choice you need a __________ IV Injectable _________ . used when there is no IV access, Given _____

large bore IV Injectable glucagon, given IM

Less blood volume = ___________ CO More blood volume= __________ CO

less= decrease more= increase

A person to acidic can appear __________ and go into __________ and death. Too alkalosis can appear __________ and have _________ and death

lethargic, coma excitable, convulsions

Pancreatitis Diagnosis - Serum ______ and _______ go up -WBC ______ -Blood sugar _________ -PT, aPTT are ________ -Serum Bilirubin ________ -H&H can go up or down ----what would cause it to go down __________, What causes it to go up __________

lipase (most specific for pancreatitis), Amylase - WBC, Blood sugar increase -PT, aPTT- prolonged -Bilirubin- increase -H&H. Down= bleeding, Up= dehydration

Treatment for FVD: --Prevent further _____ of fluid -- Replace volume: if mild deficit may be _____ fluid If severe deficit may be ___ fluid --Safety precautions: Higher risk of ______. --Monitor for _______ with IV fluid replacement

loss mild- PO Severe- IV Risk of falls Monitor for overload

Apraxia means

loss or purposeful movement in absence of motor or sensory impairment.

oprelvekin is used for

low platelet count. to stimulate growth factor.

During FVE: -Describe lung sounds expected? -Pulse will ___________ and feel _________ -If fluid does not go forward, it's going to go backward into the lungs, can lead to _____ and pulmonary edema -With fluid retention think ___________ problems FIRST

lungs- wet/ crackles (base of the lung first, sit patient up so gravity can pull fluid to base, heard best in this position Pulse will increase and be bounding (full) can cause HF HEART

what are the two poles of Bipolar?? S/S of bipolar: -Continuous _________ -Flight of _________ -Delusions are just a false _______ --- Delusions of __________: Example client thinks they are jesus -constant motor activity can lead to __________ -Poor __________ - Manipulation makes them feel secure and ___________

mania, depression high ideas false ideas Grandeur exhaustion judgement powerful

Neutropenia is a decrease in the number of neutrophils (_______ WBCs) in the blood Assess neutopenia by calculation the ______. Treatment for neutropenia: __________

mature WBCs ANC Antibiotics

Indirect Coombs test is done on the __________. Measure the number of antibodies in the blood

mother: measures number of antibodies in the blood. Identifies clients sensitized to Rh-positive blood Test repeated between 24 to 28 weeks of gestation for clients who are Rh-negative and not sensitized

decerebrate vs decorticate posturing: Anytime you have these responses to painful or noxious stimuli it indicates that the__________ response centers of the brain, the mid brain, and the brain stem are compromised -Decorticate: present with arms ___________ inward and bent in towards the body and the legs are extended. -Decerebrate: present with all four extremities in rigid _______________. this is worse. Client will be rigid, tight and burning more ___________ -so they need??

motor flexed extension calories, need tube feedings

After radiation can the client: -Sleep in same bed with spouse or children? -Use public transportation? -Return to work immediately? -Share utensils or cook for others? -will one flush of toilet after use be adequate?

no to all. should keep 6 feet apart 11-14 days flush 2-3 times

When is Rhogam given? -In NCLEX word Rho (D) immune/globulin (Rhogam) must be given _____________ the antibodies form

o Given at 28 weeks and also within 72 hours of delivery o Also given after abortion, ectopic pregnancy, amniocentesis -Before the antibodies form. it will destroy positive fetal blood cells that get into mother's negative blood.

For Thyroid scan, client must discontinue any iodine containing medication ___ week prior to scan, and must wait ___ weeks to restart medication. Ultrasound, MRI, CT can be used also.

one week, 6 weeks

External radiation (teletherapy, radiotherapy): -A carefully focused beam of high energy rays is delivered by machine ________ of the body -Client is NOT __________. Side effects include: -Erythemia, shedding of skin, _______ (#1 complaint), ____________ (all blood components are decreased. Signs and symptoms are ________ and _________ related. -Is it okay to wash off markings?? -Is it okay to use lotion on the markings?? -Protect site from ________ and UV exposure for ____ year after completion of therapy. -Provide good skin care

outside the body NOT radioactive Fatigue Symptoms are location and dose related Do NOT wash or use lotion on markings Protect from sunlight, for one year

cardiac catherization (CC) is what? Ask if they are allergic to??? Also check ________ function, because the dye is excreted through the ____________. ---____________ may be given pre procedure, especially if kidney problems, this helps to protect the kidneys Are palpitations normal? Is feelings or warmth and flushed all over normal? After procedure monitor _______, --watch puncture site for? ___________ and __________ formation Bed rest, keep extremity s__________ for _____ hours. Major complication post cath? ____________ Report __________ or __________ ASAP if client is on metformin ______ this medication for 48 hours post procedures cause we are worried about the _________.

passage of a catheter into the heart through a vein or artery to provide a comprehensive evaluation of the heart Allergy to shellfish or iodine Check Kidney function Acetylcysteine for protection (this is what is given for Tylenol overdose) Yes these are normal Monitor VS. watch for bleeding and hematoma formation Keep extremity straight for 4-6 hours Major complication is bleeding Report pain and pressure Hold metformin. worried about kidneys

Ejection Fraction (EF)

percentage of blood volume in the ventricles at the end of diastole that is ejected during systole; a measurement of contractility, Measures the volume percentage of left ventricle contents ejected with each contraction

Major complication of peritoneal dialysis? Increase what in the diet??

peritonitis (abdominal pain and cloudy effluent) -Fiber- decreased peristalsis due to abdominal fluid - bi holes in peritoneum and lose protein with each exchange

Painless bleeding in 2nd half of pregnancy (may be spotting or may be profuse) is a sign of?? Treatment includes?

placenta previa Home care with bedrest if no active bleeding. Complete previa usually requires hospitalization (from as early as 32 weeks until birth) to prevent blood loss and fetal hypoxia if client goes into labor. Fetal monitoring. If there's not much bleeding: bedrest and watch closely. Rule out other sources of bleeding such as abruption. Pad counts. Monitor blood count and monitor FHTs closely. Monitor for contractions: call the primary healthcare provider. Delivery method of choice: C-section. Do not perform vaginal exam.

What disease is marked by inflammation of the lungs. Cause can be bacterial, viral, fungal, or aspiration of foreign substances can have fine crackles or rhonchi, with cough that is either productive or nonproductive -back or abdominal pain referred pain from lungs -Fever, usually very high

pneumonia

Droplet precautions have what?

pneumonia, influenza, whooping cough, bacterial meningitis

if you have a rupture of membranes what is the biggest complication to consider?

prolapsed cord- they should go to hospital with rupture of membranes Infection can occur but takes more time. a prolapsed cord is an acute emergency so highest priority

An 18 month old is admitted to the Ed with diagnosis of rotavirus and severe dehydration. The client has no tears and no wet diaper in 5 hours. the provider has prescribed D5 1/4 NS with 20 mEq of KCL at 20 mLs per hour per pump. what would be the initial action by the nurse?

question potassium order since this baby is not urinating and increases risk for toxicity

Bone marrow produces what?

red and white blood cells

ANP (atrial natriuretic peptide) released with increase BP to cause Kidneys to excrete more NA and water, thus causing BP to _____________

release with increase BP - kidneys excrete more Na+ and water, thus decrease BP

Five Rights of Delegation

right task right circumstance right person right direction/communication right supervision/evaluation

Pyelonephritis can cause ________ shock

septic (hypotension, tachycardia, fever)

When you have polyuria think _____________ first. Losing volume

shock

When you see someone with polyuria think _________ first

shock. losing fluids

Hyperthyroidism: S/S: _________ attention span, weight________, Sweaty/ hot, ______ intolerance, ________ bulging of eye, irreversible), GI _________ perastalsis, BP and pulse _________. Thyroid size is __________. ALSO __________ workload of heart T3 and T4 will be __________, TSH will be __________

short (decreased) attention span, decrease weight, heat intolerance, Exophthalmos, GI increase perastalsis (hyperactive bowel), Increase BP and Pulse, Thyroid is large. Increase workload of heart T3 and T4 are increased, TSH is decreased

Uap's can perform tasks on ________ clients in ___________ situations

stable, uncomplicated situations

Adrenocorticotropic hormone (ACTH) is made in the pituitary and they stimulate ___________ to be made. So no matter what "fancy" word the NCLEX lady uses... you will still get the same result.... think "steroids" Increase ACTH = Increase in Cortisol level Too many steroids= Hypercortisolism (just another word)

stimulates the adrenal cortex to make CORTISOl

Treatment of Bipolar Disorder: -Decrease __________ -Dont argue or try to ___________ with them - set structured ___________ -Provide _________ to replace non purposeful activity - Supply __________ foods. the are too busy to stop and eat. (walk with client during meals) -Don't forget about fluids cause they can become dangerously ____________

stimuli reason schedule activities finger foods Dehydrated

__________- inflammation of the mouth, the oral cavity is very susceptible to irritation due to immunocompromised status

stomatitis

Canes should be used on the _______ side of the body

strong

signs of hypoglycemia What should the client do? how may grams? Glucose absorption is delayed in foods with lots of ____. Fat is not the best choice for simple, fast acting. What is the 15-15-15 rule? -once blood sugar is up, what should they do?

sweating , trembling, tachycardia, confusion, shaky, headache, hunger, decreased LOC, lethargic, Dizzy (cool and clammy you get candy) -Eat or drink simple sugar carb 15 grams of carbs Fats. eat 15 grams wait 15 minutes. if BS < 70 give 15 more grams. Once BS is up eat complex carb and protein like crackers and peanut butter to keep levels therapeutic.

If baby is born with both a cleft lip and palate, which problem do you correct first? why?

the cleft lip promotes feeding and bonding with mo

when spleen is enlarged, the ________ is involved

the immune system is involved

Agnosia

the inability to recognize familiar objects.

General radiation precautions for internal radiation include ____________, ____________, and shielding. Nursing assignments should be rotated ________, so that nurse is not continuously exposed Nurse should only care for ____ client with a radiation implant in a given shift. Private room for client wear a ______ badge at all times, Visitors must stay ____ feet, and limited to _____ minutes a day. NO visitors under 16 years old, no pregnant visitors

time, distance, and shielding Daily one film badge 6 feet, 30 minutes

After airway is established in burn client, client may need at least ______ large bore IV's Fluid replacement includes __________ and _________ solutions for replacement when doing parkland formula (4 x BSA burned x kg) for the first 24 hours. it is based on the time the injury occurred, not when the treatment was started

two Crystalloids (LR), Colloids (Albumin)

Serum potassium will go _______ in metabolic acidosis and go ________ in metabolic alkalosis. So monitor for??? Treatment of metabolic acidosis??

up (acidosis), down (alkalosis), so for metabolic disturbances monitor for muscle cramps, and life threatening arrhythmias Treat the problem, give IV sodium bicarb to help with acidosis

When using crutches upstairs up with the _____ leg, down with the _____

up with the good, down with the bad

Penicillin allergy use what type of antibiotic instead??

use azithriomycin, clarithromycin and clindamycin typically the "mycin" category

Albumin (colloid) works by holding fluid in the _________ space -Vascular volume, kidney perfusion, blood pressure, and cardiac output will all ___________. When the vascular volume increases, the workload of the heart ___________ This will help with FVD. Puts more fluid where it belongs and holds albumin holds it there ALERT: if you stress the heart too much: the client could go into fluid volume ___________, this will cause cardiac output to __________, and lung sounds to sound _________. Measurement to take hourly to ensure that an infusion is not overloading the client? _________

vascular space increase increases Fluid overload, decrease CO, lung sounds wet/ crackles. Measure CVP (right atrium pressure)

IV fluids for FVD: Isotonic: goes into the _________ space and STAYS THERE! --Examples include _________, __________, _____ and D5 1/4. ALERT: Do not use isotonic solutions with hypertension, cardiac disease, or kidney disease. ----Monitor for ___________ in solutions that contain sodium

vascular space and STAYS THERE Examples 0.9% NS, LR (given for shock), D5W Hypernatremia

Nephrotic syndrome Without albumin, fluid can not hold on to fluid in the ________ space. so where does this fluid go? _________. The circulating blood volume does what __________. The kidneys since thins the aldosterone will be produced. This causes retention of ____________ and ___________. but when there is no protein (albumin) to hold fluid into vascular space. So where does this fluid go? ______________ Total body edema= _________ (generalized edema)

vascular space, fluid will go into tissues causing third spacing The blood volume decreases and aldosterone will cause sodium and water to produce. Fluid continues into the tissues to 3rd space Anasarca

_____________ is the nurses duty to tell the truth, which the nurse does if she fully explains the consequences of a planned procedure

veracity

IV fluids should be given slowly for what populations?

very young, the elderly, history or heart or kidney problems

What is stroke volume?

volume of blood pumped out by one ventricle with each beat CO = HR x SV

too much ADH causes ___________ retention. This is known as ______? Urine output _____________ and becomes ____________ (Specific gravity will __________) Blood becomes ________. and Sodium concentration will ___________

water retention (FVE) SIADH (too many letters, too much water) Urine output will decrease become concentrated (increase in SG) blood will dilute, Sodium concentration will decrease. (Urine sodium becomes high though)

How is insulin dose determined: -Initially based on _________ (0.4-1.0 units/kg/day) -Insulin dose is adjusted until ________ is normal and until there is no more glucose or ketones in the urine

weight blood sugar

Addison's disease S/S: Fatigue, N/V/D, weight _____. ________ from losing volume (decrease in sodium, water), ____________ (bronzing of the skin and mucous membranes), _____ patchy area of skin (vitiligo) Treatment: Combat ______ (losing water and sodium) Increase _______ in diet ( processed fruit juice/broth) monitor ___ and ___ and daily ______. Monitor for decrease in blood pressure, Will they lose or gain weight? Fluid volume __________?

weight loss, hypotension, Hyperpigmentation, White patchy skin decrease Na, decrease glucose, decrease fluid, INCREASE potassium - combat SHOCK, increase sodium, monitor I & O and daily weight Lose weight Fluid volume deficit

how often should clinical breast exams be done?

yearly for women 40 years or older 20-39 years need one every three years

Males should have clinical testicular exams _________ and testicular tumors grow very ________ (majority age is 15-36 for testicular cancer. Digital rectal exam prostate specific antigen (PSA) should be checked for men over _____ years of age

yearly, very fast 50

Testing the stool for fecal occult blood should be done how often beginning at age 50?

yearly, unless previous problems or a positive family history

Digoxin: Do you give a digitalizing (loading) dose? How do we know Digoxin is working? Early and late signs of toxicity?? Before administering check the ________? All electrolytes must remain normal, but _____________ is the one that causes the most trouble. __________________+ Digoxin = Toxicity

yes CO goes up Early- GI Related (Nausea/vomiting, anorexia) Late- Arrhythmia and ANY vision changes --Seeing yellow is the most common though Check apical pulse (Left 5th midclavical) Potassium Hypokalemia + Digoxin = toxicity (any electrolyte imbalance can promote digoxin toxicity, but potassium is the worst)

can LPN's -Administer IVPB medication -Administer IVP pain medications, or to blood products -Monitor blood transfusions -Administer via NG tube, G tube or J tube -insert, maintain and remove urinary catheter -Maintain and remove peripheral IV catheters -Calculate and monitor IV flow rates

yes they can except IVP and initiate blood transfusions


Related study sets

Elsevier Nutrition/ DM Quiz- Final

View Set

unit 2 sentence structure and run-ons and fragment

View Set

Life Policy Provisions, Riders and Options

View Set